Property Flashcards

You may prefer our related Brainscape-certified flashcards:
1
Q

Under the Rule of Convenience, …

A

…a class closes when a member of the class is entitled to distribution.

If the class is closed when the interest takes effect (i.e. when the testator died), then all members of the class alive at that time will take, and any members of the class later born or conceived will not take.

T devised to grandchildren who reach the age of 25. A is 27 at the time of the testator’s death and is entitled to immediate distribution. Therefore, the class closes immediately. All grandchildren born when the class closed (A, B, C, and D) will be allowed to take when they reach 25. No grandchildren born after the testator’s death will take.

How well did you know this?
1
Not at all
2
3
4
5
Perfectly
2
Q

Where one tenant makes improvements to the property that may increase the value of the property but are not “necessary,” the other tenant is not liable unless there was an agreement between the parties to contribute to the cost. Thus, a tenant who makes improvements to property cannot seek immediate reimbursement from a co-tenant in a suit for contribution where there was no agreement between the parties to contribute to the cost.

BUT

Can tenant file partition action to recover the increase in value of the property?

A

However, a tenant who makes improvements to a property may file a partition action and request an accounting. If the court grants a partition by sale, then the property will be sold, and the proceeds will be distributed to the tenants. The remaining value will be divided between the tenants according to their shares.

How well did you know this?
1
Not at all
2
3
4
5
Perfectly
3
Q

Rule Against Perpetuities - what is it?

A

No interest in property is valid unless it must vest, if at all, not later than 21 years after some
life in being (“measuring life”) at the creation of the interest.

If there is any possibility that the
interest might vest more than 21 years after a life in being, the interest is void.

How well did you know this?
1
Not at all
2
3
4
5
Perfectly
4
Q

RAP - when does it apply?

A

The Rule applies to
contingent remainders,

executory interests,

vested remainders subject to open (class gifts),

options
to purchase (not attached to a leasehold),

rights of first refusal, and

powers of appointment.

How well did you know this?
1
Not at all
2
3
4
5
Perfectly
5
Q

When Perpetuities Period Begins to Run?

A

For interests granted by will, it runs from the date of the
testator’s death;

for deeds, it is the date of delivery.

The period runs on an irrevocable trust
from the date it is created;

it runs on a revocable trust from the date it becomes irrevocable.

How well did you know this?
1
Not at all
2
3
4
5
Perfectly
6
Q

An interest vests for purposes of the Rule when it becomes:

A

(i) possessory, or

(ii) an indefeasibly
vested remainder or a vested remainder subject to total divestment.

How well did you know this?
1
Not at all
2
3
4
5
Perfectly
7
Q

Interests Exempt from Rule

A

Except for vested remainders subject to open, the Rule Against Perpetuities does not apply to vested interests. Thus, other vested remainders, reversions, possibilities of reverter, and rights
of entry are not subject to the Rule.

Moreover, there is a charity-to-charity exception to the Rule (i.e., the Rule does not apply to any disposition over from one charity to another), and an exception for options to purchase held by a current tenant

How well did you know this?
1
Not at all
2
3
4
5
Perfectly
8
Q

RAP in Operation—Common Pitfall Cases

Executory Interest Following Defeasible Fee

A

Generally, an executory interest that follows a defeasible fee (e.g., “to A for so long as no liquor is consumed on the premises, then to B”) violates the Rule Against Perpetuities, and the executory interest is stricken. (An executory interest following a defeasible fee is valid only if the condition is specific to the fee holder or expressly limited to the perpetuities period.)

How well did you know this?
1
Not at all
2
3
4
5
Perfectly
9
Q

RAP in Operation—Common Pitfall Cases

Age Contingency Beyond Age Twenty-One in Open Class

A

A gift to an open class conditioned on members surviving beyond age 21 violates the Rule.

Example: “To A for life, then to those of A’s children who attain the age of 25.”
The remainder in A’s children violates the Rule and is void.
Some states have enacted perpetuities reform legislation that reduces such age contingencies
to 21.

How well did you know this?
1
Not at all
2
3
4
5
Perfectly
10
Q

RAP in Operation—Common Pitfall Cases

Fertile Octogenarian

A

A woman is conclusively presumed to be capable of bearing children, regardless of her age or medical condition.

Example: “To A for life, then to A’s children for life, then to A’s grandchildren in fee.” The remainder in A’s grandchildren is invalid despite the fact that A is 80 years old.

A few states have enacted perpetuities reform statutes that raise a presumption that women over a certain age (e.g., 55) cannot bear children. Also, medical testimony regarding a woman’s childbearing capacity is admissible in these states.

How well did you know this?
1
Not at all
2
3
4
5
Perfectly
11
Q

RAP in Operation—Common Pitfall Cases

Unborn Widow or Widower

A

Because a person’s widow (or widower) is not determined until his death, it may turn out to be someone who was not in being at the time of the disposition.

Example: O conveys “to A for life, then to A’s widow for life, then to A’s surviving issue in fee.” In the absence of a statute to the contrary, the gift to A’s issue is invalid because A’s widow might be a spouse who was not in being when the interest was created.

Compare: A remainder “to A’s children” would be valid because, unlike issue, they would be determined at A’s death.
Where necessary to sustain a gift, a few state statutes raise a presumption that any reference to a person’s spouse, widow, or widower is to a person in being at the time of the transfer.

How well did you know this?
1
Not at all
2
3
4
5
Perfectly
12
Q

RAP in Operation—Common Pitfall Cases

Administrative Contingency

A

A gift conditioned on an administrative contingency (e.g., admission of will to probate) violates the Rule.

Example: A gift “to my issue surviving at the distribution of my estate” is invalid because the estate might be administered beyond the period of the Rule.

A few state reform statutes eliminate this problem by raising a presumption that the transferor intended that the contingency should occur, if at all, within 21 years.

How well did you know this?
1
Not at all
2
3
4
5
Perfectly
13
Q

RAP in Operation—Common Pitfall Cases

Options and Rights of First Refusal

A

1) Options
An option is a contract, supported by consideration, that creates in the optioned a right to purchase the property on terms provided in the option. Generally, an option to purchase that is structured so that it might be exercised later than the end of the perpetuities period is void. (A significant minority of courts will construe the option as lasting only for a reasonable time.)

Exception: The Rule Against Perpetuities does not apply to options to purchase held by the current lessee.

Watch for a fact pattern on the exam where a tenant has an option to purchase beyond the perpetuities period. Remember that the Rule does not apply to such an option held by a current tenant or his assignee, but it does apply to
a former tenant and to any party to whom the current tenant might transfer the option separately from the lease (in jurisdictions permitting such a transfer).

2) Rights of First Refusal
In contrast, rights of first refusal in many states are presumed to be personal to the holder and therefore not subject to the Rule Against Perpetuities. In some states, however, rights of first refusal are subject to the Rule in the same manner as
options.

How well did you know this?
1
Not at all
2
3
4
5
Perfectly
14
Q

Application of the RAP to Class Gifts

A
**_“Bad-as-to-One, Bad-as-to-All” Rule_**
If the interest of any class member may vest too remotely, the whole class gift fails. For the class gift to vest, the class must be closed and all conditions precedent must be satisfied for every member.
**“Gift to Subclass” Exception**
Each gift to a subclass may be treated as a separate gift under the Rule.

Example: “Income to A for life, then to A’s children for their lives. Upon the death of each of A’s children, the corpus is to be distributed to that child’s issue, per stirpes.” The gifts to each of A’s children’s issue are considered separately. Thus, the gifts to issue of A’s children living at the time of the
disposition are good, but the gifts to the issue of afterborn children of A violate the Rule and are void.

**Per Capita Gift Exception**
A gift of a fixed amount to each member of a class is not treated as a class gift under the Rule.

Example: “$1,000 to each of my great-grandchildren, whether born before or after my death.” This creates gifts to individuals, each of whom is judged separately under the Rule.

How well did you know this?
1
Not at all
2
3
4
5
Perfectly
15
Q

A possibility of reverter is devisable. Example?

A

After the land owner died, leaving her entire estate to her daughter, the daughter is the holder of the possibility of reverter.

How well did you know this?
1
Not at all
2
3
4
5
Perfectly
16
Q

To my son for life, reminder to my son’s children and their heirs. At the time of T’s death, the son had one child. Two years after T’s death, the son had another child.

A

When the testator dies, the son has a present possessory life estate and the son’s first child, who is ascertainable because he/she was born before testator’s death, has a vested remainder subject to open because the remainder was devised to a class: the son’s children.

When the second child is born, the second child is added to the class and also has a vested remainder subject to open because the son could have more children that join the class.

How well did you know this?
1
Not at all
2
3
4
5
Perfectly
17
Q

Under the Doctrine of Worthier Title…

A

…. the land owner retains a reversion in the property. Upon A’s death, the property reverts to the land owner in fee simple absolute.

It applies when A receives an estate less than a fee simple (i.e. a life estate); the land owner’s heirs receive a remainder; both interests created by the same instrument; and both interests are legal.

How well did you know this?
1
Not at all
2
3
4
5
Perfectly
18
Q

A restraint on alienation is a condition on the ownership of real estate that restricts conveyance. A forfeiture restraint provides that the grantee loses the estate if the grantee attempts to convey the property. A forfeiture restraint is permitted for…

A

…life estates and future interests but is not enforceable for fee simple estates. The grant to the son–“to my son and his heirs”–is a fee simple estate.

How well did you know this?
1
Not at all
2
3
4
5
Perfectly
19
Q

When joint tenants mortgage the property together, the mortgage….

A

…does not sever the joint tenancy.

In addition, a joint tenant may sever the joint tenancy by voluntarily or involuntarily transferring her interest in the property, but a joint tenant cannot devise her interest.

How well did you know this?
1
Not at all
2
3
4
5
Perfectly
20
Q

Zoning violation v. zoning restriction

A

While a zoning violation may make title unmarketable, a zoning restriction on the property does not make title unmarketable.

How well did you know this?
1
Not at all
2
3
4
5
Perfectly
21
Q

Under the Uniform Vendor and Purchaser Risk Act, absent a written agreement to the contrary, the risk of loss is placed on …

A

…seller during the time between execution of a binding contract and the closing date.

However, that risk transfers to the purchaser if legal title or possession has passed to the purchaser.

How well did you know this?
1
Not at all
2
3
4
5
Perfectly
22
Q

When a buyer breaches a contract to sell real property, the seller can recover …. consequential damages, such as mortgage interest payments that the seller pays after the buyer’s breach.

A

foreseeable

The seller cannot recover unforeseeable consequential damages.

When a buyer breaches a contract to sell real property, the seller can recover expectation damages. Expectation damages are measured by the difference between the contract price and the market price of the real property at the time of the breach.

When a buyer willfully breaches a contract to sell real property, the seller can recover punitive damages from the buyer.

When a buyer breaches a contract to sell real property, the seller can recover reasonable reliance damages. Reasonable reliance damages include the costs of inspections paid by the seller.

How well did you know this?
1
Not at all
2
3
4
5
Perfectly
23
Q

The right of the mortgagor to pay off the mortgage debt after default but before foreclosure (known as the right of redemption) … be waived at the time the mortgage is created.

A

may not

Redemption is permitted at any time after default and before the foreclosure sale.

Many states permit redemption even after the foreclosure sale, if the redemption occurs within the statutorily designated time (usually six to 12 months after the foreclosure sale).

How well did you know this?
1
Not at all
2
3
4
5
Perfectly
24
Q

The proceeds of a foreclosure sale go first to reimburse….

A

…the costs of the foreclosure sale.

How well did you know this?
1
Not at all
2
3
4
5
Perfectly
25
Q

Assumption

A

If the grantee signs an assumption agreement, he becomes primarily liable to the lender, while the original mortgagor is secondarily liable as a surety.

However, the mortgagee may opt to sue either the grantee or the original mortgagor on the debt.

If no assumption agreement is signed, the grantee is not personally liable on the loan, and the original mortgagor remains primarily and personally liable.

Nonetheless, if the grantee does not pay, the loan may be foreclosed, wiping out the grantee’s investment.

How well did you know this?
1
Not at all
2
3
4
5
Perfectly
26
Q

Priorities

A

A mortgage’s priority is usually determined by the time it was placed on the property.
Foreclosure does not destroy any interests senior to the interest being foreclosed.

It generally destroys all junior interests, but failure to include a junior interest holder in a foreclosure
action results in preservation of that party’s interest.

How well did you know this?
1
Not at all
2
3
4
5
Perfectly
27
Q

Modification of Priority

A

Although priority among mortgages is generally determined by chronology, this priority may be changed by:

(i) the operation of the recording statute if a prior mortgagee fails to record;

  • *(ii) a subordination agreement between a senior and junior mortgagee;**
  • *(iii) a purchase money mortgage;**

(iv) the modification of a senior mortgage (junior
mortgage has priority over the modification);

  • *(v) the granting of optional future advances by a mortgagee with notice of a junior lien (junior lien has priority over
    advances) ; or**

(vi) subrogation (a senior mortgage is refinanced with a new mortgage)

How well did you know this?
1
Not at all
2
3
4
5
Perfectly
28
Q

A purchase money mortgage (“PMM”) is …

A

…a mortgage given in exchange for funds used to purchase the property.

PMMs are given either to the seller as part of the purchase price or to a third-party lender.

PMMs have priority over mortgages, liens, and other
claims against the mortgagor that arise prior to the mortgagor’s acquisition of title.

However, subsequent mortgages or liens may defeat PMM priority by operation of the recording acts.

As between two PMMs, a seller’s mortgage generally has priority over a third party’s. If there are two third-party PMMs, priority is determined by chronological order.

How well did you know this?
1
Not at all
2
3
4
5
Perfectly
29
Q

Proceeds of Foreclosure Sale

A

Proceeds are applied first to the expenses of the sale, attorneys’ fees, and court costs;

then to
the principal and accrued interest on the foreclosed loan;

next to any other junior interests
in the order of their priority;

and finally to the mortgagor.

How well did you know this?
1
Not at all
2
3
4
5
Perfectly
30
Q

A person with a remainder interest in mortgaged property has a duty to pay the …, and the life tenant has a duty to pay the … on the mortgage.

A

principal

interest

How well did you know this?
1
Not at all
2
3
4
5
Perfectly
31
Q

“subject to the mortgage”

v.

“assumes the mortgage”

A

If a mortgagor transfers the property “subject to the mortgage” and mortgage payments are not made, the mortgagee may foreclose and force the property to be sold, but the transferee does not have personal liability for the debt.

In contrast, if the transferee of mortgaged real property “assumes the mortgage” and mortgage payments are not made, the mortgagee may foreclose and force the property to be sold, and, in addition, the transferee is personally liable for any deficiency.

Finally, in a novation, the transferee of real property and the mortgagee agree that the transferee will assume the mortgage and the mortgagor will be released from liability.

How well did you know this?
1
Not at all
2
3
4
5
Perfectly
32
Q

Emblements (or fructus industriales) are ….

A

…annual growing crops resulting from human cultivation.

Emblements are considered part of the land and pass with a conveyance of the land unless the grantor expressly reserves the crop in the deed of conveyance.

i.e.

The farmer conveyed the land to the buyer and did not expressly reserve the pumpkin crop. Therefore, the pumpkins passed to the buyer with the conveyance of the land.

How well did you know this?
1
Not at all
2
3
4
5
Perfectly
33
Q

An easement by estoppel.

A

An easement by estoppel may be created if an act or representation of the owner of the servient estate causes justifiable reliance by the owner of the dominant estate, and the owner of the benefited estate would suffer damages if the easement is not recognized.

How well did you know this?
1
Not at all
2
3
4
5
Perfectly
34
Q

An easement by prescription.

A

An easement by prescription is the easement equivalent of adverse possession. To create an affirmative easement by prescription, the use of the property must be open and notorious, actual, continuous for the prescription period, hostile, and exclusive.

How well did you know this?
1
Not at all
2
3
4
5
Perfectly
35
Q

An easement by necessity.

A

An easement by necessity is created where there is severance of title to land held in common ownership and strict necessity for the easement at the time of severance.

How well did you know this?
1
Not at all
2
3
4
5
Perfectly
36
Q

An easement implied by prior use.

A

An easement implied by prior use is created where there is a severance of title to land held in common ownership;

an existing, apparent, and continuous use when severance occurs;

and the parties intended use to continue after division of land.

How well did you know this?
1
Not at all
2
3
4
5
Perfectly
37
Q

license - transferable?

A

A license is personal to the licensee, and therefore it is not transferable unless the licensor intended that the license be transferable.

How well did you know this?
1
Not at all
2
3
4
5
Perfectly
38
Q

The language in the lease created a restriction on the use of the property. The question is whether the restriction runs with the land to the sublessee.

A

The restriction certainly touches and concerns the land, it can therefore be presumed that it was intended that the restriction run with the land, and the sublessee took with notice of the restriction because the lease was recorded. The problem is with vertical privity between the lessee and the sublessee. On the servient side vertical privity exists only if there is a transfer of the entire interest in the land. The lessee transferred only five years of the 45 years remaining on the lease, so vertical privity is lacking. As a result, the restriction could not be enforced as a covenant, which means the lessor could not recover damages.

However, since privity is not an element of an equitable servitude, the lessor could enforce the restriction as an equitable servitude and get an injunction to prevent the violation of the restriction.

How well did you know this?
1
Not at all
2
3
4
5
Perfectly
39
Q

At common law, if the lease was silent, the …. had a duty to repair.

A

tenant

How well did you know this?
1
Not at all
2
3
4
5
Perfectly
40
Q

Under the doctrine of constructive adverse possession,….

A

…an adverse possessor who enters under “color of title” (i.e., with a facially valid but for some other reason invalid deed) will be deemed in constructive possession of all the property described in the deed so long as the portion of the property constructively possessed is reasonable in proportion to the property actually possessed.

How well did you know this?
1
Not at all
2
3
4
5
Perfectly
41
Q

Notice statute

No conveyance, transfer, or mortgage of real property shall be good against subsequent purchasers for value and without notice, unless the same be recorded.

“No conveyance or mortgage of an
interest in land is valid against any
subsequent purchaser for value
without notice thereof, unless it is
recorded.”

A

To prevail under a notice statute, a person must: take subsequent in time to another person claiming ownership, be a bona fide purchaser for value, and take the property without notice.

(Last BFP)

Subsequent bona fide purchaser
(i.e., for value, without notice)
prevails.

How well did you know this?
1
Not at all
2
3
4
5
Perfectly
42
Q

Race-notice statute

Every conveyance of real property is void as against a subsequent purchaser in good faith and for value whose conveyance is first duly recorded, unless the conveyance shall have been duly recorded prior to the record notice of action.

“No conveyance or mortgage of an
interest in land is valid against any
subsequent purchaser for value
without notice thereof whose
conveyance is first recorded.”

A

Under a race-notice statute, to prevail a claimant must: take subsequent to another person claiming ownership, be a bona fide purchaser for value, take without notice, and record first. Notice can be actual, constructive, or inquiry notice.

(BFP who records 1st)

Subsequent bona fide purchaser
(i.e., for value, without notice) who
records first prevails.

How well did you know this?
1
Not at all
2
3
4
5
Perfectly
43
Q

Race Statutes

“No conveyance or mortgage of an
interest in land is valid against any
subsequent purchaser whose
conveyance is first recorded.”

A

Under a pure race statute, whoever records first wins.

Notice is irrelevant.

Very few
states have such statutes.

Grantee who records first prevails.

*** If neither buyer records, the common law rule of first in time, first in right applies.

How well did you know this?
1
Not at all
2
3
4
5
Perfectly
44
Q

Is an oral contract for the sale of land ever valid?

A

The sale of land normally needs to be in writing to satisfy the Statute of Frauds.

But, there is an exception for part performance. An oral contract for the sale of land is valid with a showing of a combination of any, or all three, of the following: (1) payment of all or part of the purchase price; (2) taking of possession; and (3) making substantial improvements to the property.

How well did you know this?
1
Not at all
2
3
4
5
Perfectly
45
Q

What are valid arguments when seeking to rezone property?

A

The rezoning request is consistent with the long-range land use plans of the local government.

There was an error in the original zoning classification of the buyer’s property.

Changes have occurred in the buyer’s neighborhood which prevent the use of his land as originally zoned.

NOT: Providing evidence to the rezoning board that the seller misinformed the buyer that his intended use would be permitted. Arguing to the board that the land-sale contract was invalid or misconstrued is a remedy best sought under a breach of contract action, not to justify a rezoning of the property.

How well did you know this?
1
Not at all
2
3
4
5
Perfectly
46
Q

If a senior mortgage is modified, a junior mortgage prevails over the modification if …

A

…the modification materially prejudices the holder of the junior mortgage, such as by:

(1) increasing the amount of principal; or
(2) increasing the interest rate (if the rate under the original mortgage was fixed).

Modifications that normally do not materially prejudice the holder of the junior mortgage include:

(1) extension of the mortgage maturity date; and (
2) rescheduling installment payments.
i. e.

Here, the term extension did not materially prejudice Junior Bank’s interest because it lowered the monthly payment. Senior Bank’s mortgage maintained priority despite the modification.

How well did you know this?
1
Not at all
2
3
4
5
Perfectly
47
Q

A developer owned an office building
subject to a first mortgage with a creditor in the
amount of $1 million. Subsequently, the developer
borrowed $100,000 from a bank secured
by a second mortgage on the building to help
pay the first mortgage and other expenses of
the building. The developer’s financial condition
worsened, and he was unable to make the
required payments on the first mortgage to the
creditor. The developer approached the creditor
and offered to give her a deed to the building
in satisfaction of all of his obligations to the
creditor. The developer delivered to the creditor
a quitclaim deed to the building, which recited
as consideration the release of the developer
from all liability on the mortgage to the creditor.
The deed was duly recorded.
Shortly thereafter, the office market greatly
improved, and the building was worth $1.5
million. The developer then brought an action
against the creditor, claiming that the deed was
an equitable mortgage, and the bank served
notice on the creditor that it was preparing to
foreclose its mortgage on the building.
Against which parties will the creditor
prevail?

A

(A) The developer only.

The conveyance from the developer to the creditor discharged the first mortgage on the property because the conveyance was in satisfaction of the mortgage obligation. The first mortgage, however, was never foreclosed and therefore did not wipe out the second mortgage. When the first mortgage was discharged by the conveyance, the second mortgage became a first mortgage and is still a valid
encumbrance on the property.

How well did you know this?
1
Not at all
2
3
4
5
Perfectly
48
Q

The purchaser of a note and trust deed is subrogated to (i.e., takes over) the interest of the seller. What does it mean?

A

subrogated (i.e., takes over)

How well did you know this?
1
Not at all
2
3
4
5
Perfectly
49
Q

The general rule is that when a mortgage is foreclosed,
the buyer at the sale will take title as it existed when the mortgage was placed on the property.

A

Thus, foreclosure generally will destroy all interests junior to the mortgage being foreclosed, but will not discharge senior interests.

However, those with interests subordinate to those of the foreclosing party are necessary parties to the foreclosure action. Failure to include a necessary party results in the preservation of that party’s interest despite foreclosure and sale.

i.e.

The bank’s original mortgage was senior to the financing company’s mortgage. However, where the landowner enters into a modification agreement with the senior mortgagee, raising its interest rate or otherwise making it more burdensome, the junior mortgagee will be given priority over the modification. Thus, the bank’s modification would not have priority over the financing company’s mortgage. Nevertheless, because the financing company failed to include the bank in its foreclosure action, the bank’s mortgage interest survives under its modified terms, even though the modification did not have priority.

How well did you know this?
1
Not at all
2
3
4
5
Perfectly
50
Q

equitable mortgage

A

A landowner needing to raise money may “sell” the land to a person who pays cash and may give the lender an absolute deed rather than a mortgage.

However, if a court concludes that the deed was really given for security purposes, it will treat it as an equitable mortgage and require that the creditor foreclose it by judicial action, like any other mortgage.

The following factors indicate an equitable mortgage:

(i) the existence of a debt or promise of payment by the deed’s grantor;
(ii) the grantee’s promise to return the land if the debt is paid; (iii) the fact that the amount advanced to the grantor/debtor was much lower than the value of the property;

(iv) the degree of the grantor’s financial distress; and
(v) the parties’ prior negotiations.

At a foreclosure sale, the highest bidder
takes the property. Therefore, to take possession of the property, the friend must not only institute
foreclosure proceedings but also be the successful bidder at the sale.

How well did you know this?
1
Not at all
2
3
4
5
Perfectly
51
Q

What is mortgage? What is note?

A

A mortgage is a security interest in property and a note is evidence of the underlying debt.

Physical possession of the mortgage and
note is not required for ownership.

i.e.

Thus, because the brokerage firm bought the mortgage and
note and recorded its interest, the brokerage firm is the owner of both, even though it left possession
of the documents with the finance company.

The investor has no interest in the mortgage and
note because he had record notice of the brokerage firm’s interest (because the brokerage firm recorded the mortgage). Having notice of the brokerage firm’s interest, the investor cannot claim the protection of the recording act. Also, the investor cannot claim holder in due course status,
because that status requires no notice of any other claims to the property. Therefore, the investor
has no interest in the mortgage and note.

How well did you know this?
1
Not at all
2
3
4
5
Perfectly
52
Q

Under the title theory, title is in the mortgagee (lender) until the mortgage has been satisfied or foreclosed.

A

Thus, the mortgagee is entitled to possession on demand at
any time.

How well did you know this?
1
Not at all
2
3
4
5
Perfectly
53
Q

If a junior mortgage is placed on the property and the senior lender later makes an optional advance while having
notice of the junior lien, ….

A

…the advance will lose priority to the junior lien.

(An optional advance
is one that the senior lender is not contractually bound to make.)

How well did you know this?
1
Not at all
2
3
4
5
Perfectly
54
Q

When mortgagor transfer the
mortgaged property, does he
remain personally liable on the mortgage loan?

A

The mortgagor
remains personally liable on the mortgage loan regardless of any subsequent transfers of the mortgaged property.

When a grantee signs an assumption agreement, promising
to pay the mortgage loan, he becomes primarily liable to the lender (who is a third-party beneficiary of the assumption agreement), while the original mortgagor becomes secondarily liable as a surety.

When a mortgagor sells property and conveys a deed, the grantee takes subject to the mortgage, which remains on the land (unless the proceeds of the sale are used to pay off the mortgage). However, a grantee who does not sign an assumption agreement does not become personally liable on the loan. Instead, the original mortgagor remains primarily and
personally liable.

How well did you know this?
1
Not at all
2
3
4
5
Perfectly
55
Q

“due-on-sale” clause

A

A mortgage containing a “due-on-sale” clause allows the
lender to demand full payment of the loan if the mortgagor transfers an interest in the property without the lender’s consent.

Under federal law, due-on-sale clauses are generally enforceable.

How well did you know this?
1
Not at all
2
3
4
5
Perfectly
56
Q

“shelter rule”

A

In general, a person who takes from a bona fide purchaser will prevail against any interest that the transferor-bona fide purchaser would have prevailed against, even if the person taking the property has actual or record notice of the
prior interest.

How well did you know this?
1
Not at all
2
3
4
5
Perfectly
57
Q

Purchase money mortgages

A

Where a seller of property receives a mortgage as part of the purchase price, a purchase money mortgage results. Purchase money mortgages may also arise when a third party lends money to the buyer for the purchase of property and takes a mortgage on the property in return.

In general, the seller’s purchase money mortgage will take priority over the third-party purchase money mortgage.

Purchase money mortgages, however, are subject to later liens by virtue of recording acts.

In the case where a mortgage is modified by agreement between the parties, any increase in the debt resulting from the modification will be subject to a junior lien, even if the original mortgage itself had priority over the junior lien.

In the same way, an optional (as opposed to an obligatory)
advance that is made after the junior lien will have a lower priority than the junior lien. Again, this is the case even if the original mortgage is first in priority.

How well did you know this?
1
Not at all
2
3
4
5
Perfectly
58
Q

A buyer bought a home from a real estate
developer for $700,000. The buyer paid
$100,000 of the purchase price herself. The
buyer’s employer provided $100,000 of the
purchase price by giving the buyer a loan
and taking a mortgage. The developer loaned
$500,000 to the buyer to finance the remainder
of the purchase price, and in return took a
mortgage on the property. One week later, a
bank obtained a judgment against the buyer
for a delinquent credit card balance. The bank
properly recorded its judgment as a lien against
the property. Another month after that, the buyer
incurred some extraordinary medical expenses,
and asked the employer for another $100,000,
which the employer provided and added onto the
principal balance the buyer owed on the loan.
Finally, six months later, the buyer asked the
developer to change the terms of the loan, so
that the buyer would have more time to pay. The
developer and the buyer agreed that the buyer
could have an additional five years to pay the
balance of the loan in exchange for an increase
in the principal of the loan. Shortly thereafter,
the buyer lost his job and defaulted on all of his
payments. The employer brought an action to
foreclose its mortgage. All mortgages and liens
were promptly and properly recorded.
Regarding the distribution of the proceeds of
an eventual sheriff’s sale of the property, which
of the following statements is true?

A

(A) The bank is paid in full before the developer
is paid in full.

The distribution of sale proceeds in this case would be:

(i) the original amount of the employer’s purchase money
mortgage,

(ii) the bank’s judgment,
(iii) the $100,000 advance by the employer, and finally,
(iv) the amount of the increase in the debt to the developer due to the agreed modification of the principal of the original loan.

The original unmodified purchase money mortgage of the developer would remain on the land because it was senior to the mortgage being foreclosed (the employer’s).

How well did you know this?
1
Not at all
2
3
4
5
Perfectly
59
Q

When a mortgagee and an assuming grantee subsequently
modify the original obligation, …

A

…the original mortgagor is completely discharged of liability.

How well did you know this?
1
Not at all
2
3
4
5
Perfectly
60
Q

An owner obtained a loan of $60,000 from a
bank in exchange for a promissory note secured
by a mortgage on his land, which the bank
promptly and properly recorded. A few months
later, the owner obtained another loan of $60,000
from a lender, in exchange for a promissory note
secured by a mortgage on the land, which the
lender promptly and properly recorded. Subsequently,
the owner sold the land to a buyer for
$150,000 and conveyed a warranty deed. The
buyer expressly agreed with the owner to assume
both mortgages, with the consent of the bank and
the lender. A few years later, the bank loaned the
buyer an additional $50,000 in exchange for an
increase in the interest rate and principal amount
of its mortgage on the land. At that time, the
balance on the original loan from the bank was
$50,000. Shortly thereafter, the buyer stopped
making payments on both mortgages and disappeared.
After proper notice to all appropriate
parties, the bank instituted a foreclosure action
on its mortgage, and purchased the property at
the foreclosure sale. At that time the principal
balance on the lender’s mortgage loan was
$50,000. After fees and expenses, the proceeds
from the foreclosure sale totaled $80,000.
Assuming that the jurisdiction permits
deficiency judgments, which of the following
statements is most accurate?

A

(C) The bank keeps $50,000, the lender is
entitled to $30,000, and only the lender can
proceed personally against the owner for its
deficiency.

The bank’s original mortgage has priority in the proceeds, followed by the lender’s mortgage, and only the lender can proceed against the owner because the bank modified its mortgage after the owner had transferred to the buyer.

Generally, the priority of a mortgage is determined by the
time it was placed on the property, and the proceeds of a foreclosure sale will be used to pay off the mortgages in the order of their priority.

However, if the landowner enters into a modification
agreement with the senior mortgagee, raising its interest rate or otherwise making the agreement more burdensome, the junior mortgage will be given priority over the modification.

Thus, if the first mortgage debt is larger because of the modification, the second mortgage gains priority
over the increase in the debt.

Here, the bank and the buyer modified the original mortgage by increasing the principal amount and the interest rate. This modification is not given priority over the lender’s mortgage, and foreclosure proceeds will not be applied against it because the senior lender’s mortgage was not fully satisfied from the proceeds.

With regard to the deficiency, the owner is liable to the lender because when a grantee signs an assumption agreement, becoming primarily liable to the lender, the original mortgagor remains secondarily liable on the promissory note as a surety.

Here, the buyer assumed the lender’s mortgage and became primarily liable; however, the owner remained secondarily liable as surety and can be required to pay off the rest of the lender’s mortgage loan.

On the other hand, the owner will not be liable to pay off
the balance of the bank’s loan, because when a mortgagee and an assuming grantee subsequently modify the original obligation, the original mortgagor is completely discharged of liability.

The owner had nothing to do with the modification agreed to by the bank and the buyer that increased
the amount of the mortgage debt, and will not be even secondarily liable for that amount.

How well did you know this?
1
Not at all
2
3
4
5
Perfectly
61
Q

A grantor may deliver a deed to an escrowee with instructions
that it be delivered to the grantee when certain conditions (e.g., death of the grantor) are met.

A

When the conditions occur, title passes automatically to the grantee and relates back to the date
of delivery to the escrowee.

How well did you know this?
1
Not at all
2
3
4
5
Perfectly
62
Q

Under the estoppel by deed theory,….

A

….if a grantor purports to convey an estate in property that she does not then own, her subsequent acquisition of title to
the property will automatically inure to the benefit of the grantee, but only as against the grantor.

If the grantor transfers her after-acquired title to a bona fide purchaser for value (“BFP”), the BFP gets good title.

Under the majority view, a recorded deed obtained from
a grantor who had no title at that time but who afterwards obtains title is not in the chain of title and so does not give constructive (or record) notice to a subsequent purchaser.

However, a subsequent purchaser is charged with knowledge of whatever an inspection of the property would have disclosed and anything that would have been disclosed by inquiring of the possessor.

How well did you know this?
1
Not at all
2
3
4
5
Perfectly
63
Q

notice statute

A

last BF-value

Under a notice statute, a subsequent bona fide purchaser (i.e., a person who gives valuable consideration and has no actual, record, or inquiry notice of the prior instrument)
prevails over a prior grantee who failed to record.

How well did you know this?
1
Not at all
2
3
4
5
Perfectly
64
Q

A state statute provides as follows:
Any judgment properly filed shall, for
10 years from the date of filing, be a
lien on the real property then owned
or subsequently acquired by any
person against whom the judgment is
rendered.
A landowner conveyed a lot in that state to
his aunt, who had had a judgment lien recorded
against her two years earlier in the county in
which the land was located. One year later, the
aunt conveyed the property to a buyer by general
warranty deed. The deed did not mention the
lien, but the buyer was aware of it. Two years
later, the buyer conveyed the property to a
creditor by special warranty deed. The creditor
was not aware of the lien and her deed also made
no mention of it. One year after that transaction,
the creditor conveyed the property to a developer
by general warranty deed. The developer’s
deed did not mention the lien but the developer
was aware of it. The next year, the developer
entered into a contract to convey the property to
an entrepreneur. The entrepreneur’s title search
disclosed the judgment lien against the aunt,
and the entrepreneur refused to proceed with
the transaction because title was not marketable.
The developer brought an action against the
entrepreneur for specific performance and was
denied relief. He then brought an action against
the aunt, the buyer, and the creditor for breach of
warranty.
Assuming that all transactions concerning the
property were promptly and properly recorded,
and that the party holding the judgment lien has
taken no action as of yet to enforce it, which
parties, if any, will be liable to the developer?

A

The developer will prevail only against the creditor because only the creditor has committed an actionable breach of the covenant against encumbrances.

A grantor making a conveyance by general warranty deed generally makes five covenants for title, and warrants against title defects created both by herself and by all prior titleholders.

The covenant of seisin, the covenant of right to convey, and the covenant against encumbrances are present covenants and are breached, if at all, at the time of conveyance.

The covenant for quiet enjoyment and the covenant of warranty are future covenants and are breached only on interference with the possession of the grantee or
his successors.

Unlike the future covenants, the present covenants do not “run” with the grantee’s estate and cannot be enforced against the covenantor by successive grantees in most jurisdictions.

Here, because the party holding the judgment lien has not taken any action to enforce it, there is no disturbance of possession and the future covenants have not been breached. Since the only covenant that has been breached was the covenant against encumbrances, only the developer’s
grantor, the creditor, is liable.

How well did you know this?
1
Not at all
2
3
4
5
Perfectly
65
Q

In land contracts and deeds, property may be described in various ways; i.e., by reference to a government survey, by metes and bounds, by courses and angles, by references to a recorded plat, by reference to adjacent properties, by the name of the property, or by a street and number system.

When is a description in a deed sufficient?

A

A description in a deed is sufficient if
it furnishes a good lead as to the identity of the property.

How well did you know this?
1
Not at all
2
3
4
5
Perfectly
66
Q

A landowner owned a large tract of mineralrich
land in a sparsely populated area. He
entered into a lease with a prospector who was
interested in developing the land for mining. The
term of the lease was two years and gave the
prospector an option to buy the property at any
time after the first year. The prospector did not
record the lease. Six months later, the prospector
left the land for a period of time to prospect in
Mexico, leaving no goods on the land that would
identify him. The landowner then conveyed the
property in fee simple to a developer, who had
inspected the property while the prospector was
in Mexico and was unaware of the prior transaction.
The developer did not immediately record
her deed. After three months in Mexico, the
prospector returned to the land and encountered
the developer.
A statute in the jurisdiction provides, in part:
“No conveyance or mortgage of an interest in
land, other than a lease for less than one year, is
valid against any subsequent purchaser for value
without notice thereof whose conveyance is first
recorded.”
If the developer brings an action to quiet title,
how should the court rule?

A

The developer’s ownership of the land is subject to the prospector’s interest because the prospector’s interest was first in time.

The common law rule that priority is given to the grantee who was first in time still applies unless operation of the jurisdiction’s recording statute changes the result.

The statute in this question is a race-notice statute, under which a subsequent bona fide purchaser is protected only if she records her interest before the prior grantee does. While the developer is a bona fide purchaser, she must still win the race to the recording office to prevail over the prospector’s prior interest.

Since neither party has recorded in this fact pattern, the developer will take the property subject to the prospector’s prior interest.

How well did you know this?
1
Not at all
2
3
4
5
Perfectly
67
Q

race-notice statute

A

BF-v who 1st records

a subsequent purchaser for value without notice of any prior conveyance is protected if she records before the prior grantee

How well did you know this?
1
Not at all
2
3
4
5
Perfectly
68
Q

A landlord leased an office building to a
tenant for 10 years. The tenant, a secondyear
law student, was familiar with the state’s
recording act, which provided:
No conveyance is valid against any
subsequent purchaser for value without
notice unless the conveyance is recorded.
No lease for three years or more is
valid against a subsequent purchaser
for value without notice unless the
lease has been recorded.
Believing it would be obvious to any prospective
purchaser that the tenant was in possession
of the property, she failed to record the lease.
Shortly thereafter, the landlord entered into
a contract to sell the leased property to a buyer.
Before purchasing the property, the buyer merely
drove by it, and thus did not notice the tenant’s
occupancy. The standard title search did not
reveal the lease because it was unrecorded.
The buyer tendered the purchase money to the
landlord, and the landlord conveyed to the buyer
the property by warranty deed. The buyer subsequently
found the tenant in possession of the
premises and ordered her to vacate. The tenant
refused and asked the buyer where she should
send the rent checks.
In an action by the buyer to evict the tenant,
how should the court rule?

A

The tenant will prevail because the buyer did not properly inspect the property.

A title search is not complete without an examination of possession. If the possession is unexplained by the record,
the purchaser is obligated to make inquiry. The purchaser is charged with knowledge of whatever an inspection of the property would have disclosed and anything that would have been disclosed by the possessor. Thus, the buyer is on constructive notice of the tenant’s possession and anything
that would have been disclosed by inquiring of the tenant. Because the buyer had this notice, he is not protected by the notice recording statute, and he will take subject to the tenant’s lease.

How well did you know this?
1
Not at all
2
3
4
5
Perfectly
69
Q

When a purchaser has paid only part of the purchase price under an installment land contract, most courts hold that the purchaser is protected by the recording acts only to the extent
of payment made.

Depending on the equities involved in the case, the court has three options:

A

(i) create a tenancy in common in the property, with the contract purchaser receiving a share of the property equal to the proportion of payments made;

(ii) award the land to the prior claimant,
but give the contract purchaser a right to recover the amount she paid (with interest), secured by a lien on the property; or

(iii) award the land to the contract purchaser, but require the contract purchaser to make the remaining payments to the prior claimant. This last obligation is also
secured by a lien on the property

How well did you know this?
1
Not at all
2
3
4
5
Perfectly
70
Q

If property
is specifically devised or bequeathed in the testator’s will, but the testator no longer owns that
property at the time of death, the gift

A

is adeemed; i.e., it fails.

i.e.

Here, the mother specifically
devised the land to the daughter in her will. However, the mother conveyed the same land to the
investor prior to her death. Thus, the land was no longer in the mother’s estate at her death, so the
daughter takes no interest in the land under the will.

How well did you know this?
1
Not at all
2
3
4
5
Perfectly
71
Q

The owner of a gourmet food store entered
into oral negotiations with the president of a
food products corporation to secure an exclusive
distributorship in the state for a popular truffle
sauce. After some discussion, the parties agreed
on all salient points and shook hands on the
deal. They agreed further that the corporation’s
general counsel would reduce the agreement to
writing and that the agreement would become
effective after it was drawn up and initialed
by the corporation’s general counsel and by
the store owner’s attorney. The corporation’s
general counsel duly committed the agreement
to writing and sent the writing to the store
owner’s attorney, but without initialing it first.
The attorney looked over the agreement, made
no changes, initialed it, and mailed the agreement to the corporation’s counsel on May 1. On May 2, the president of the corporation decided that a large national grocery chain would be a better distributor for the sauce than the gourmet food store, so he had the general counsel call the store’s attorney to say that the deal was off. The corporation’s counsel received the written agreement with the attorney’s initials on it on May 3. Nevertheless, the corporation began to distribute its products through the national grocery chain. If the gourmet food store owner files suit against the corporation and its president for breach, is she likely to prevail?

(A) Yes, because the mailing of the written
agreement to the attorney constituted an
irrevocable offer.
(B) Yes, because the owner and the corporation’s
president entered into a valid oral
contract.
(C) No, because the corporation’s general
counsel never initialed the written agreement
and there was, therefore, failure of an
express condition.

A

(B) The gourmet food store owner is likely to prevail, because she and the corporation’s president formed a valid oral contract when they agreed on all salient points after negotiations. One element of their agreement was that the terms would be put in writing, a process sometimes called “memorializing” the agreement. The writing does not constitute the agreement itself, but is merely a written record of it. Another element of their oral agreement concerned when it would take effect: on completion of the “memorialization.” The general counsel’s failure to initial the writing, whether deliberate or inadvertent, was a breach of the oral agreement that he would do so, but cannot be used to avoid performance of the contract. Note that even though the contract involves
the sale of goods and may be for $500 or more, a violation of the Statute of Frauds does not affect the contract’s validity, just its enforceability. Here, the memorialization of the contract makes it enforceable under the Statute (even without his initials).

(A) is incorrect because the oral agreement
was already in effect at the time the writing was mailed. The processes of offer and acceptance
took place during the oral negotiations. Even if the mailing of the writing could somehow
be seen as an offer, there are no facts, such as detrimental reliance, that show it to be irrevocable.
(C) is incorrect because, as described above, the writing was a memorial of the existing oral agreement. Even if the requirement of the general counsel’s initialing is characterized as an
express condition, the corporation’s president would not be permitted to prevent the occurrence of the condition and then claim the benefit of its nonoccurrence.

How well did you know this?
1
Not at all
2
3
4
5
Perfectly
72
Q

The order department of a machine tools manufacturing company received a phone call
from a factory owner who placed an order for two of the company’s standard “Type-A” machines. The factory owner and the company came to an oral agreement whereby the total price for both machines was agreed to be $10,000. The first machine was to be delivered on May 1, with payment of $5,000 due 30 days after delivery, and the second machine was to be delivered on June 1 on the same terms (payment of $5,000 due 30 days after delivery). Although the company did not carry the machine in stock,
no retooling was required because the Type-A
machine was a standard model. The first machine was duly delivered on May 1. The second machine arrived on June 1, but the factory owner refused to accept delivery and also refused to pay for the first machine. The company sued the factory owner on June 2. Assume that it cost the company $3,000 to manufacture each Type-A machine, and that the company could resell the machine for only $3,000. What damages should be awarded aside from any incidental damages?
(A) $3,000.
(B) $5,000.
(C) $7,000.
(D) $10,000.

A

(B) The company should recover $5,000 because the oral contract between it and the factory owner is enforceable to the extent the factory owner received and accepted the goods.

A promise for the sale of goods of $500 or more is not enforceable unless evidenced by a writing signed by the party to be charged. [UCC §2-201(1)] However, an oral contract for such goods is enforceable to the extent of goods received and accepted by the buyer. [UCC §2-201(3)(c)] Oral contracts for specially manufactured goods not suitable for sale in the ordinary course of the seller’s business also are enforceable when the seller has begun substantially to perform.

Here, the parties’ agreement was oral. The factory owner accepted one machine, but neither machine was specially manufactured. He is bound to pay the $5,000 contract price for the accepted machine but is not bound to pay for the rejected machine.

(A) is incorrect because the company is entitled to the contract price for the machine the factory owner accepted, not just restitution. If a contract is unenforceable because of noncompliance with the Statute of Frauds, a party can generally sue
for the restitution of any benefit that has been conferred.
However, as discussed above, an oral
contract for the sale of goods of $500 or more is enforceable to the extent of goods received and
accepted by the buyer. Therefore, because the factory owner accepted one machine, the company is entitled to the $5,000 contract price of that machine, not just $3,000 in restitutionary damages.
(C) is incorrect because the company is not entitled to any damages as to the rejected machine. As indicated above, the company is entitled to $5,000 for the first machine. There is no enforceable contract regarding the second machine, and the company is not entitled to damages for that machine. If there were an enforceable contract for the second machine, (C) would state a proper measure of damages—if a buyer breaches by refusing to accept goods, the seller is entitled to recover the difference between the contract price ($5,000) and the market or resale price ($3,000), here, $2,000. Thus, the company would be entitled to $5,000 for the accepted machine and $2,000 for the rejected machine, or $7,000.

(D) is incorrect for the same reason that (C) is incorrect— the company is only entitled to the contract price of the machine accepted by the factory owner. If the contract for the second machine were enforceable, (D) would still not be a proper measure of damages. Under the UCC, the seller has a right to force goods on a buyer who has not accepted them only if the seller is unable to resell the goods or if the goods have been lost or damaged after the risk of loss passed to the buyer. [UCC §2-709] Because the company can resell the second machine, which has not been lost or damaged, it could not recover the full price of the second machine from the factory owner even if the contract was fully enforceable.

How well did you know this?
1
Not at all
2
3
4
5
Perfectly
73
Q

A leading computer supply retailer contacted
a manufacturer of flash drives on October 25
to supplement the supply of flash drives at the
retailer’s area stores. In response, the manufacturer offered to supply 50 boxes of flash drives at $200 a box, for a total price of $10,000,
delivery one week after acceptance, and sent
a letter containing those contractual terms to
the retailer on October 26. The president of the
retailer signed the letter on October 27 without
making any changes to it and sent it back to the
manufacturer by first class mail. The next day,
having not heard from the retailer, the manufacturer’s sales manager contacted the president of the retailer and informed him that, if the retailer accepted by the end of the month, it would receive a 1% discount on the total price. When the retailer’s president responded that he had already sent the acceptance, the sales manager assured him that the discount would still apply. On October 31, the retailer received a circular from a competing source offering comparable flash drives for 5% less than the manufacturer’s price. The retailer immediately faxed a rejection to the manufacturer. The manufacturer received the rejection immediately but took no action on it. The next day, November 1, the manufacturer received the signed contract from the retailer. What is the status of the parties’ agreement?

(A) An enforceable contract was formed for
$9,900 because the retailer accepted before
October 31 and needed no additional consideration
for the oral modification.

(C) An enforceable contract was formed for
$10,000 because, even though the retailer
accepted before October 31, the Statute
of Frauds makes the oral modification
between the parties unenforceable.

A

(C) A contract was formed for $10,000 because the oral modification is unenforceable under the
Statute of Frauds.

Under the UCC, a contract for the sale of goods priced at $500 or more is not enforceable unless evidenced by a writing. Furthermore, contract modifications must also meet the Statute of Frauds requirement if the contract as modified is within the Statute’s provisions.

Here, the Statute of Frauds is applicable to both the original contract and the contract as modified.
Since the modification was not in writing, it is not enforceable; hence, the terms of the original
contract, which satisfies Statute of Frauds requirements, are effective.

How well did you know this?
1
Not at all
2
3
4
5
Perfectly
74
Q

A gambler lived in a state where gambling was illegal. Nevertheless, he gambled on a regular basis. He asked his friend to lend him $5,000 to bet on a football game. His friend agreed to lend him $5,000 if the gambler would bet half of it on the friend’s behalf. The gambler agreed, took his friend’s $5,000, and placed the bet. The gambler won on 4-to-1 odds. He gave his friend his $5,000 back but refused to tender any winnings.
If his friend sues the gambler to recover the winnings due under the contract, who will
prevail?

(A) His friend, because he fully performed his
part of the bargain.
(B) His friend, because the court will not allow
the gambler to unfairly profit from his
illegal contract.
(C) The gambler, because the contract was
illegal and the court will not enforce an
illegal contract.
(D) The gambler, because the contract was
illegal and the court will only act to put the
parties in the status quo ante, and his friend
already has his money back.

A

(C) The gambler will prevail.

The general rule is that a court will not enforce a contract if its subject matter or consideration is illegal; the court will leave the parties as it finds them.

Here, the subject matter of the contract, placing gambling bets, is illegal in the state. Thus, (A) is wrong because his friend’s performance is irrelevant. (B) is wrong because the court will refuse to help either party to an illegal contract, even where one party has gained unfairly. (D) is wrong because the court will not put the parties back into the position they were in prior to entering into the contract, but rather will leave them where they stand.

How well did you know this?
1
Not at all
2
3
4
5
Perfectly
75
Q

Mutual mistake of an existing material fact to a contract is a defense that allows the adversely affected party to …

A

…rescind as long as that party did not assume the risk of mistake.

***

Mutual mistake arises where both parties are mistaken as to an existing fact relating to a basic
assumption of the contract and the mistake has a material effect on the agreed-upon exchange.

How well did you know this?
1
Not at all
2
3
4
5
Perfectly
76
Q

Where only one of the parties entering into a
contract is mistaken about facts relating to the agreement, this unilateral mistake will not prevent
formation of a contract unless ….

A

…the nonmistaken party is or had reason to be aware of the mistake made by the other party.

How well did you know this?
1
Not at all
2
3
4
5
Perfectly
77
Q

On reaching majority, an infant may affirm; that is, choose to be bound by her contract.

A

When a voidable promise is reaffirmed, the promise will be enforced according to the terms of the reaffirmation rather than the original obligation.

How well did you know this?
1
Not at all
2
3
4
5
Perfectly
78
Q

A woman contracted with a dog breeder for the purchase of a show dog. Prior to the sale, the
breeder falsified the dog’s pedigree records to
indicate that there were several champions in the
dog’s bloodline. After a few weeks of enjoying
the dog’s companionship, the woman discovered
the breeder’s actions when she attempted to register the dog with a local dog club. The woman wishes to keep the dog, but is understandably unhappy with the high price that she paid. What is the woman’s best course of action for relief?

(A) The woman should notify the seller and sue
for warranty damages.
(B) The woman should seek to rescind the
contract based on fraudulent misrepresentation.
(C) The woman should seek reformation of the
contract to reflect a fair price for the dog.
(D) The woman should take no action because
she wishes to keep the dog.

A

(A) The woman should notify the seller and sue for damages because the dog the woman received was not of the quality she was promised, in violation of an express warranty made by the seller.
Any affirmation of fact made by a seller to a buyer creates an express warranty if the statement
is part of the basis of the bargain.

Here the woman was looking for a show dog, and the breeder represented to her that this dog was part of a champion bloodline. The dog’s pedigree clearly was a part of the basis of the bargain because the buyer paid a higher price for the dog than she would have if the dog was not from a champion bloodline. The breeder’s actions created an express warranty. The dog the woman received, however, was not as the breeder represented, thus the breeder breached this warranty. Nevertheless, the woman has decided to keep the dog. If a buyer accepts goods that breach one of the seller’s warranties, the buyer may recover damages. The basic measure of damages in such a case is the difference between the value of the goods as delivered
and the value they would have had if they had been according to contract, plus incidental and
consequential damages.
To recover damages for any defect as to accepted goods, the buyer must,
within a reasonable time after she discovers or should have discovered the defect, notify the seller of the defect. This would be the best course of action among the options given for the woman because she would keep the dog and get back some of the money she overpaid for the dog.

(B) is not the correct answer. While it is true that the woman could rescind the contract based on the seller’s fraudulent misrepresentation, it is not the best option in this case. Rescission is a remedy whereby the original contract is considered voidable and rescinded. The parties are left as though a contract had never been made. The woman here does not want to avoid the contract entirely since she wants to keep the dog.

(C) is incorrect. Misrepresentation as to the subject matter of an agreement is not grounds for reformation because a court will not remake the parties’ bargain. Rescission and/or damages are the proper remedies for misrepresentation.

(D) is incorrect because it is not the best option for the woman to get relief. As discussed above, a buyer can decide to keep nonconforming goods and still recover damages.

How well did you know this?
1
Not at all
2
3
4
5
Perfectly
79
Q

A gallery owner offered to sell a piano to an acquaintance for $400. The acquaintance had been to the gallery owner’s house a few
weeks prior to the offer and had seen a Steinberg piano in his living room, so she accepted. Unbeknownst to the acquaintance, the gallery owner also owned a Hairwin piano, which he kept at the gallery, and that was the piano that
he intended to sell. The acquaintance had never
been to the gallery, and the gallery owner was
aware that the acquaintance had never seen the
Hairwin. If the acquaintance sues the gallery owner to obtain the Steinberg, which party is likely to prevail?

(A) The acquaintance, because the gallery
owner knew of the ambiguity.
(B) The acquaintance, because that is the result
under the objective test.
(C) The gallery owner, because there was a
mutual mistake.
(D) The gallery owner, because he subjectively
intended to sell the Hairwin instead of the
Steinberg.

A

(A) The acquaintance will likely prevail because the gallery owner knew of the ambiguity.

Contract language with more than one possible meaning leads to different results depending on the awareness of the parties. If one party was aware of the ambiguity and the other party was not at the time of contracting, a contract will be enforced according to the intention of the party who was unaware of the ambiguity.

Here, the gallery owner knew he had two pianos, and the acquaintance did not. Thus, there is a contract on the terms as understood by the acquaintance; there is a contract for the Steinberg.

How well did you know this?
1
Not at all
2
3
4
5
Perfectly
80
Q

The doctrine of waste governs the obligations between a life tenant and the holder of the remainder regarding the payment of a mortgage on the property. Under this doctrine, …

A

… a life tenant is obligated to pay interest on any encumbrances on the land, but he does not have to pay anything on the principal of the debt; reversioners or remaindermen must pay the principal in order to protect their interests.

How well did you know this?
1
Not at all
2
3
4
5
Perfectly
81
Q

There are three types of restraints on alienation:

A

(i) disabling restraints, under which any attempted transfer is ineffective;
(ii) forfeiture restraints, under which an attempted transfer results in a forfeiture of the interest; and
(iii) promissory restraints, under which an attempted transfer breaches a covenant.

Any total restraint on a fee simple—either forfeiture, disabling, or promissory—is void.

How well did you know this?
1
Not at all
2
3
4
5
Perfectly
82
Q

A rancher deeded his ranch “to my son so
long as he lives, and on his death to my son’s
widow for the remainder of her life, and on her
death to each of her lineal descendants who are
living on her death, to share and share alike.”
The deed was validly executed, delivered, and
recorded. At the time of the conveyance, the
son was married but had no children. Three
years later, the rancher died. His will left all
of his estate to his sister. Two years after the
rancher’s death, the son was killed in an automobile accident, leaving his widow with twin
girls. Upon the widow’s death many years later,
she was survived by both girls, her only lineal
descendants. Who now owns the ranch?

(A) The girls are tenants in common, with each
owning an undivided one-half; their interest
vested on the son’s widow’s death, well
within the period of lives in being plus 21
years.
(B) The girls are tenants in common, with
each owning an undivided one-half; they
received a vested remainder, which is not
subject to the Rule Against Perpetuities.
(C) The sister owns the ranch in fee simple,
because the Rule Against Perpetuities
voided the gift over to the son’s widow and
the following gift to her lineal descendants.
(D) The sister owns the ranch in fee simple,
which she received upon the death of the
son’s widow.

A

(D) The sister owns the ranch, because the gift over to the son’s widow’s descendants was void.

This is the case of the unborn widow. Remember, the Rule Against Perpetuities is measured and enforced at the time of the grant, not later. If there is any way, at the time of the grant, that a gift over may vest later than any life in being plus 21 years, it is void.

Take the grants one at a time, in order.

The gift to the son is valid; he was alive at the time of the grant.

The gift to the son’s widow also is valid. Although her life estate is a contingent remainder (because we would not know the identity of the son’s widow until the son dies), we would know her identity (and thus the remainder will become vested) at the moment of the son’s death, clearly within the time period of the Rule.

But the gift to the widow’s descendants violates the Rule Against Perpetuities. The son’s widow was not necessarily born and in existence at the time of the conveyance, and we will not know who the widow’s descendants are until she dies, which may be well outside the lives in being at the time of the rancher’s transfer, plus 21 years. Since the gift to the widow’s descendants
is void, the grant effectively reads: “to my son for life, then to my son’s widow for life.”
That would have left a reversion in the rancher, which the sister inherited under the rancher’s
will. The fact that the son’s widow was in fact alive at the time of the rancher’s grant does not
matter because the Rule Against Perpetuities is applied at the time of the grant. Subsequent facts
are irrelevant.

How well did you know this?
1
Not at all
2
3
4
5
Perfectly
83
Q

A co-tenant has no duty to improve the
property and cannot force the other co-tenants to contribute to the cost of improvements made
by him. Can the value of improvements ever be recouped?

A

Only in an action for partition can the value of improvements be recouped.

How well did you know this?
1
Not at all
2
3
4
5
Perfectly
84
Q

By mutual agreement, a brother and sister
purchased a 10-acre parcel of land and took
title “as joint tenants with right of survivorship.”
Three years after the purchase, the brother asked if he could build an apartment house on his half of the property; the sister agreed. He then built an apartment house on the eastern five acres of the property. Two years later, the brother died, leaving his entire estate to his son. In an action for partition, what is the best argument in favor of the brother’s estate being judged the owner of the eastern five acres?

(A) The sister’s conduct during the brother’s
lifetime estopped her from asserting title to
the eastern half of the property.
(B) The taking of title as joint tenants does not
conclusively presume that the property is
held as joint tenants.
(C) The joint tenancy was terminated at the
time the oral agreement was made.
(D) A joint tenant may will away his interest in
property to a lineal descendant

A

(A) Because the sister informed her brother that he could develop the eastern five acres of the land, and because he reasonably relied on her statement to his detriment, she may be estopped to deny her brother’s likely intended effect, i.e., to develop the property and pass on the benefit of
the improved portion to his son.

(B) is incorrect. Where the language of the grant is clear and a joint tenancy is created, no presumption is needed or applies. The joint tenancy cannot later be changed by subsequent informal action.

(C) is wrong because an oral agreement is not effective to terminate a joint tenancy.

(D) is an incorrect statement of the law. A joint tenant may not pass an interest to anyone at death, due to the right of survivorship.

How well did you know this?
1
Not at all
2
3
4
5
Perfectly
85
Q

A landowner devised her parcel of land to her
daughter, her heirs, and assigns, “so long as the
property is used for residential purposes, then to
my niece, her heirs, and assigns.” The remainder
of the landowner’s property passed through the
residuary clause of her will to her grandson. The
daughter lived on the land for 25 years; then, on
her death, ownership passed to her husband. In
the meantime, the niece had also died, leaving
her entire estate to her son. The husband has
leased the land to a developer, who has obtained
the necessary permits to build a shopping center
on it. The grandson and the niece’s son both file quiet title and ejectment actions against the
husband, and the cases are consolidated. How should the court, applying common law, rule as to ownership of the land?

(A) For the husband.
(B) For the niece’s son.
(C) For the grandson, because he received a
right of reversion from his grandmother.
(D) For the grandson, because he received
a possibility of reverter from his grandmother.

A

Remember! Executory interest following defeasable fee violates RAP.

(D) The court should rule for the grandson because he received a possibility of reverter from his grandmother. The landowner attempted to give the daughter and her successors a fee simple subject to an executory interest, with the niece and her successors holding the executory interest. However, the attempted gift to the niece and her successors fails under the Rule Against Perpetuities because the niece’s interest could vest in possession more than 21 years after a life in being.
Thus, (B) is incorrect. After the void interest is stricken, the daughter and her successors have a
fee simple determinable and the landowner retained a possibility of reverter, which passed to the grandson through the residuary clause in the landowner’s will. When the husband, the daughter’s successor, ceased using the property for residential purposes, the possibility of reverter matured, leaving ownership in the grandson. (A) is therefore incorrect. (C) is incorrect because a possibility of reverter, not a right of reversion, is the interest left in the grantor when a fee simple determinable is created.

How well did you know this?
1
Not at all
2
3
4
5
Perfectly
86
Q

A college student assigned her apartment
rental lease, by written agreement, to one of
her sorority sisters. The sorority sister, in turn,
assigned the lease to a classmate. If the classmate fails to pay the rent, can the landlord bring suit against the sorority sister to recover this money?
(A) Yes, because the sorority sister remains in
privity of estate with the landlord.
(B) Yes, because the sorority sister remains in
privity of contract with the landlord.
(C) No, because the sorority sister is no longer
in privity of estate with the landlord.
(D) No, because the fact that the college student
was allowed to assign the lease means that
the sorority sister is allowed to assign it
also.

A

L — T (priv of K) — A — A (priv of estate)

(C) The landlord cannot bring suit against the sorority sister. Absent an express assumption, an
assignee is not liable on the original covenants once she reassigns. Hence, (A) is wrong.

(B) is wrong because an assignee is not in privity of contract with the lessor unless the assignee
expressly assumes the lease obligations.

(D) is incorrect because the sorority sister’s ability
to subsequently assign the lease is unrelated to the issue of liability. The sorority sister, as an
assignee of the college student, is liable for covenants running with the land on the basis of privity of estate. Once the sorority sister assigned the lease to her classmate, the privity of estate ends and the sorority sister’s liability is terminated.

***

An assignee is in privity of estate with the lessor, and is liable for those covenants in the original lease that run with the land.

How well did you know this?
1
Not at all
2
3
4
5
Perfectly
87
Q

An attorney was staying at a hotel while traveling to work on a case. The normal room charges were $150 per night. The attorney made a deal with the hotel management to pay $700 per week for his room for an indefinite period. The attorney never told the hotel management exactly how long he planned to stay or how he
would pay, but each Sunday he would present the cashier with a check for $700, plus any additional taxes and room charges, to pay for the previous week’s lodging. He did this for four weeks, and then the case he was working on was
settled. On a Thursday morning, before checkout
time for that day, the attorney tendered the
hotel cashier a check for $400 plus additional
charges incurred from Sunday through Wednesday nights. The cashier promptly took
the check and then demanded that the attorney
pay an additional $1,000 plus taxes for 10 more
days’ lodging, covering the rest of the week
plus an additional week because he did not give
a week’s notice that he planned to vacate. The
attorney refused to pay the additional charges. If
the hotel sues the attorney and the court determines that the parties created a tenancy, for
how many additional days’ lodging will he be
required to pay?

(A) None, because a tenancy at will was created.
(B) Three days, because a periodic tenancy was
created.
(C) Seven days, because a periodic tenancy was
created.
(D) Ten days, because a periodic tenancy was
created.

A

(D) The attorney must pay 10 days’ lodging because his notice to terminate will not become effective until then.

In the usual case, a hotel guest is treated as a licensee rather than a tenant. Here, however, the court found that the parties created a tenancy because they specifically agreed to a week-to-week arrangement at a special weekly rate.

A periodic tenancy arises when the parties
do not fix the duration of the tenancy. It continues from period to period and is automatically renewed for another period until terminated by the giving of proper notice.
The period is generally based on an express understanding between the parties or an implied understanding based on the payment of rent.

The attorney’s arrangement with the hotel constitutes a periodic weekto- week tenancy because it was of indefinite duration and “rent” was paid each week.

To terminate a periodic tenancy of less than one year, a full period in advance of the period in question is required by way of notice. For a week-to-week tenancy, notice at least one full week prior to vacating would have to be given prior to the beginning of the period. Thus, the attorney’s notice had no effect on his liability for that week’s rent and would operate as one week’s notice starting at the beginning of the next week.

(A) is incorrect. If the parties had expressly agreed that either party could terminate at any time, a tenancy at will would have been created and the attorney would be liable for nothing. However, the payment of rent on a regular periodic basis will cause a court to treat the tenancy as a periodic tenancy.

(B) is incorrect because it does not take the one
week’s notice requirement into account.

(C) is incorrect because the notice must fix the last day of the period as the date of termination rather than some intervening day. Thus, the notice period does not begin to run until the end of the current week.

How well did you know this?
1
Not at all
2
3
4
5
Perfectly
88
Q

A landlord’s promise in a lease to maintain the property does not terminate because the property is sold.
Although no longer in privity of estate, the original landlord and tenant remain in privity of contract, and the original landlord remains liable on the covenant unless there is …

A

…a novation.

A novation substitutes a new party for an original party to the contract. It requires the assent of
all parties, and completely releases the original party.

How well did you know this?
1
Not at all
2
3
4
5
Perfectly
89
Q

A landlord leased an apartment to a tenant
for five years. The lease provided that the
landlord will: (i) keep the apartment building at
a comfortable temperature 24 hours per day, and
(ii) have the carpets cleaned once a year. Two
years later, the landlord began turning off the air
conditioning at 10 p.m. The tenant’s apartment
became hot and stuffy, and she demanded that
the landlord honor the covenant. The landlord
refused. The following month, the pipes burst
in the tenant’s only bathroom, rendering it
unusable. The resultant flooding soiled some
of the carpeting, which had not been cleaned
in the past 12 months. The tenant reported the
problems to the landlord, who did not return the
tenant’s phone calls.
Which of the following are valid reasons for
the tenant to terminate the lease?
(A) Only that the landlord did not keep the
apartment building at a comfortable temperature
24 hours per day.
(B) Only that the landlord did not fix the
bathroom pipes.
(C) That the landlord did not keep the apartment
building at a comfortable temperature
24 hours per day and did not fix the
bathroom pipes.
(D) That the landlord did not keep the apartment
building at a comfortable temperature
24 hours per day, did not have the carpets
cleaned, and did not fix the bathroom pipes.

A

(B) The tenant will be successful in terminating the lease because the landlord breached the implied warranty of habitability by failing to fix the bathroom pipes.

The general rule at common law was that the landlord was not liable to the tenant for damages caused by the landlord’s failure to maintain the premises during the period of the leasehold. Today, however, a majority of jurisdictions,
usually by statute, provide for an implied warranty of habitability for residential tenancies.
_In the absence of a local housing code, the standard applied is whether the conditions are
reasonably suitable for human residence._If the landlord breaches the implied warranty, the tenant may: (i) terminate the lease, (ii) make repairs and offset their cost against future rent, (iii) abate rent, or (iv) seek damages.

Here, a court is likely to consider the lack of a functioning bathroom as making the premises unsuitable for human residence, allowing the tenant to terminate the lease. (A) is therefore incorrect.

(C) would be a stronger answer if the tenant had vacated the premises within a reasonable time. The doctrine of constructive eviction provides that where a landlord does an act or fails to perform some service that he has a legal duty to provide, and thereby makes the property uninhabitable, the tenant may terminate the lease and seek damages. However, a tenant cannot claim a constructive eviction unless: (i) the injurious acts were caused by the landlord, (ii) the premises are uninhabitable, and (iii) the tenant vacates the premises within a reasonable time. Here, the landlord’s failing to keep the apartment building at a comfortable temperature 24 hours per day meets conditions (i) and perhaps (ii), but the tenant remains in possession. Therefore, the tenant cannot claim constructive eviction and (C) is incorrect. (D) is incorrect for the same reason.

How well did you know this?
1
Not at all
2
3
4
5
Perfectly
90
Q

A developer created an exclusive residential
subdivision. In his deed to each lot, the following
language appeared:

Grantee agrees for himself and assigns
to use this property solely as a singlefamily residence, to pay monthly fees as levied by the homeowners’ association for upkeep and security guard services, and that the backyard of this property shall remain unfenced so that bicycle paths and walkways may run through each backyard, as per the subdivision master plan [adequately described], for use by all residents of the subdivision.
The developer sold lots to an actuary, a baker,
and a coroner. All deeds were recorded. The
subdivision was developed without backyard
fences, with bicycle paths and walkways in
place in accordance with the general plan. The
actuary in turn sold to an accountant by a deed
that omitted any mention of the covenants above,
and the accountant had no actual knowledge
thereof. Shortly thereafter, the accountant started
operating a tax preparation business out of his
home. The baker in turn sold to a barber, who
knew of, but refused to pay, the monthly fees
levied by the homeowners’ association. The
coroner leased her property for 10 years to a
chiropractor, who erected a fence around the
backyard, unaware of the covenant against such
fencing.
According to common law principles, which
of the following statements is correct?
(A) If the developer, still owning unsold lots,
sues the accountant to have him cease
operating the tax preparation business, the
accountant would win because there is no
privity between the developer and the accountant.
(B) If the homeowners’ association sues the
barber to collect the monthly fees for
upkeep and security guard services, the
homeowners’ association would win
because the covenant regarding fees is
enforceable in equity against the barber.
(C) If the barber sues the chiropractor to obtain
removal of her backyard fence, the barber
would win because the covenant regarding
fencing is enforceable in equity against the
chiropractor.
(D) If the chiropractor sues the accountant to
have him cease operating the tax preparation
business, the chiropractor would win
because the covenant regarding singlefamily
use is enforceable at law against the
accountant.

A

An equitable servitude is a covenant that, regardless of whether it runs with the land at law, equity will enforce against the assignees of the burdened land who have notice of the covenant.

The benefit of an equitable servitude runs to successors if: (i) the original parties so intended, and (ii) the servitude touches and concerns the land.

The burden runs if (i) and (ii) are met and (iii) the
subsequent purchaser has actual or constructive notice of the covenant.

Privity of estate is not needed to enforce an equitable servitude because it is enforced not as an in personam right against the owner of the servient tenement, but as an equitable property interest in the land itself.

Here, the original parties intended for the fencing covenant to be enforceable by and against assignees, as shown by the specific language of the covenant (“Grantee agrees for himself and assigns”) and its purpose to provide bicycle paths and walkways running through each backyard for the use of all subdivision residents.

The benefit of the covenant touches and concerns the barber’s property because it increases his enjoyment thereof by providing him with such paths and walkways. Therefore, the barber is entitled to enforce the covenant.

The burden of the covenant touches and
concerns the land occupied by the chiropractor because it restricts the landholder in her use of
the parcel
(i.e., her rights in connection with the enjoyment of the land are diminished by being
unable to fence in the backyard).

The chiropractor will be deemed to have inquiry notice of the restriction because the subdivision is sufficiently developed in accordance with a general plan for the subdivision.

Moreover, any neighbor in a subdivision can enforce a covenant contained in a subdivision deed if a general plan existed at the time he purchased his lot. As has been noted, the maintenance of access to all backyards for use as bike paths and walkways was part of such a general plan.

Finally, the fact that the chiropractor did not succeed to the coroner’s entire estate,
but rather a leasehold interest, is irrelevant because privity is not required to enforce an equitable
servitude. Therefore, all of the requirements are in place for the existence of an equitable servitude,
which can be enforced by the barber against the chiropractor. (A) is incorrect because
there is privity between the developer and the accountant. There was horizontal privity between
the original covenanting parties because, at the time the actuary entered into the covenant with
the developer, they shared an interest in the land independent of the covenant (i.e., they were in
a grantor-grantee relationship). The accountant holds the entire interest held by the actuary at
the time the actuary made the covenant; thus, there is vertical privity. (B) is incorrect because
the remedy sought is the payment of money. Breach of a real covenant, which runs with the
land at law, is remedied by an award of money damages, whereas breach of an equitable servitude
is remedied by equitable relief, such as an injunction or specific performance. Because the
homeowners’ association seeks to obtain from the barber the payment of money, it is inaccurate to
refer to this as a situation involving an equitable servitude. (D) is incorrect because, as explained
above, if equitable relief is sought, the covenant must be enforced as an equitable servitude rather
than a real covenant.

How well did you know this?
1
Not at all
2
3
4
5
Perfectly
91
Q

The doctrine of reciprocal negative servitudes.

A

When a developer subdivides land into several parcels and some of the deeds contain
negative covenants, but some do not, negative covenants or equitable servitudes, binding all of the parcels in the subdivision, may be implied under the doctrine of reciprocal negative servitudes.
To enforce a reciprocal negative servitude, the court will need to find:

_(i) a common scheme for
development, and
_

(ii) notice of the covenants.

A common scheme may be evidenced by a general pattern of prior restrictions.

Notice may be _a_ctual (i.e., direct knowledge of covenants in prior deeds), _i_nquiry (i.e., neighborhood appears to conform to common restrictions), or _r_ecord (i.e., prior deeds containing covenants are in grantee’s chain of title).

When the owner had a scheme for an exclusively residential subdivision that included lots when the sales began (as evidenced by the recorded plat), a court will imply a reciprocal negative servitude limiting the remaining lots to the same use.

i.e.

Owner had 250 lots. Sold 175 with negative covenants. Sold remaining 75 to a different buyer at a later time.

How well did you know this?
1
Not at all
2
3
4
5
Perfectly
92
Q

A developer divided his tract of land into four
standard lots, which he conveyed to a doctor, a
pilot, a carpenter, and an athlete,
respectively.
Each deed granted by the developer contained
a covenant requiring that the property be used
only for single-family housing. All deeds were
duly recorded in the office of the county recorder
of deeds. The doctor and the pilot proceeded
to build single-family houses on their lots. The
carpenter and the athlete did not develop their
properties immediately.

The doctor later sold her property to a nurse
and included the covenant limiting use to singlefamily dwellings in the deed.

The pilot sold his property to a flight attendant, but did not include the covenant in the deed.

The carpenter sold her property to an electrician, and the deed contained the restriction.

The athlete sold his property to a physical therapist, and the deed did not contain the restriction.

All but the electrician’s deed were duly recorded.

Subsequently, the nurse died and her property
passed by will to her daughter.

The flight attendant gave her land to her son “for life.”
The electrician sold his property for value to
a plumber.

All three transfers of title were recorded, but none of the deeds mentioned the covenant.

Which of the current owners below is not bound by the covenant?

(A) The nurse’s daughter, who received the
property by will.
(B) The flight attendant’s son, who received the
property for life.
(C) The plumber, who purchased the property
from the electrician.
(D) The physical therapist, who purchased the
property from the athlete

A

(B) The covenant may not be enforced at law against the son.

If all requirements are met for the burden of a covenant to run, the successors in interest to the burdened estates will be bound by the arrangement entered into by their predecessors as effectively as if they had expressly agreed
to be bound.

The requirements are:

(i) The covenanting parties must have intended that successors in interest to the covenantor be bound by the terms of the covenant. The requisite intent may be inferred from circumstances surrounding creation of the covenant.

This requirement is satisfied because the developer’s deed to each of the grantees contained a covenant requiring that the property be used only for single-family housing.

(ii) By virtue of the recording statutes, a subsequent purchaser of the promisor’s land must have actual, inquiry, or constructive (record)
notice of the arrangement at the time she purchased the land; otherwise, she is not bound.

Here, the daughter, the son, and the physical therapist have at least constructive notice of the restriction because it is in their chain of title. Because the electrician’s deed was never recorded, the plumber had a duty to inquire of the electrician where he obtained the property. Thus, the notice requirement is met.

(iii) The covenant must “touch and concern” the land; i.e., the performance of the burden must diminish the landowner’s rights, privileges, and powers in connection with her enjoyment of the land. The current owners’ rights as landowners are diminished because they cannot use their land to construct multifamily dwellings.
(iv) Finally, there must be horizontal and vertical privity.

Horizontal privity requires that, at the time the promisor entered into the covenant with the promisee, the two shared some interest in the land independent of the covenant.

The developer and each of the four original owners, as grantor and grantees, shared an interest in the land independent of the covenant.

Vertical privity exists when the successor in interest to the covenanting party holds the entire interest that was held by the covenantor at the time she made the covenant.

Here, the daughter, the plumber, and the physical therapist took the entire interest (fee simple absolute) from their predecessors. However, the son possesses only a life estate in the property, which is less than the fee simple absolute held by the flight attendant. Thus, vertical privity is lacking and the son cannot be bound by the covenant.

(A) is incorrect because the daughter is bound by the covenant as discussed above. The fact that she took the property by will is irrelevant. In fact, taking in this manner increases the chance that the daughter will be bound because it means she is not a bona fide purchaser for value and thus not protected by the notice requirement of the recording act.

(C) is incorrect because the plumber also is bound because he is charged with inquiry notice of the electrician’s unrecorded deed, which contained the singlefamily restriction.

(D) is incorrect because even though the physical therapist’s deed did not contain the restriction, she will be charged with constructive notice of it. The restriction appears in the developer’s deed to the athlete and thus is in the physical therapist’s chain of title. Hence, she also is bound by the covenant.

How well did you know this?
1
Not at all
2
3
4
5
Perfectly
93
Q

A homeowner owned a parcel of land on which she built a single-family residence. To pay for the construction, she obtained financing from a bank in exchange for a mortgage on the land. The bank promptly and properly recorded its mortgage. When the house was completed, except for the absence of an oven in the kitchen,
the homeowner leased the house to a tenant for
a three-year term. There was no provision in the lease agreement regarding kitchen appliances.
The homeowner bought an oven from an appliance company and had it installed in the space provided around the built-in cabinets in the
kitchen. To make the purchase, the homeowner
signed a security agreement with the appliance
company granting it a security interest in
the oven in exchange for financing. The appliance company did not file or record its security interest in the oven.
By the end of the lease term, the homeowner
was in serious default on her mortgage payments
to the bank and to the appliance company. In
preparing foreclosure proceedings against the
homeowner, the bank learned that the tenant was
planning to remove the oven and take it with
him when he moved out within the next few
weeks. The bank filed an action against the tenant claiming ownership of the oven, and joined the homeowner and the appliance company as parties.
Which party has a superior claim to the oven?
(A) The bank, because its mortgage interest
attaches to all fixtures on the real estate and
it has priority over the appliance company.
(B) The tenant, because removal of the oven
will not cause substantial damage to the
real estate.
(C) The homeowner, because the oven
was annexed to the real estate after the
mortgage was given.
(D) The appliance company, because it has
a valid security interest in the oven even
though it was not recorded.

A

(A) The bank will win because it recorded its mortgage on the property.

In a fixture case involving common ownership, the majority rule is that the annexor’s-intention test applies regardless of whether the owner makes the annexation before or after mortgaging the land.

The mortgage attaches to all fixtures on the real estate in the absence of an agreement to the contrary.

As between a mortgagee and the holder of a security interest in chattel affixed to the land, whichever interest is first recorded in the local real estate records wins.

While a purchase money security interest can gain priority if it is recorded within 20 days after affixation, the appliance company never filed.

(B) is wrong because it states the standard for divided ownership cases, i.e., cases in which the person who brings the chattel onto the land does not own the land. Here, the tenant has no interest in the oven; the competing interests are those of the homeowner and the creditors.

(C) is wrong because, as discussed above, the mortgagee’s interest applies to the homeowner’s fixtures regardless of the time of affixation.

(D) is wrong because, as discussed above, the appliance company did not record a fixture filing.

How well did you know this?
1
Not at all
2
3
4
5
Perfectly
94
Q

A landlord leased a vacant commercial
building to a tenant for a 10-year term. On
taking possession, the tenant installed a bar,
booths, special lighting, and a raised dance
floor. The bar and booths were simply placed on
the floor and were not secured to it. The lights
were installed by a qualified electrician and
were directly wired into the building’s electrical
system. The special dance floor was bolted to
the building’s cement floor by installation crews
from the company that made the dance floor.
After installation of the above, the tenant then
operated her business for almost 10 years. At
that time, she decided that business was so good
that she would move to a larger space down the
street. She told the landlord that she would not
be renewing her lease and that she would be
removing the lights, booths, bar, and dance floor.
The landlord told her that none of the installations
could be removed because they are now
part of the building.
If a court were called upon to resolve this
dispute, how would it likely rule?
(A) The tenant may remove the dance floor,
bar, and lights, but not the booths.
(B) The tenant may remove the bar and the
booths, but not the dance floor and the
lights.
(C) The tenant may remove the bar, booths,
dance floor, and lights.
(D) The tenant may not remove the bar, booths,
dance floor, or lights.

A

(C) All of the items listed may be removed either because they were not affixed to the real property or because they were used in the tenant’s business and thus may properly be termed trade fixtures. As long as the tenant can remove trade fixtures with little damage to the real property,
the tenant may always take them when the tenant moves.

(A) is incorrect. For a chattel to be deemed a fixture (and thus remain as a part of the realty) it must be affixed to the real property.
The booths, along with the bar, were not even attached to the realty; thus, they cannot under any circumstances be considered fixtures, and the tenant may always take them.

(B) is incorrect. If these were not trade fixtures, an argument could be made that the tenant could take the items not affixed (bar and booths) but not the ones affixed (lights and dance floor). But even items strongly attached to the realty may be detached by the tenants who installed them if, as here, they are tools of the trade.

(D) is incorrect. The booths and bar can be removed because they were not affixed
to the building, and even though the lights and dance floor were attached, they can be removed as trade fixtures.

How well did you know this?
1
Not at all
2
3
4
5
Perfectly
95
Q

An easement can be abandoned provided two things are satisfied:

A
  • nonuse of the easement
  • together with some affirmative act taken on the property which would indicate an intent to abandon the easement.
  • Mere nonuse would not be enough, no matter how long the easement was not used. It is the act taken on the property that shows the intent to abandon that is crucial.
How well did you know this?
1
Not at all
2
3
4
5
Perfectly
96
Q

…. , has the right to choose the location of an easement by necessity.

A

The holder of the servient estate

The owner of the servient parcel has the right to locate the easement, provided the location is reasonably convenient.

How well did you know this?
1
Not at all
2
3
4
5
Perfectly
97
Q

The doctrine of equitable conversion places the risk of loss on …. as soon as the enforceable contract is entered into.

A

the purchaser

How well did you know this?
1
Not at all
2
3
4
5
Perfectly
98
Q

The fee simple owner of an unimproved
parcel of wooded land orally agreed to sell it
to a buyer under an installment land contract.
The buyer agreed to pay $5,000 down and $100
a month for the next 10 years, and the owner
would retain the deed until the buyer finished
paying the installments.
After making the down payment, the buyer
moved onto the property and began clearing
some of the trees for a road and a cabin. He
regularly made the payments for several months
but then was killed by a falling tree. His properly
executed will conveyed his real estate to a friend
and the remainder of his estate, consisting of
personal property and $200,000 in cash, to his
son. During the next several months, his estate
failed to make payments on the installment
contract. The owner then notified the estate that
he was rescinding the deal and asserting ownership of the parcel, and offered to return the
amount the buyer had paid him, less expenses, as restitution.
The buyer’s estate initiated a quiet title action,
naming the owner, the friend, and the son as
parties. The estate’s filings indicated that it
was prepared to complete the conveyance and
redeem the land by paying the accelerated full
balance of the contract from the proceeds of the
estate. The friend believes he should receive title
to the parcel free of any obligation on the installment contract.
Which of the following doctrines is inapplicable?
(A) The doctrine of equitable conversion.
(B) The doctrine of part performance.
(C) The equitable mortgage doctrine.
(D) The doctrine of exoneration.

A

(C) The court will not apply the equitable mortgage doctrine in making its determination.

A landowner needing to raise money may “sell” the land to a person who will pay cash, giving the
“buyer” an absolute deed rather than a mortgage. If the court concludes that the deed was really given as security for a loan, rather than a true conveyance, it will treat the deed as an “equitable” mortgage and require that the lender foreclose it by judicial action, like any other mortgage.

Here, there was no such security arrangement between the owner and the buyer; rather, they entered into an installment land contract arrangement. Thus, the equitable mortgage doctrine would not support the court’s determination that the friend receive title to the parcel.

(A) is incorrect because equitable conversion is applicable here.

Under that doctrine, once a contract is made
and each party is entitled to specific performance, equity regards the purchaser as the owner of the real property
. If the purchaser dies before title has passed, his interest is characterized as real property in his estate and will go to the taker of the estate’s real property on closing, which in this case is the friend.

(B) is incorrect. Although the Statute of Frauds applies to land sale contracts, the doctrine of part performance allows a court to order specific performance despite the absence of a writing if additional facts are present. In most jurisdictions, part performance can be established by two of the following:

(i) possession of the land by the purchaser;

(ii) making of substantial improvements; and/or (iii) payment of all or part of the purchase price.

Here, the buyer’s estate could be entitled to specific performance of the contract because he had (i) moved onto the property, (ii) begun clearing trees, and (iii) made several payments before his death.

(D) is incorrect. The court must apply the doctrine of exoneration, which is followed by a few states,
for the friend to receive the parcel free of the obligation on the installment contract.

The doctrine provides that when a testator makes a devise of real property that is subject to a mortgage, the devisee is entitled to have the mortgage satisfied out of the testator’s residuary estate.

Thus, if the court applies exoneration, it will order that the mortgage be paid from the $200,000 cash residuary that was left to the buyer’s son.

How well did you know this?
1
Not at all
2
3
4
5
Perfectly
99
Q

A seller entered into a written contract to
sell a tract of land to a buyer. The buyer was to
pay $1,500 per month for five years, at which
time the seller would deliver a warranty deed.
The contract was silent as to the quality of title
to be conveyed. After making 12 payments,
the buyer discovered that a neighbor had an
easement of way over the land, which was not
discussed at the time the seller and buyer entered into the contract. The neighbor had not used the easement over the previous year because she had been out of the country. On the basis of the easement, the buyer wishes to cancel the contract.
Which party is more likely to prevail?
(A) The seller, because the neighbor’s easement
has been extinguished.
(B) The seller, because the buyer has no basis
on which to rescind the contract.
(C) The buyer, because the obligation to convey
marketable title is implied.
(D) The buyer, because the seller has breached
the covenant against encumbrances.

A

(B) The seller is more likely to prevail because the buyer has no basis on which to rescind the
contract.

Absent a provision to the contrary, a contract for the sale of land contains an implied promise by the seller that she will deliver to the buyer a marketable title at the time of closing.
This promise imposes on the seller an obligation to deliver a title that is free from reasonable
doubt; i.e., free from questions that might present an unreasonable risk of litigation. Title is
marketable if a reasonably prudent buyer would accept it in the exercise of ordinary prudence.
An easement that reduces the value of the property (e.g., an easement of way for the benefit of a neighbor) generally renders title unmarketable.

If the buyer determines, prior to closing, that the seller’s title is unmarketable, he must notify the seller and allow a reasonable time to cure the defect.

If the seller is unable to acquire title before closing, so that title remains unmarketable, the buyer can rescind, sue for damages caused by the breach, or obtain specific performance with an abatement of the purchase price.

However, the buyer cannot rescind prior to closing on grounds that the seller’s title is unmarketable.

Where an installment land contract is used, the seller’s obligation is to furnish marketable title when delivery is to occur, e.g., when the buyer has made his final payment.

Thus, a buyer cannot withhold payments or seek other remedies on grounds that the seller’s title is unmarketable prior to the date of promised delivery.

Here, there is a valid easement on the property, but the seller has four years in which to cure this defect.Thus, the buyer cannot yet rescind on grounds that title is unmarketable.

How well did you know this?
1
Not at all
2
3
4
5
Perfectly
100
Q

A landowner devised her campground in her
will “to my niece, her heirs, and assigns, so
long as it is used for camping and recreational
purposes; if used for any other purpose during
her lifetime,
then to the Girl Scouts of America.”
Subsequently, the landowner died. The residuary
clause of her will left all property not devised
in the remainder of the will to her daughter and
sole heir. Soon thereafter, the daughter died
intestate, her only heir being her son. Last month, the niece entered into a contract to sell the campground to a buyer for its reasonable
market value. After the buyer received the
title report called for in the contract, he refused
to proceed with the purchase, claiming that the
niece could not convey good title. The niece,
the Girl Scouts of America, and the buyer then
execute a new contract calling for the former
two parties to sell the property at the same
price to the latter. The jurisdiction follows the
common law Rule Against Perpetuities, and a
statute provides that future estates and interests
are alienable, and may be devised or inherited,
all in the same manner as possessory estates or
interests.
Should the buyer proceed with the new
purchase transaction?
(A) Yes, because good title can now be conveyed
by the sellers.
(B) Yes, if the Girl Scouts of America promises
never to use its right of entry should the
buyer use the property for other than
camping and recreational purposes.
(C) No, because the daughter’s son has not been
included as a party selling the property.
(D) No, because no one can convey good title to
the property during the niece’s lifetime.

A

(C) Good title cannot be obtained without the daughter’s son’s inclusion in the conveyance.

All contracts for the sale of land contain, unless the contract expressly provides otherwise, an implied covenant by the seller that she will deliver to the buyer a marketable title at the date of closing.
Private restrictions or encumbrances, including executory interests and possibilities of reverter,
will render title unmarketable unless the holders of those interests join in the transaction.

Here, the Girl Scouts have a valid executory interest that does not violate the Rule Against Perpetuities because their interest must vest, if at all, during the niece’s lifetime. However, the niece’s fee simple determinable is not limited in duration like the interest of the Girl Scouts. Thus, the niece’s heirs will only have a fee simple determinable rather than a fee simple absolute. Because the Girl Scouts’ interest vanishes on the niece’s death, and no other provision was made for the property if it should thereafter be used for noncamping or nonrecreational purposes, the transferor (the landowner) retained a possibility of reverter. This interest passed to the daughter and then to the daughter’s son, and so the son’s interest must be included in the conveyance.

(A) is wrong because the buyer has contracted to receive good title to a fee simple absolute without any restrictions on the use of the property. Unless the son joins in the contract to convey his possibility of reverter, good title cannot be conveyed.

(B) is wrong because the son’s interest is the one blocking good title; the Girl Scouts have already agreed to sell their interest. Furthermore, their interest is not a right of entry but a shifting executory interest that would automatically divest the buyer’s interest if he violated the use restriction, and so their promise would be meaningless.

(D) is wrong because the niece, the Girl Scouts, and the daughter’s son together can validly convey a fee simple absolute to the buyer.

How well did you know this?
1
Not at all
2
3
4
5
Perfectly
101
Q

Reformation is ….

A

… the remedy whereby the writing setting forth the agreement between the parties is changed to make it conform to the original intent of the parties.

Reformation may be available where there is a
mutual mistake (i.e., the writing does not conform to the original agreement and the parties are
not aware of the discrepancy).

As long as the parties were in agreement as to the terms before the contract was reduced to writing, reformation can be had regardless of whether both parties signed the contract without noticing the deviation from the oral agreement or one party knew of the deviation and the other did not.

How well did you know this?
1
Not at all
2
3
4
5
Perfectly
102
Q

Because land is considered unique,
specific performance is always appropriate for the enforcement of a valid land sale contract. This
option is available to

A

either the buyer or the seller.

How well did you know this?
1
Not at all
2
3
4
5
Perfectly
103
Q

When a contract for the sale of land is signed, equitable conversion takes place and it is, for all practical purposes, ….

A

the buyer’s land and the buyer’s risk.

How well did you know this?
1
Not at all
2
3
4
5
Perfectly
104
Q

A seller received a written offer in the mail
signed by a buyer to purchase the seller’s land for $50,000. The written offer was legally sufficient to form a written contract for the sale of the land.
The seller called the buyer and said that the offer
was acceptable, but that she wanted her attorney
to review it. The seller asked her attorney to
prepare a formal contract for the sale of the
land according to the same terms and conditions
in the written offer. When the attorney
had finished, the seller signed the contract and
mailed it to the buyer. Later that day, before the
buyer had received the contract, a developer
called the seller and offered to buy the land for
$60,000, which the seller accepted immediately
over the phone. The seller called the buyer and
told him that she had received and accepted a
higher offer. The seller then signed a written
contract to sell the land to the developer. When
the developer received the contract, he signed
it and then promptly and properly recorded it,
and sent the seller the specified down payment.
The buyer received the written contract from the
seller the next day. The recording statute in the
jurisdiction provides: “Any conveyance of an
interest in land, other than a lease for less than
one year, shall not be valid against a subsequent
purchaser for value, without notice thereof,
whose conveyance is first recorded.”
In an appropriate action brought by the buyer
against the seller and the developer for specific
performance and to quiet title, will the buyer
win?
(A) Yes, because the buyer’s written offer satisfies
the Statute of Frauds.
(B) Yes, because the contract of sale prepared
by the attorney satisfies the Statute of
Frauds.
(C) No, because the buyer never entered into a
binding contract with the seller.
(D) No, because the recording statute protects
the developer.

A

(D) The buyer will lose because the race-notice recording statute protects the developer.

The developer paid a fair price for the land and had no knowledge of the buyer’s claim to the land at the time he purchased the property. He would thus qualify as a bona fide purchaser for value and, because he was the first to record, he would have priority over the buyer.

(A) and (B) are incorrect because the recording act determines priority among purchasers of property while the Statute of Frauds deals only with the validity of an individual contract.

(C) is incorrect because _the seller’s
signing the contract, which contained the same terms and conditions as the buyer’s offer, *constituted an acceptance*, which became effective upon dispatch under the “mailbox rule” of contract law.
_

How well did you know this?
1
Not at all
2
3
4
5
Perfectly
105
Q

A seller contracted to convey her property to
a buyer for $75,000. A title search revealed the
following:

(i) There were 25 years left on a lease of the property, which was recorded. The buyer
agreed to take title subject to the lease but was
not aware that the lease gave the lessee, his heirs, and assigns an option to purchase the land.

(ii) The roof of the garage on the property extended approximately one-half inch across the property line into the airspace of an adjoining neighbor. The garage did not interfere with any current or future use of the adjoining lot.
(iii) The home on the property was subject to a $5,000 lien arising from a dispute involving some remodeling work. The seller promised to pay off the lien at closing with the proceeds from the sale.
(iv) The property was subject to an easement by necessity in favor of the adjoining neighbor. Last month, the city extended the main road to the neighbor’s land, but the neighbor planned to continue to use the easement because it was more convenient.

In a jurisdiction that has a standard racenotice recording statute and maintains the common law Rule Against Perpetuities without any modern statutory reformation, which encumbrance renders the seller’s title unmarketable?
(A) The lessee’s option.
(B) The encroachment of the garage’s roof.
(C) The $5,000 lien.
(D) The easement.

A

(A) The lessee’s option to purchase renders the seller’s title unmarketable. Ordinarily, an option of this duration would violate the Rule Against Perpetuities and be stricken. An interest violates the Rule Against Perpetuities if there is any possibility, however remote, that it will vest more than 21 years after some life in being at the creation of the interest. Here, the relevant measuring life would be the lessee. The lessee could die within the next three years, and the option could be exercised by his successors more than 21 years after his death. There is a special exception to the Rule, however, for options to purchase attached to leaseholds. Because the one who holds the option in this case is the current lessee, the Rule does not apply. Thus, the option is valid, and it renders the
seller’s title unmarketable.

(B) is incorrect because only a significant encroachment will render title unmarketable. A one-half-inch encroachment on airspace would not be considered significant, particularly because it does not interfere with the use of the adjoining property. Thus, this encroachment will not affect marketability.

(C) is incorrect because a lien on property will not
render title unmarketable if the seller pays the lien at closing. Unless the contract provides otherwise, the seller need not provide marketable title until closing. A seller has the right to satisfy a lien at the closing with the proceeds of the sale. Therefore, as long as the purchase price is sufficient and the lien is satisfied simultaneously with the transfer of title (e.g., by using escrows), the buyer cannot claim that the title is unmarketable. The closing will result in marketable title. In this situation, the $75,000 purchase price is clearly sufficient to satisfy the $5,000 lien. Thus, the seller may satisfy the lien at the closing and convey marketable title to the buyer.

(D) is incorrect because although an easement that reduces the value of the property renders title unmarketable, this easement has been terminated. An easement by necessity terminates as soon as the necessity ends. Because the main road now provides access to the neighbor’s land, she no longer needs the
easement over the seller’s land and it is extinguished.

106
Q

A landowner gratuitously conveyed his
interest in land to a friend by quitclaim deed.
The friend promptly and properly recorded her
deed. Six months later, the landowner conveyed
his interest in the same land to an investor for
$50,000 by warranty deed, which was promptly
and properly recorded. As between the friend and the investor, who has the superior right to title to the land?
(A) The friend, regardless of the type of recording
statute.
(B) The friend, because she recorded prior to
the investor’s recording.
(C) The investor, regardless of the type of
recording statute.
(D) The investor, because it took by warranty
deed rather than quitclaim deed.

A

(A) Because the friend recorded prior to the subsequent conveyance, she has the superior right to title regardless of the type of recording statute.

A conveyance that is recorded can never be divested by a subsequent conveyance through operation of the recording statutes.

By recording, the grantee gives constructive (or “record”) notice to everyone. Hence, proper recording prevents anyone from becoming a subsequent bona fide purchaser (“BFP”).

Because the landowner’s conveyance
to the friend was recorded at the time of the landowner’s conveyance to the investor, the investor cannot prevail.

The investor will clearly lose under a pure race statute because the friend recorded first.

The investor will also lose under notice and race-notice statutes because the conveyance to the friend was recorded at the time of the conveyance to the investor. The investor, therefore, had record notice and cannot claim the protection that these types of statutes provide for subsequent purchasers for value who take without notice. Thus, (A) is correct and (C) is incorrect.

The fact that the friend is merely a donee rather than a BFP does not mean that her recording has no effect. It is only the subsequent taker who has to be a BFP rather than a donee to utilize the recording statute. The prior grantee, regardless of her status, protects her interest by recording because it prevents anyone from becoming a subsequent BFP.

(B) is incorrect because, as noted above, the friend will prevail under any type of recording act, but not necessarily because she recorded prior
to the investor’s recording. If the jurisdiction has a notice statute, whether the friend recorded
prior to the investor’s recording is irrelevant. Rather, it is the fact that the friend recorded prior
to the investor’s purchase that gives the friend superior title in a notice jurisdiction, because the
investor would have record notice of the conveyance and thus would not qualify as a BFP. (D) is incorrect because the quitclaim/warranty deed distinction does not affect who has title to the land; that status merely affects the parties’ respective causes of action and ability to recover against the landowner.

107
Q

A landowner conveyed his land to a famous
guitarist. The guitarist put the deed in his
guitar case and took off for a three-week band
tour. While the guitarist was out of town, the
landowner offered to sell the same land to his
neighbor for $5,000. Although the neighbor
knew that the guitarist had already bought the
land, the neighbor paid the landowner $5,000
and promptly recorded the deed she received.
Thereafter, the neighbor conveyed the land to her
ex-husband for $15,000. The ex-husband knew
nothing about the guitarist’s deed and promptly
recorded the deed he received. Two weeks later,
the guitarist returned and recorded his deed to
the land. A month after that, the ex-husband
conveyed the land to a buyer for $17,000. The
buyer knew that the guitarist held a deed to the
land, but paid the ex-husband $17,000 anyway.
The buyer immediately recorded and filed an
appropriate action against the guitarist and the
ex-husband to determine ownership of the land.
The land is situated in a state with the following
statute: “No conveyance or mortgage of an
interest in land is valid against any subsequent
purchaser for value without notice thereof whose
conveyance is first recorded.”
How will the court most likely rule?

A

(C) The buyer has superior rights to both the guitarist and the ex-husband.

Under a race-notice statute such as the one in this question, a subsequent purchaser is protected only if he purchases without
notice of prior conveyances and records first. However, even though the neighbor had notice of the guitarist’s deed and was therefore not a bona fide purchaser, and could not have prevailed against the guitarist, that does not mean that a bona fide purchaser from her would not prevail against the guitarist. The ex-husband was a subsequent bona fide purchaser from the neighbor and recorded before the guitarist. Thus, the ex-husband would prevail over the guitarist. The buyer, who had
notice of the guitarist’s deed when she subsequently acquired her deed from the ex-husband, will also prevail over the guitarist. This is due to the “shelter doctrine,” which is applied to the recording acts. Under that doctrine, once a bona fide purchaser enters the chain of events, he can subsequently deed the land to a party who had notice of the prior deed and that party is also accorded the status of a bona fide purchaser. Thus, once the ex-husband, a bona fide purchaser, enters the picture and gets the protection of the act, he can sell to the buyer, who gets the protection of the act even though she purchased with notice. The buyer comes under the shelter doctrine.
_The theory behind the shelter doctrine is that it extends full protection to bona fide purchasers
in that it does not limit their ability to market the property._Otherwise, if they could not sell to
parties with notice, their ability to market the property would be impaired.

  • *The exception to the shelter doctrine is that it does not apply to the first purchaser with notice—the neighbor.**
  • Thus, if the ex-husband had subsequently sold the land back to the neighbor, she would not have been accorded the status of a bona fide purchaser.* The reason for the exception is obvious—to discourage fraud. Note also that because the buyer has a deed from the ex-husband, she prevails over the ex-husband.
108
Q

A homeowner agreed to sell his home to an
accountant. He gave the accountant a general
warranty deed and the accountant gave him
$86,000, his asking price. The deed was
recorded. A few years later, the accountant sold
the property to a doctor, who paid her $125,000
for the property. To save on attorney’s fees,
the accountant went to her local office supply
store to purchase a general warranty deed
form. The store was out of those forms, but the
clerk suggested that she use the form labeled
“Quitclaim Deed,” asserting that it would
transfer the title just as well as the other form.
The accountant purchased the form and filled in
the blanks with the appropriate information she
copied from her old deed. The doctor did not
hire a lawyer to represent him in the purchase of
the house.
The doctor accepted the deed from the
accountant and gave her $125,000. Soon after
the doctor moved into the house, it was discovered that the homeowner’s title was not good.
The true owner now demands that the doctor
vacate. Title is judicially determined to be with
the true owner, and the doctor is forced out.
Does the doctor have any action against the
homeowner or the accountant based on any
covenant for title?
(A) Yes, the doctor can sue both and can recover
$125,000.
(B) Yes, the doctor can sue the homeowner, but
not the accountant, and can recover his full
$125,000.
(C) Yes, the doctor can sue the homeowner, but
not the accountant, and can recover only
$86,000.
(D) No, the doctor can sue neither the
homeowner nor the accountant.

A

(C) The doctor can sue the homeowner but can recover only $86,000.

The homeowner gave a warranty of title to the accountant when he gave her a general warranty deed. This warranty runs with the land and can be enforced by any subsequent purchaser. _Damages are limited, however, to
the purchase price received by the warrantor.
_Because the homeowner received only $86,000 from the accountant, that is all that the doctor can recover. The doctor gets nothing from the accountant because she gave only a quitclaim deed, which gives no warranty of title.

109
Q

A retiree contracted to purchase her
hometown diner, which was struggling. She
borrowed the $80,000 purchase price from a
local bank, granting the bank a mortgage on
the diner. Due to a clerical error, the bank’s
mortgage was not recorded. Business remained
very slow after the purchase, so the retiree
decided to give the diner a makeover. To finance
the renovations, she applied for a $20,000 loan
from a credit union, offering to secure this
debt with a mortgage on the diner as well. To
encourage this opportunity, the bank executed an
agreement to subordinate its interest to the credit
union’s mortgage. The credit union then loaned
the retiree the $20,000 but never recorded its
mortgage. Several months later, the bank discovered
its clerical error and properly recorded its
mortgage on the diner. Unfortunately, the diner’s
final renovations were garish and brought in few
customers. The retiree now has defaulted on both
mortgages. A statute in the jurisdiction provides:
“No unrecorded conveyance or mortgage of
real property shall be good against subsequent
purchasers for value without notice, whose
conveyance is first recorded.”
Whose mortgage has priority?
(A) The credit union’s, because the bank’s
mortgage was unrecorded when the credit
union’s mortgage was executed.
(B) The credit union’s, because the bank agreed
to subordinate its interest.
(C) The bank’s, because the credit union never
recorded its interest.
(D) The bank’s, because a purchase money
mortgage is senior to all competing liens.

A

(B) The credit union’s mortgage has priority because the bank agreed to subordinate its interest.
Priority among mortgages on the same real estate is normally determined by chronology.
_However, this may be modified by the operation of a recording act, special rules governing
purchase money mortgages, or the execution of a subordination agreement._A first mortgagee
may enter into an agreement with a junior mortgagee to subordinate its priority to the junior
mortgagee, and such agreements generally are enforced if they are specific.

(C) is incorrect because a subordination agreement will determine priority despite the existence of a recording act. This jurisdiction has a race-notice statute, under which a subsequent bona fide purchaser (i.e., a person, including a mortgagee, who gives valuable consideration and has no notice of the prior instrument) prevails over a prior grantee only if he records before the prior grantee records.
Here, only the bank recorded its interest. Thus, the bank would prevail under the recording act
if there were no subordination agreement.

(A) would state the priority rule for a notice jurisdiction, in which a subsequent bona fide purchaser prevails over a prior grantee regardless of whether the subsequent purchaser records. However, a valid subordination agreement would govern in a notice jurisdiction as well. _Furthermore, the credit union had notice of the bank’s mortgage here
because the bank entered into a subordination agreement with the credit union.
_

(D) is incorrect.

A purchase money mortgage, given when the mortgagor buys the property, has priority over
non-purchase money mortgages that arise prior to the mortgagor’s acquisition of title. However,
priority can be defeated by subsequent mortgages or liens through operation of the recording acts, and it can be relinquished through a subordination agreement.

110
Q

An owner of a parcel of land instructed
his lawyer to draw up an instrument deeding
the land to his friend’s “nieces.” _The owner
acknowledged the deed before a notary and
signed it. As directed by the owner, the lawyer
recorded the deed and then returned it to the
owner.
_ The owner put the deed in the drawer of
his desk, intending to present it to the friend’s
nieces when they came to visit him next month.
The following week, however, the owner died,
leaving his daughter as his sole heir at law. The
daughter discovered the deed to the land in the
owner’s desk. She filed an appropriate action to
quiet title in the land, naming the friend’s only
two nieces as defendants. The only evidence
presented at the trial was the deed itself, the
evidence of recordation, and the lawyer’s testimony regarding the owner’s intent.
Who should the court rule owns the land?
(A) The nieces, because recordation of a
notarized deed is prima facie evidence of
delivery.
(B) The nieces, because a deed is prima facie
valid absent evidence to the contrary.
(C) The daughter, because the evidence is
insufficient to support a valid delivery.
(D) The daughter, because the grantees in the
deed are too indefinite.

A

(A) The nieces own the land because recordation is prima facie evidence of delivery.

  • *To be valid, a deed must be “delivered,” which means that the grantor must have taken some action (not necessarily a manual handing over of the deed) with the intent that it operate to pass title immediately.**
  • *Recording a deed that has been acknowledged before a notary is such an action and is presumed to carry with it the requisite intent.** Even without the knowledge of the grantee, delivery to the recorder’s office will satisfy the delivery requirement. If the grantor intends the recording of the document to be the final act in vesting title in the grantee, then such recording constitutes delivery.
111
Q

The state in which the land is located has the
following statute: “Any conveyance of an estate
in land, other than a lease for less than one
year, shall not be valid against any subsequent
purchaser whose conveyance is first recorded.”

What is this statute?

A

pure race recording jurisdiction - notice is irrelevant; whoever records first has priority.

112
Q

To satisfy a debt owed to a creditor, a son
executed and delivered to the creditor a warranty
deed to a large tract of undeveloped land. The
creditor promptly recorded the deed. Shortly
thereafter, she built a house on the property and
has lived there ever since. The son never actually
owned the land. It belonged to his father, but the
father had promised to leave the property to the
son.
Later, the father died and his will devised the
property to the son. Pressed for money, the son
then sold the land to an investor by warranty deed,
which the investor promptly recorded. Although
the investor paid full value for the property, he
purchased it strictly for investment and never
visited the site. He therefore did not realize that
the creditor was living there, and knew nothing of
the son’s earlier deed to the creditor.
The jurisdiction in which the land is located
has the following statute: “A conveyance of an
estate in land (other than a lease for less than
one year) shall not be valid against any subsequent
purchaser for value without notice thereof
unless the conveyance is recorded.”
Which of the following is the most likely
outcome of a quiet title action brought by the
creditor against the investor?
(A) The creditor prevails, because the son had
no title to convey to the investor.
(B) The creditor prevails, because the investor
was not a purchaser for value without notice
of the creditor’s interest.

A

(B) The creditor will prevail in a suit to quiet title because the investor had notice of the creditor’s
interest in the property and, thus, is not a bona fide purchaser for value.

When a grantor purports to convey property that he does not own, his subsequent acquisition of title to that property vests in the grantee under the doctrine of estoppel by deed.

_Most courts, however, hold that this is *personal estoppel,* which means that title inures to the grantee’s benefit only as against the grantor, not a subsequent bona fide purchaser._ If the grantor transfers his after-acquired title to an innocent purchaser for value, the bona fide purchaser gets good title.

There is a split of authority as to whether the original grantee’s recordation of the deed imparts sufficient notice to prevent a subsequent purchaser from being a bona fide purchaser, but the majority view is that it does not because it is not in his chain of title. Thus, it is not the fact that the creditor recorded that prevents the
investor from being a bona fide purchaser. The fact that the creditor built a home and was living
on the property gave the investor constructive notice of her interest. A title search is not complete without an examination of possession. If the possession is unexplained by the record, the subsequent purchaser is charged with knowledge of whatever an inspection of the property would have disclosed and anything that would have been disclosed by inquiring of the possessor. Therefore, the investor is charged with knowledge of the creditor’s possession and with what the creditor would have told him about her possession; i.e., that the property was conveyed to her by the son prior to his conveyance to the investor. Consequently, the investor does not qualify as a bona fide
purchaser.

(A) is incorrect because, although the son is estopped to deny that he acquired title for the benefit of the creditor, he could have conveyed valid title to a subsequent purchaser for value who had no notice of the creditor’s interest. Therefore, it is not exactly correct to say that the son had no title to convey.

113
Q

victim obtained a judgment against a tortfeasor
for $100,000. The tortfeasor, who had few
assets, did not pay the judgment. On April 1
of the following year, the tortfeasor inherited
a parcel of land from her uncle. On May 1, the
tortfeasor entered into a contract with a buyer
to sell the land for $120,000. The contract was
not recorded. The buyer immediately applied
to a bank for a loan of $100,000. The bank
approved the buyer’s loan, and on May 15, a
closing was held. The tortfeasor deeded the land
to the buyer, and the buyer executed a mortgage
for $100,000 to the bank. Due to an error by the
title company, the deed from the tortfeasor to the
buyer was not recorded, although the mortgage
to the bank was recorded. Neither the buyer
nor the bank had any knowledge of the victim’s
judgment. On May 20, the victim recorded his
judgment in the county recorder’s office where
the land was located. At that time, he had no
knowledge of the buyer’s or the bank’s rights.
When he learned about them, he immediately
brought a proceeding to foreclose his judgment
lien, naming the tortfeasor, the buyer, and the
bank as parties.
The jurisdiction has a typical grantor/grantee
recording index, and has enacted the following
statute:
Any judgment properly filed in the county
recorder’s office shall, for 10 years from filing,
be a lien on the real property then owned or
subsequently acquired by any person against
whom the judgment is rendered. No conveyance
or mortgage of real property shall be good
against subsequent bona fide purchasers for
value and without notice unless the same be
recorded according to law.
As between the victim and the bank, which
party’s interest in the land will be given priority?
(A) The bank, because the bank recorded its
mortgage before the victim recorded his
judgment lien.
(B) The bank, because the victim is not
protected by the recording statute.
(C) The victim, because the victim’s judgment
was filed in the recorder’s office before the
buyer’s deed was recorded.
(D) The victim, because the judgment lien
extends to after-acquired property.

A

(B) The victim will not likely prevail against the bank because a majority of courts hold that the
judgment lienor is not protected by the recording statute.

If the statute here, which is a notice statute, were applicable to protect the victim, he would have priority over the bank because his judgment lien was recorded before the buyer’s deed was recorded. Under this view, the bank’s mortgage would have been considered “wild” and would be deemed unrecorded because the preceding conveyance, the buyer’s deed, was actually unrecorded. A searcher in the public records
would therefore have been unable to find the mortgage. Hence, if the statute were applicable to protect the victim, he would have priority over the bank.

However, most courts reason that either
(i) a judgment creditor is not a bona fide purchaser because he did not pay contemporaneous value for the judgment, or

(ii) the judgment attaches only to property “owned” by the debtor, and not to property previously conveyed away, even if that conveyance was not recorded. Under the statute in the present question, a judgment does not attach until it is recorded.

Here, the victim’s judgment did not attach to the land until after the bank obtained a mortgage on it, and the recording statute does not change that result. The failure of the buyer to record, and the resultant treatment of the bank as unrecorded, is irrelevant. Thus, the bank’s mortgage is superior to the victim’s lien.

114
Q

A landowner owned a large tract of undeveloped
land in fee simple. Although no excavation
had been done on the land, it was believed to
contain gold. The landowner therefore began
to mine the land, financing his operation with
a $100,000 mortgage to a bank. Subsequently,
the landowner sold all of the interest in gold
on the land to a miner. Shortly thereafter, the
landowner conveyed his ownership in the land to
a mining company. Realizing that none of their
interests had been recorded, the bank recorded
its mortgage first, the miner recorded her deed
second, and the mining company recorded its
deed third. None of the parties dealing with the
landowner had any knowledge of the others at
the time of their transactions.
The jurisdiction in which the land is located
has the following statute: “No conveyance or
mortgage of an interest in land is valid against
any subsequent purchaser for value without
notice thereof, unless it is recorded.”
If the mining company brings an action to
quiet title in the land, what is the most likely
result?
(A) Because the rights of the bank, the miner,
and the mining company are different in
nature, the court would most likely validate
all of the interests, with the mining company
having the ownership subject to the
payment of the mortgage to the bank.
(B) The mining company would be successful
in quieting title to the land.

A

(B) The mining company would likely be successful.

Under a notice statute, which the jurisdiction
in this question has, a subsequent bona fide purchaser prevails over a prior grantee who fails to record. The important fact under a notice statute is that the subsequent purchaser had no actual or constructive notice at the time of the conveyance, not at the time of recording.

When the property was conveyed to the mining company, it had neither actual nor constructive notice of the conveyances to the bank or the miner, whose interests were not recorded at that time. Therefore, the mining company was a bona fide purchaser and would be entitled to protection under the statute.

(A) is incorrect because _the recording statute applies to all conveyances and mortgages of
an interest in land, including a conveyance of the mineral interests.
_Thus, both the bank’s and the miner’s interests are not enforceable against the mining company.

115
Q

A valid deed requires:

A
  • a writing containing a description of the land and parties,
  • words of intent, and
  • the grantor’s signature.
116
Q

A defective deed may be voidable,
which means that it would be set aside only if the property had not been conveyed to a bona fide
purchaser,

or it could be void, meaning that the deed would be set aside regardless of the property
having passed to a bona fide purchaser.

A

Deeds obtained by means of, among other things, duress, undue influence, or mistake are considered voidable.

Deeds that were forged, never delivered,
or obtained by fraud in the factum are void.

117
Q

A deed, once it has been delivered, is …

A

… merely evidence of title, and its destruction
does not cause any change in the title.

118
Q

To buy a house, an investor secured a $10,000
mortgage from a bank. The bank promptly and
properly recorded its mortgage. Subsequently,
the investor financed certain improvements to
the house with a $2,000 mortgage on the land
from a finance company. The finance company
promptly and properly recorded its mortgage.
Before the investor made a payment on either
mortgage, the federal government announced
that it would begin storing nuclear waste
products in the area. The value of property,
including the investor’s house, plummeted.
The investor did not pay either the bank or the
finance company, and the bank brought a proper
action to foreclose, notifying both the investor
and the finance company. A buyer bought the
house at the foreclosure sale for $6,000, which
was its fair market value. There are no special
statutes in the jurisdiction regarding deficiency
judgments.
What does the investor owe?
(A) $5,000 to the bank and $1,000 to the finance
company.
(B) $4,000 to the bank and $2,000 to the
finance company.
(C) Nothing to the bank and $2,000 to the
finance company.
(D) $4,000 to the bank and nothing to the
finance company.

A

(B) A bsent any anti-deficiency statutes, the investor remains personally liable to pay for any shortfall arising from the foreclosure sale. Proceeds from the sale are used to satisfy the loan that was foreclosed first. Hence, all of the proceeds ($6,000) went to the bank. Thus, the investor must pay the balance still due the bank ($4,000) and the entire amount of the finance company’s mortgage ($2,000), which is terminated by the foreclosure of the senior mortgage.

(A) is wrong because foreclosure sales are not allotted proportionally between senior and junior interests.

(C) is wrong because foreclosure does not extinguish the underlying debt.

(D) is wrong because the finance
company’s mortgage does not remain on the land after foreclosure of the senior mortgage; hence, the investor is liable for that debt as well.

119
Q

When a grantee assumes the mortgage, …

A

…. the grantee expressly promises the grantor-mortgagor that he will pay the mortgage obligation as it becomes due.

The mortgagee then becomes a third-party beneficiary of the grantee’s promise to pay and can sue the grantee directly if the grantee fails to pay.

After the assumption, the grantor-mortgagor becomes a surety who is secondarily liable to the mortgagee on the note if the grantee fails to pay.

120
Q

A buyer purchased a tract home in a new
development, putting up 10% of the purchase
price as a down payment and financing the rest
through a mortgage with a bank. After four
years, the buyer put her house on the market,
continuing to make all mortgage payments promptly. The buyer eventually sold the house to
a third party, who took subject to the mortgage.
After the third party took possession, the bank
received no further mortgage payments from
either the buyer or the third party. In most states, which of the following best describes the remedy or remedies available to the bank?
(A) The bank may foreclose on the land, but
may not sue either the buyer or the third
party on the underlying debt.
(B) The bank may foreclose on the land, or it
may sue the buyer on the underlying debt.
(C) The bank may foreclose on the land, or it
may sue the third party on the underlying
debt.
(D) The bank may foreclose on the land, or it
may elect to sue either the buyer or the third
party on the underlying debt.

A

(B) The bank may foreclose or sue the buyer on the debt.

In the majority of jurisdictions, when a
mortgagor transfers title to another, and the transferee takes “subject to” the mortgage, that means that the transferee will not !!!!!!! be liable to the mortgagee on the promise underlying the mortgage.
So the third party cannot be sued on the debt.

But the mortgage follows the property and if the transferee does not make the payments, the mortgagee may foreclose on the mortgage.

In any event, the mortgagor, the buyer, remains liable on the mortgage after the transfer and she can be sued on the debt.

If the facts showed that the third party had “assumed” the mortgage, rather than merely taking “subject to” it, then he could have been sued on the debt.

SUBJECT = STILL SCREWED

(A) is incorrect. The buyer signed the mortgage and promised to pay the debt. She will always continue to be liable for her promise regardless of whether the third party is liable. And, because the mortgage follows the land, the mortgagee may always foreclose.

(C) is incorrect. The third party did not “assume” the mortgage. He can thus not be held liable on the underlying debt, although if he does not make the payments, the mortgagee may foreclose, since the mortgage always follows the land.

(D) is incorrect. Only the buyer is liable on the underlying debt since the third party only took “subject to” the mortgage.
Because the mortgage follows the land, the mortgagee may always foreclose if the mortgage is not paid.

121
Q

A buyer entered into a contract with a seller
to buy a parcel of land for $40,000. Although
the buyer was expecting to receive a large
inheritance in a few weeks, he had very limited
funds on hand and was able to personally
finance only $10,000. To cover the remaining
balance, the buyer obtained a loan from the
seller for $30,000, giving the seller a nonnegotiable
promissory note in that amount secured
by a mortgage on the land and orally promising
to pay the seller in full when he received his
inheritance money. A few weeks later, the seller
transferred possession of the mortgage note to
an investor for $25,000 without informing the
buyer. The next day, the seller received a check
from the buyer in the amount of $30,000. A few
days later, the seller left the country with the
$65,000 she had made on the sale of the land.
Which of the following correctly states the
investor’s rights against the buyer?
(A) The investor has no enforceable interest
in the land and no rights against the buyer
because the seller did not transfer the mortgage
to him and the buyer paid the mortgage
amount in full.
(B) The investor has no enforceable interest
in the land and no rights against the buyer
because the seller did not inform the buyer
of the note’s transfer.
(C) The investor has an enforceable interest in
the land to the extent of $25,000.
(D) The investor has an enforceable interest in
the land to the extent of $30,000.

A

(B) The investor has no enforceable interest in the land and no rights against the buyer because the seller did not inform the buyer of the note’s transfer.

If the original mortgagee transfers possession
of a nonnegotiable note without giving notice of the transfer to the mortgagor, the mortgagor’s
payment to the original mortgagee is effective even though it is not made to the person entitled
to enforce the instrument.
[Restatement (Third) of Property: Mortgages §5.5]

Here, the promissory note given to the seller by the buyer was properly transferred to the investor. But because the seller did not inform the buyer of the transfer, the buyer’s $30,000 payment to the seller effectively discharged the mortgage. Thus, (C) and (D) are incorrect. (A) is incorrect because when a note is properly transferred, a mortgage will automatically follow it as a general rule; no special written assignment of the mortgage is necessary.

122
Q

A buyer purchased a parcel of land from a
seller for $500,000. The buyer financed the
purchase by obtaining a loan from the seller for
$300,000 in exchange for a mortgage on the
land. The seller promptly and properly recorded
his mortgage. Shortly thereafter, the buyer gave
a mortgage on the land to a creditor to satisfy
a preexisting debt of $100,000 owed to the
creditor. The creditor also promptly and properly
recorded its mortgage. Within a year, the buyer
stopped making payments on both mortgages,
and the seller brought an action to foreclose on
his mortgage. The creditor was not included
as a party to the foreclosure action. The seller
purchased the property
at a public foreclosure
sale in satisfaction of the loan. The creditor
subsequently discovered the sale and informed
the seller that it was not valid.
Who has title to the land?
(A) The seller, because he gave a purchase
money mortgage and the creditor’s mortgage
was for a preexisting debt.
(C) The seller, but he must redeem the creditor’s
mortgage to avoid foreclosure.
(D) The buyer, because the seller’s foreclosure
action was invalid without the inclusion of
the creditor as a necessary party.

A

(C) The seller has title to the land, but he must redeem the creditor’s mortgage to avoid foreclosure.
As a general rule, the priority of a mortgage is determined by the time it was placed on the
property. When a mortgage is foreclosed, the purchaser at the sale will take title as it existed when the mortgage was placed on the property. Thus, foreclosure will terminate interests junior to the mortgage being foreclosed but will not affect senior interests. However, if a lien senior to that of a mortgagee is in default, the junior mortgagee has the right to pay it off (i.e., redeem it) to avoid being wiped out by its foreclosure. Thus, those persons with interests subordinate to those of the foreclosing party are necessary parties to the foreclosure action. Failure to include a necessary party results in the preservation of that party’s interest despite foreclosure and sale.

Hence, the seller’s failure to include the creditor as a party to the foreclosure action preserved the creditor’s mortgage on the property. To avoid the creditor’s foreclosing (because the buyer was in default of the creditor’s mortgage as well), the seller will need to pay off the creditor’s mortgage. (A)
is wrong because it is irrelevant. While a purchase money mortgage (“PMM”), given when the mortgagor buys the property, is considered to have priority over non-PMM mortgages executed at about the same time, even if the other mortgages are recorded first, that rule is not applicable here because the facts indicate that the seller’s PMM was executed and recorded before the creditor’s mortgage came into existence.

(D) is wrong because the failure to include the creditor in the foreclosure action does
not invalidate the action,
it just preserves the creditor’s junior mortgage on the property.

123
Q

To secure a loan of $100,000 from a bank, the
owner in fee simple of a parcel of land conveyed
a deed of trust for the land to the bank. The
deed of trust contained a “power of sale” clause,
permitted by the jurisdiction, which allowed the
bank to sell the property in the event of default
without the necessity of a judicial foreclosure
action. After several years, the owner defaulted
on his loan payments to the bank. The bank
informed the owner that it was exercising its
power of sale. After appropriate notices, the
bank conducted a public sale of the land. The
bank was the sole bidder and obtained the
property for $80,000, which was $10,000 less
than the outstanding balance on the loan plus the
expenses of the sale. One month later, the owner
notified the bank that he wanted to pay off the
loan and extinguish the deed of trust, and was
prepared to tender $80,000 to do so. The bank
insisted that the owner must tender $90,000 to
pay off the loan.If a court in the jurisdiction will require the bank to accept only $80,000 under the circumstances above, what is the likely reason?
(A) The owner had the power to revoke the
trust as long as he was alive.
(B) The bank did not have the authority to bid
on the property at other than a judicial
foreclosure sale.
(C) The owner was exercising a statutory power
rather than an equitable power.
(D) The bank does not have the power to clog
the equity of redemption.

A

(C) If the owner can compel the bank to accept his offer, it will be because he has a statutory power to redeem the property after the foreclosure sale has occurred.

_*In all states*, the equity of redemption provides the borrower with an equitable right, at any time prior to the foreclosure sale, to redeem the land or free it of the mortgage or lien_ by paying off the amount due or, if an acceleration clause applies, the full balance due.

Only about half the states, however, give the borrower a statutory right to redeem for some fixed period after the foreclosure sale has occurred; the amount to be paid is generally the foreclosure sale price, rather than the amount of the original debt.

Thus, if the owner can redeem the land for $80,000, it will be based on the jurisdiction’s statutory power of redemption.

(A) is wrong because the deed of trust is a security interest (similar to a mortgage)
to which the revocation rules for trusts do not apply. The deed of trust was created in part to allow the lender to foreclose on the property without going through a judicial foreclosure proceeding.
(B) is wrong because, in states that permit a nonjudicial sale with deeds of trust containing a
power of sale, the lender may bid at the sale, and in many cases the lender is the sole bidder.

(D) is wrong because the prohibition against “clogging the equity of redemption” refers to the rule that a borrower’s right to redeem his own mortgage cannot be waived in the instrument itself.

Here, there is nothing to indicate that the owner’s deed of trust prohibited him from redeeming the
property prior to foreclosure. However, it is only through a statutory right of redemption that the
owner would be able to redeem the property for $80,000 after the foreclosure sale had occurred

124
Q

Zoning regulations and restrictive covenants in private deeds are completely separate concepts. Both must be complied with, and neither provides any excuse for violating the other.

A

Thus, a variance from the government regulation (i.e. zoning commision may grant variance froma zoning regulation) does not prevent enforcement of the private covenant.

125
Q

A landowner divided her large tract of land
into two adjoining parcels of land, a ranch and
a farm, and sold the ranch to her brother. The
brother promptly and properly recorded the
deed. Because the ranch had no direct access to
a public road, the landowner wrote into the deed,
“my brother, his heirs and assigns shall have
the right to use the existing dirt path along the
eastern border of the farm for ingress and egress
to the ranch.” A few years later, the landowner
sold the farm to a developer, who decided to
subdivide the farm into lots for single-family
residences. Because no street in the proposed
subdivision would align with the dirt path
mentioned in the deed from the landowner to
the brother, the brother would be without ingress
and/or egress to his land. The brother instituted
an appropriate action to enjoin the blocking of
the dirt path.
Which party is most likely to prevail?
(A) The brother, because the owner of the servient
tenement cannot obstruct an express
easement.
(B) The brother, because he has a way by
necessity.

A

(A) Judgment will be for the brother because the owner of the servient tenement cannot obstruct an express easement.

The language in the deed from the landowner to the brother creates an express easement with the landowner’s farm as the servient estate and the brother’s ranch as the dominant estate. If, as here, the servient parcel is transferred, its new owner takes it subject to the burden of the easement, unless they are a bona fide purchaser with no notice of the easement.

Here, the deed containing the express easement was recorded and was in the developer’s chain of title. Thus, the developer had record notice of the easement. As the holder of the servient parcel, the developer has no right to obstruct the brother’s use of the easement.

(B) is incorrect because the brother
has an express easement, not an easement by necessity. While it is true that, in the absence of an express easement, the brother may have had a claim of an easement by necessity, an easement by necessity will not be implied when an express easement is provided.

126
Q

A developer purchased a 25-acre tract of
land abutting an industrial park. The property
was rezoned for mixed residential and commercial use and subdivided into 50 half-acre lots. The developer sold the 30 lots furthest from
the industrial park to buyers using deeds that
restricted the lots to residential use. Only 15
of those lots were improved with single-family
homes. Over the next 10 years, the owners of the
other 15 lots constructed commercial buildings
on their land. Five years later, the developer sold
the remaining 20 lots to an investor who planned
to erect a shopping mall on the land. An original
owner of one of the lots with a single-family
home objects to the investor’s proposed use.
If the investor is permitted to build the shopping mall, what is the likely reason?
(A) The neighborhood has significantly
changed.
(B) The servitude has been abandoned.

A

(B) If the investor is permitted to build the shopping mall, it will be because the servitude has been abandoned.

If a benefited party acquiesces in a violation of the servitude by one burdened party, he may be deemed to have abandoned the servitude as to other burdened parties.

Here, there is no evidence that the original owner objected to the 15 lots being developed as commercial properties. Thus, she cannot enforce an implied servitude against the investor.

(A) is incorrect because the doctrine of changed neighborhood conditions is inapplicable here. Under the doctrine, if the neighborhood has changed significantly since the time the servitude was created, with the result that it would be inequitable to enforce the restriction, injunctive relief will be withheld.

Here, the industrial park existed prior to the subdivision. The only change has occurred within the subdivision itself, but it will be permitted because the servitude has been abandoned by acquiescence.

127
Q

The owner of a hotel in a resort town was
approached by a seminar speaker who wanted
to lease space in which to conduct a two-week
seminar. The owner leased to the speaker the
hotel’s grand ballroom, the period of the lease
being August 1 through August 14. To provide
the proper atmosphere for the seminars, the
speaker attached curtain rods to the walls of the
ballroom, using lightweight screws to attach the
rods. The speaker then strung light blue ring
curtains through the rods. After the seminar, on
August 16, the speaker arrived to remove the
curtains and rods. The owner brought an action
to enjoin the speaker from removing the curtains
and the rods from the grand ballroom.
How should the court rule?

(A) In favor of the speaker, because he had a
short-term lease and the curtains and rods
were easily removable.

(D) In favor of the owner, because the speaker
did not remove the curtains and rods before
the lease expired.

A

(D) The court should rule in the owner’s favor.

A tenant must remove annexed chattels before the termination of the tenancy or they become the property of the landlord. Although the seminar speaker was probably entitled to remove the curtains and rods at the end of the lease, he forfeited them by waiting for two days after the lease expired to remove them.

(A) is wrong because it goes to whether the curtains and rods were intended to be fixtures.

Because of the delay in their removal, whether the curtains or rods were fixtures is irrelevant. This choice would be correct, however, had the speaker attempted to remove the curtains on August 14. The short-term lease and the fact that the rods are easily removable constitute evidence that the speaker lacked the requisite intent to permanently improve the property, and thus he could have removed them if he had acted promptly.

128
Q

A landowner and his neighbor purchased
adjoining undeveloped lots. After both built
homes on their respective lots, the landowner
suggested to the neighbor that a common
driveway be built where the two lots joined.
The neighbor agreed. The landowner and the
neighbor split the cost of constructing the
driveway and entered into a written agreement
to equally share the costs of its upkeep and
maintenance. The agreement was recorded in the county recorder’s office. Two years later, the neighbor built a new driveway located entirely on his lot. The common driveway, which the landowner continued to use but which the neighbor no longer used, began to deteriorate. The landowner asked the neighbor for money to maintain the common driveway, but the neighbor refused to contribute. Three years later, the neighbor conveyed his lot to a friend. The friend entered into possession and used only the driveway built by the neighbor. By this time, the common driveway had deteriorated badly and contained numerous potholes. The landowner asked the friend to pay half of what it would take to repair the common driveway. The friend refused. The landowner repaired the driveway and sued the friend for 50% of the cost of repairs.
Will the landowner prevail?

(B) Yes, because the agreement between the
landowner and the neighbor was recorded.
(C) No, because the neighbor abandoned use of
the easement.
(D) No, because the landowner is not in privity
of contract with the friend.

A

(B) The landowner will prevail because recording the agreement gave the friend constructive notice, thus preventing her from claiming the protection of the recording act as a defense to enforcement of the covenant.

A covenant at law will run with the land and be enforceable against subsequent grantees if:

(i) the contracting parties intended it to run;
(ii) there is privity of estate between the original promisor and promisee (horizontal privity), as well as between the promisor and his successor (vertical privity);
(iii) the covenant touches and concerns the property; and
(iv) the burdened party has notice of the covenant.

If common driveway owners agree to be mutually
responsible for maintaining the driveway, the burdens and benefits of these covenants will run to successive owners of each parcel. The implied cross-easements for support satisfy the horizontal privity requirement because they are mutual interests in the same property. Each promise touches and concerns the adjoining parcel. So here, where the friend is in vertical privity with the neighbor (holding the same interest he held) and has constructive notice, she will be bound by the agreement to maintain the driveway.

(C) is wrong for the same reason.
Whether the easement has been abandoned does not affect the enforceability of the separate
covenant.

(D) is wrong because privity of estate, not privity of contract, is required for the burden
of the covenant to run.

129
Q

A landowner and her neighbor owned
adjoining parcels of land. The landowner’s
property was situated to the west of the neighbor’s property. A highway ran along the east of the neighbor’s property. Twelve years ago, the landowner asked the neighbor if it would be all right for the landowner to use an eight-foot
strip along the northern part of the neighbor’s
land to access the highway. The only other way
for the landowner to get to the highway was to
use a one-lane unpaved road that meandered
through the woods for two miles. The neighbor
agreed, and the landowner used the strip of land
regularly to access the highway. The statutory
period for adverse possession in this jurisdiction
is 10 years.
What is the landowner’s interest in the neighbor’s
eight-foot strip of land?

(A) An easement appurtenant.

(D) Not an easement.

A

(D) The landowner’s interest in the neighbor’s eight-foot strip of land is not an easement. In effect, the landowner only has a “license” (i.e., a revocable privilege) to use the land. The answer is best reached by the process of elimination. Because an easement is an interest in land, the Statute of Frauds applies. Here, the agreement between the landowner and the neighbor was not in writing; thus, the Statute of Frauds requirements for the creation of an express easement were not met. Therefore, (A) is incorrect.

130
Q

For an easement to be extinguished by estoppel,
there must be:

A

(i) some conduct or assertion by the owner of the easement,
(ii) a reasonable reliance by the owner of the servient tenement, coupled with
(iii) a change of position.
i. e.

If the neighbor tells the landowner that she will no longer be using the driveway, and the
landowner thereafter builds a garage over the driveway, the easement has been terminated by
estoppel.

131
Q

Under the concept of fixtures, a chattel that has been annexed to real property is converted
from personalty to realty. As an accessory to the land, it passes with ownership of the land rather
than with a transfer of the personal property of an estate. The manifest intent of the annexor
determines whether the chattel becomes a fixture. The factors for evaluating the annexor’s intent are:

A

(i) the relationship between the annexor and the premises,

  • *(ii) the degree of annexation, and
    (iii) the nature and use of the chattel.**

i.e. organ is a fixture: (i) the philanthropist was the fee owner of the mansion and had the organ built to his specifications when the mansion was constructed; (ii) the organ was built into the wall of the mansion and could not be easily removed; and (iii) the appearance of the organ and how it complemented the rest of the mansion probably were more important to the philanthropist than its function.

Constructive annexation occurs when an article of personal property (an “accession”) becomes an integral part of the property, even though it is not physically annexed to the property, in the same sense that a fixture becomes an integral part of the realty.

i.e. The organ bench is an accession because it was created as an integral part of the organ and significantly contributes to an important aspect of the organ: its overall appearance. Removing the bench and replacing it with a bench made of different wood or carvings would damage the aesthetic value of the organ.

132
Q

A landowner conveyed a parcel of land “to
my brother, his heirs and assigns, but if my
brother dies and is not survived by children by
his present wife, then to my cousin and his heirs
and assigns.” Shortly after taking possession,
the brother discovered rich metal deposits on the
land, opened a mining operation, and removed
and sold a considerable quantity of valuable
ore without giving the cousin any notice of his
action. The brother has no children. The brother,
his wife, and the cousin are all still living.
The cousin brought an action in equity for an
accounting of the value of the ore removed and
for an injunction against further removal.
If the decision is for the brother, what is the
likely reason?
(A) The cousin has no interest in the land.
(B) The right to take minerals is an incident of
a defeasible fee simple.

A

(B) If the decision is for the brother, it will be because the right to take minerals is an incident of a fee simple interest.

The owner of a defeasible fee has the same right to possession and privileges of use as the owner of a fee simple absolute. Only unconscionable conduct by the owner that substantially reduces the value of the land could possibly be enjoined by the person who would take the land should the current owner’s estate terminate.

(A) is incorrect because the cousin has an executory interest.

133
Q

A farmer owned land in fee simple. He
executed two deeds, the first conveying an
undivided one-half interest in the land to a
husband and a wife as joint tenants with right
of survivorship, and the second conveying an
undivided one-half interest in the land to the
husband’s only child. The child was 13 years old
at the time. The common law joint tenancy is
unmodified by statute. The farmer handed the
two deeds to the husband. The husband promptly and properly recorded the deed to himself and his wife and put the deed to his child in a safedeposit box without recording it. No actual consideration was paid for the deeds. The same year, the husband, the wife, and the child were killed simultaneously in an airplane crash. They all died intestate. The applicable statute in the jurisdiction provides that “when title to property or its devolution depends on priority of death and there is insufficient evidence that the persons have died otherwise than simultaneously, the property of each person shall be disposed of as if he had survived.” An appropriate action was instituted by the heirs of the husband, the wife, and the child. The farmer, who is not an heir of any of the deceased, is a party to the action. In whom should the court determine that title to the land is vested?

(C) One-half in the farmer, one-quarter in the
heirs of the husband, and one-quarter in the
heirs of the wife.
(D) One-half in the heirs of the child,
one-quarter in the heirs of the husband, and
one-quarter in the heirs of the wife.

A

(D) The court should determine that title to the land is one-half in the heirs of the child, one-quarter in the heirs of the husband, and one-quarter in the heirs of the wife.

The husband and the wife held the one-half interest in the land as joint tenants with right of survivorship. Thus, had one of them survived, he or she would own the entire one-half interest. The operation of the simultaneous death statute in the jurisdiction, which disposes of property as if each survived, results in their property being distributed as though they were tenants in common; i.e., one-half of their interest passes through the husband’s estate as though he survived and one-half of their interest passes through the wife’s estate as though she survived.

The child always held her one-half interest as a tenant in common, so her one-half interest clearly passes to her heirs without any need to resort to the simultaneous death statute.

(C) is wrong because it implies that the
one-half interest was not effectively conveyed to the child. The delivery of the deed to the child’s
parent during the time that she was a minor is effective delivery. The fact that the deed was not recorded has no effect on its effectiveness.

134
Q

A brother and a sister inherited their childhood
home from their father, taking title as
tenants in common. The brother moved into
the home, while the sister returned to her
residence in a distant city. Although there was
no discussion between the brother and the sister
concerning their common ownership, the brother
paid all taxes, insurance, and other carrying
charges on the home. He paid no rent or other
compensation to the sister, nor did the sister
request any such payment. Thirty years later,
a series of disputes arose between the brother
and sister concerning their respective rights to
the home. The jurisdiction where the land is
located recognizes the usual common law types
of co-tenancies, and has an ordinary 20-year
adverse possession statute.
If the brother brings an action against the
sister to quiet title to the home in himself, who
will prevail?

(C) The sister, because there is no evidence that
the brother has performed sufficient acts to
constitute her ouster.
(D) The sister, because one co-tenant cannot
acquire title by adverse possession against
another.

A

(C) The sister will prevail in a quiet title action brought by the brother because there is no evidence that he has performed sufficient acts to constitute her ouster.

Actual possession of property held in concurrent ownership by one concurrent owner for the statutory adverse possession period will not be sufficient to give that possessor title to the whole estate to the exclusion of his co-tenant,
unless there has been an ouster.

Here, the brother occupied the home under unity of possession with the sister. Thus, his possession is not adverse to that of the sister.

(D) is incorrect because, had the brother ousted the sister prior to possessing the land for the required period, he could claim title to the home because his possession would have been
adverse.

135
Q

A homebrewing supplier entered into a written
five-year lease to rent a unit in a strip mall from
a landlord. The rent was payable in monthly
installments on the first of each month. Two
months before the end of the five-year term,
the landlord sent a letter to all of her tenants
providing that those tenants wishing to renew
their leases could do so at a rent increase of
10%. The homebrewing supplier did not respond
to the letter, but continued to send checks for
the original amount of rent to the landlord. Each
check was marked “new rent payment,” and the
landlord deposited every one. Six months later,
the landlord notified the homebrewing supplier
that he was four months’ in arrears in his rent.
Is the homebrewing supplier liable for the
10% difference in rent over the last four months?
(A) Yes, because he has a month-to-month tenancy
for the increased rent amount.
(B) Yes, because he has a year-to-year tenancy
for the increased rent amount.

A

(B) The homebrewing supplier is liable for the 10% rent difference over the last four months because he has a year-to-year tenancy for the increased rent amount.

When a tenant continues in possession after the termination of his right to possession, the landlord has two choices of action:

She may treat the hold-over tenant as a trespasser and evict him under an unlawful detainer statute,
or she may, in her sole discretion, bind the tenant to a new periodic tenancy.

_*In a commercial lease*, if the original lease term was for a year or more, a year-to-year tenancy results._

If the landlord notifies the tenant before termination of the tenancy that occupancy after termination will be at an increased rent, the tenant will be held to have acquiesced to the new terms if he does not surrender. The tenant will be held to the new terms even if he objects to the increased rent, provided that the rent increase is reasonable.

Here, the original lease was a commercial lease for a five-year term, so the landlord’s decision to hold the homebrewing supplier to a new tenancy
makes it a year-to-year tenancy. Because the landlord notified the homebrewing supplier of the
10% rent increase prior to the termination of the original five-year term, the new tenancy is at
the increased amount of rent. Note that the landlord’s depositing of the homebrewing supplier’s checks would not create an accord and satisfaction even though they were marked “new rent payment.” In addition to the requirement that a bona fide dispute must exist as to the amount
owed, the check must conspicuously state that it is in full satisfaction of the debt before an accord and satisfaction can occur.

(A) is incorrect because the new tenancy is year-to-year, not month-to-month. Most courts would rule a residential hold-over tenant a month-to-month tenant, irrespective of the term of the original lease, but the homebrewing supplier is a commercial tenant.

136
Q

An investor financed the purchase of a parcel
of land with a $500,000 loan from a bank,
secured by a mortgage on the land. After an
unsuccessful search to find a developer to subdivide
and build homes on the land, the investor
leased the existing home to a tenant for five
years. During the fourth year of the lease, the
investor defaulted on the bank loan. The bank
brought a foreclosure action, joining the investor
and the tenant in the proceeding. The land was
sold to a buyer at the foreclosure sale for the
outstanding amount of the loan. After the buyer
obtained a judgment in an ejectment action
against the tenant, the tenant brought an appropriate
action for damages against the investor.
As between the tenant and the investor, for
whom should the court rule?

(B) The tenant, because the investor breached
the covenant of quiet enjoyment.
(C) The investor, because a landlord may assign
the rents and reversion interest that she
owns to a third party.

A

(B) The court should rule for the tenant because the investor breached the covenant of quiet enjoyment.
_There is implied in every lease a covenant that neither the landlord nor someone with
paramount title will interfere with the tenant’s quiet enjoyment and possession of the premises.
_
The covenant of quiet enjoyment may be breached by actual eviction—where the landlord
or paramount title holder excludes the tenant from the entire leased premises. Having been
deprived of possession of the entire premises, the tenant may treat the lease as terminated, and
his liability for further rent under the lease is discharged. The tenant may also collect damages from the landlord for breach of covenant.

Here, because foreclosure destroys all interests junior to the mortgage being foreclosed, the tenant’s lease was wiped out by the bank’s foreclosure.
The buyer obtained paramount title at the foreclosure sale and properly evicted the tenant from the premises. Thus, the investor is in breach of the covenant of quiet enjoyment and the tenant can recover damages from her.

(C) is incorrect because the investor did not assign the rents and her reversion interest to the buyer. Although an assignee is liable to the tenants for performance of all covenants made by the original landlord, _nothing in the facts
indicates the buyer purchased the land subject to the lease.
_
Moreover, the fact that the buyer
obtained an ejectment order from the court shows that she did not take title subject to the lease.

137
Q

A landlord entered into a written lease of
a bakery for a term of 25 years with a baker.
The parties agreed to a right of first refusal if
the bakery was offered for sale during the term
of the lease. The lease also permitted assignments and subleases on notice to the landlord.
Three years later, the baker retired and, after
notifying the landlord, transferred the lease to a
chocolatier. Twenty-one years later, the landlord
entered into a contract with a buyer for the
sale of the bakery for $100,000. The landlord
had informed the buyer of the lease but had
forgotten about the right of first refusal. When
the chocolatier learned of the sale to the buyer,
she informed both the landlord and the buyer
that she wanted to exercise her option and was
prepared to purchase the bakery for the contract
price. The jurisdiction’s Rule Against Perpetuities
is unmodified by statute.
Can the chocolatier enforce the option?
(A) Yes, because an option held by a tenant on
leased property cannot be separated from
the leasehold interest.
(B) Yes, because the option touches and
concerns the leasehold estate.
(C) No, because the transfer to the chocolatier
made the option void under the Rule
Against Perpetuities.

A

(B) The chocolatier can enforce the option to purchase because it is a covenant that runs with the land.
When a tenant makes a complete transfer of the entire remaining term of his leasehold interest, it
constitutes an assignment. The assignee and the landlord are then in privity of estate, and each is
liable to the other on all covenants in the lease that run with the land. The covenant runs with the
land if the original parties so intend and the covenant “touches and concerns” the leased land, i.e., burdens the landlord and benefits the tenant with respect to their interests in the property.

Here, the transfer of the lease to the chocolatier was an assignment, making all covenants in the lease that run with the land enforceable by the assignee. The right of first refusal burdens the landlord’s power of alienation over the bakery, and there is nothing to indicate that the parties intended the option to be personal to the baker. Hence, the chocolatier can enforce the option and purchase the property.

(A) is incorrect because most courts do not bar an option from being separated from the leasehold interest if that is the parties’ intent. The tenant may transfer the leasehold interest
while retaining the option to purchase, or vice versa.
Whether the option in this case stayed with the leasehold interest depends on whether it was a covenant that runs with the land.

(C) is incorrect because options and rights of first refusal are not subject to the Rule Against Perpetuities when connected to leaseholds. If a tenant assigns the leasehold, the option generally is considered a running covenant, exercisable by the assignee absent a contrary intent. If the option had been separated from the leasehold estate, so that it was no longer exercisable by the tenant, the Rule would have become applicable to the option (and it would have invalidated the option here because it could have been exercised more than 21 years after a life in being).

Here, however, the option was not severed from the leasehold; the entire interest was transferred to the chocolatier as the new tenant. Hence, the Rule Against Perpetuities is not applicable.

138
Q

Nonassignment clauses in leases are valid and enforceable even though strictly construed, and allow the landlord in most states to refuse to consent to an assignment even if the refusal is unreasonable.

If a tenant makes an assignment in violation of a nonassignment clause in the lease, the transfer is ….

A

…. not void.

However, the landlord may terminate the lease if specifically provided by the nonassignment clause. Alternatively, he may sue for damages if he can prove any.

139
Q

A grantor executed and delivered a deed to
his son conveying his land as follows: “To my
son for life, but if my son dies survived by his
spouse and children, then to my son’s spouse for
life, with the remainder in fee simple to my son’s
children.”
A year later, the son died survived by his
spouse and two offspring, a girl and a boy. The
boy died intestate two days after the son, leaving
one child as his only heir.
The common law Rule Against Perpetuities is
unmodified in the jurisdiction.
What are the respective interests of the
spouse, the girl, and the child in the land?

(A) The spouse has a life estate, the girl has an
absolutely vested remainder, and the child
has nothing.
(C) The spouse has a life estate, and the
girl and the child have absolutely vested
remainders.

A

(C) The spouse has a life estate, the girl has an absolutely vested remainder, and the child, by intestate succession, will inherit the boy’s absolutely vested remainder.

The remainder to the son’s children **was vested subject to open** upon the birth of his first child. **Because the son cannot have any more children after his death, all members of the class are ascertained at that time and the remainder
becomes _indefeasibly vested._**

Because the grant was to the son’s “children” rather than “issue” or “descendants,” there is no unborn child problem.

(A) is wrong because the boy’s vested remainder is inheritable by the child.

140
Q

A landowner devised her land by will to her
husband for life, then “to my grandchildren
in fee simple.” When the landowner died, she
had one granddaughter. Three months later, a
grandson was born. The landowner’s husband
died shortly after that. Six months later, a second
grandson was born. A year after that, a second
granddaughter was born.
At this point, the first granddaughter owns
the land in fee simple along with:
(A) No one else.

(B) is incorrect because, as discussed above, persons in gestation at the time set for distribution are included in the class.
(C) The first and second grandsons.

A
(C) The first granddaughter owns the land along with the first and second grandsons *_**because the
class closed on the death of the landowner’s husband.**_*

When a gift is made to a group of persons generically described as a class, such as to someone’s “grandchildren,” the rule of convenience provides that the class closes when some member or members of the class can call for a distribution of their share of the class gift. Persons born after that date are excluded from the class.

Here, the class of the landowner’s grandchildren entitled to share in the fee simple estate closed when they became entitled to take, which was at the termination of the preceding estate (the life estate of the landowner’s husband). The second grandson qualifies as a class member because persons in gestation at the time set for distribution are included in a class. Hence, both grandsons share in the estate with the first granddaughter.

(A) is incorrect because the class does not close when the will takes effect. When the landowner died, the granddaughter received a vested remainder subject to open, but the class did not close because the life estate in the landowner’s husband precluded distribution at that point.

(B) is incorrect because, as discussed above, persons in gestation at the time set for distribution are included in the class.

141
Q

In compliance with a federal statute requiring
buildings to be made accessible to persons with
disabilities, a lawyer installed wheelchair ramps
at both entrances to his office building located
on a parcel of land that he had owned for many
years. One year later, the lawyer entered into a
contract with an accountant to sell the parcel of
land, including the office building. After having
the property surveyed, the accountant notified
the lawyer that she was not going to complete the sale because the wheelchair ramp on the south side of the building extended over the property line and into the adjoining tract of land, making the title unmarketable. The lawyer insisted that the accountant proceed with the sale, and brought an action to compel her performance.
If the court were to find that title is marketable,
what is the most likely reason?

(B) The lawyer currently owns the adjoining
tract of land and acquired it and the office
building land as part of a larger parcel.
(C) The wheelchair ramp extends only 10
inches over the property line.

A

(B) The court could find the title marketable by finding that an implied easement for the benefit of the office building land was created from the existing use when that parcel of land was severed from the adjoining tract of land by the sale. The requirement that the seller provide the buyer with marketable title means that the title must be free from questions that present an unreasonable risk of litigation. A significant encroachment constitutes a title defect, regardless of whether an adjacent landowner is encroaching on the seller’s land or vice versa. However, under the circumstances in (B), a court will be able to avoid the encroachment problem by implying an easement from the existing use (a quasi-easement). If a use exists on the “servient” part of the tract that is reasonably necessary for the enjoyment of the “dominant” part, and a court determines that the parties intended the use to continue after division of the property, an easement will be implied.
Given that title will not be marketable otherwise, the court will deem that the parties intended
for the use to continue.
(C) is incorrect because an encroachment of 10 inches is significant enough to make title unmarketable. Whether an encroachment is significant enough to make title unmarketable is ultimately a question of fact; however, structures that encroach more than a few inches over the property line are generally found by the courts to be significant encroachments.

142
Q

A 10-lot subdivision was approved by the
proper governmental authority. The authority’s
action was pursuant to a map filed by the
developer, which included an undesignated
parcel in addition to the 10 numbered lots. The
shape of the undesignated parcel is different and
somewhat larger than any one of the numbered
lots. Subdivision building restrictions were
imposed on “all the lots shown on said map.”
The developer contracts to sell the undesignated
parcel, described by metes and bounds, to an
investor. Is title to the parcel marketable?
(A) Yes, because the undesignated parcel is
not a lot to which the subdivision building
restrictions apply.
(D) No, because the map leaves it uncertain
whether the undesignated parcel is subject
to the building restrictions.

A

(D) Title to the undesignated parcel is unmarketable because the map leaves it uncertain whether the parcel is subject to the building restrictions.

Marketable title is one that is free from reasonable doubt in fact or in law.

Here, there is confusion because the building restrictions apply to all the lots shown on the map, but the parcel at issue is not one of the 10 numbered lots. Thus, it is unclear whether the parcel is subject to restrictions that will reduce the uses of the lot or its market value.

(A) is wrong because it is not clear that the undesignated parcel is not subject to the subdivision restrictions. It was included on the map and the restrictions apply to “all lots shown.”

143
Q

A foreign correspondent wished to purchase
a parcel of land from a developer that was not
yet on the market. Before he left the country,
he gave his attorney $100,000 and his power of
attorney. He instructed the attorney that, should
the land be put up for sale, she was authorized
to: offer up to $100,000 for it, enter into a binding contract to purchase it on the correspondent’s behalf, and, if he did not return in time, close on the property. In early January, the developer put the land on the market. The attorney offered $75,000 for it, which the developer readily accepted. On January 15, the attorney, on
the correspondent’s behalf, entered into a written
contract to purchase the land for $75,000. Closing was set for February 15. During this
time, the attorney heard nothing from the correspondent.
When he had not returned by the date
of closing, the attorney attended the closing and
tendered the $75,000. The developer tendered
a deed made out to the correspondent as the
grantee. On February 20, news was received
that the correspondent had been killed by a stray
bullet on January 14. The correspondent’s will
left his entire estate to his niece. The developer
believes the conveyance to the correspondent is
invalid, and brings a suit to quiet title to the land.
Who is the court most likely to find is the
owner of the land?
(A) The niece, because the attorney held the
deed on constructive trust for the correspondent’s estate.
(C) The developer, because a deed to a nonexistent person is void and conveys no title.

A

(C) The developer owns the land because a deed to a nonexistent person is void and conveys no title.
Because the correspondent was dead when the deed was delivered, the deed passed nothing and was a nullity. Note that the developer will be required to return the $75,000 to the correspondent’s estate to avoid unjust enrichment.

(A) is wrong because title never passed from the developer. Furthermore, even if it had, it would not have passed even bare legal title to the attorney because she was not a grantee and the developer did not intend to pass title to her.

144
Q

An investor entered into a written agreement
to purchase a parcel of land from a seller
for $50,000, with closing to take place on July
1. Two weeks before the date set for closing,
the investor entered into a written agreement
to sell the same parcel of land to a buyer for
$100,000, with closing to occur on August 1.
On July 1, the investor received a deed to the
land and informed the buyer that she would not
be proceeding with their deal because she had
received a better offer.
If the buyer sues for specific performance,
who is likely to prevail?

(B) The buyer, because the investor had marketable title at the time the buyer sought to
enforce the contract.
(C) The investor, because she had no title to
the land as of the date she entered into the
agreement with the buyer.

A

(B) The buyer will most likely prevail in a suit for specific performance because the investor had
marketable title at the time the buyer sought to enforce the contract.

There is an implied covenant in every land sale contract that at closing the seller will provide the buyer with marketable title; i.e., title that is free from questions that might present an unreasonable risk of litigation. An inability to establish a record chain of title will generally render the title unmarketable.

Here, although the investor did not have marketable title at the time she entered into the agreement with the buyer, the covenant only requires that the investor be able to deliver marketable title at the closing. When the buyer brought the action for specific performance, the investor held marketable title to the land. Thus, (C) is incorrect.

145
Q

A landowner executed a will, devising a parcel
of land “to my sister for life, then to my brother
for life, then to my nieces and nephews.” When
the landowner died, he was survived by the sister
and the brother’s son. The sister and the brother’s son contracted to sell the land to a buyer for $225,000. At the time set for closing, the sister and the brother’s son tendered a quitclaim deed to the buyer, who refused to complete the sale. The sister and the brother’s son bring suit against the buyer for specific performance. The jurisdiction in which the land is located does not follow the Doctrine of Worthier Title. Will specific performance likely be granted?

(A) Yes, because a quitclaim deed conveys
whatever interest the grantors have in the
property.
(D) No, because title is unmarketable.

A

(D) The sister and the brother’s son will not prevail in a suit against the buyer for specific performance because title is unmarketable.

Marketable title is title reasonably free from doubt, i.e., title that a reasonably prudent buyer would be willing to accept. Title may be unmarketable where the owners of the present and future interests attempt to convey a fee simple absolute title if the future interests are held by *persons who are unborn or unascertainable.*

Here, the sister has a life estate, and the brother’s son has a vested remainder subject to open because there may be other nieces
and nephews born during the sister’s life who become entitled to share in the remainder.

Life estates and vested remainders are freely transferable, which means that the sister and the brother’s son together can transfer the land to a purchaser, but the title is not marketable. It may turn out that the sister has a child, who is entitled to share in the remainder, but who did not join in the conveyance to the buyer. Because the child would not be bound by the conveyance, he would own an interest in the land. Thus, although the interests are considered alienable, the sister and the brother’s son cannot convey good title because there are outstanding interests in the unborn nieces and nephews.

(A) is incorrect because, although a quitclaim deed does convey whatever interest
the grantors have in the property, it does not affect the covenant to provide marketable title.

146
Q

A buyer entered into a written contract with
a seller to purchase his commercial property
for $100,000. The contract did not specify
the quality of title to be conveyed, and made
no mention of easements or reservations. The
closing was set for November 25, three months
from the signing of the contract. Shortly
thereafter, the buyer obtained a survey of the
property, which revealed that the city had an
easement for the public sidewalk that ran in front
of the store. Because this actually enhanced the
value of the property, the buyer did not mention
it to the seller.
Subsequently, the buyer found a better location
for her business. On November 1, the buyer
notified the seller that she no longer intended to
purchase the property. The seller told her that he
intended to hold her to her contract. At closing,
the buyer refused to tender the purchase price,
claiming that the seller’s title is unmarketable
and citing the sidewalk easement as proof of that
fact.
In a suit for specific performance, will the
seller likely prevail?

(B) Yes, because the buyer was aware of the
visible easement and it enhanced the value
of the property.
(C) No, because an easement not provided for
in the contract renders title unmarketable.

A

(B) The seller will prevail in his suit for specific performance because the easement was visible, the buyer was aware of it at the time she entered into the contract (i.e., she knew a public sidewalk ran in front of the store), and the easement enhanced the value of the property.

There is an implied covenant in every land sale contract that, at closing, the seller will provide the buyer with marketable title. Marketable title is title reasonably free from doubt, which generally means free from encumbrances and with good record title.

Easements are generally considered encumbrances that render title unmarketable; so if an easement is not provided for in the contract, it usually renders the seller’s title unmarketable. There is an exception, however. A majority of courts have held that a beneficial easement that was visible or known to the buyer does not constitute an encumbrance.

In this case, the sidewalk was visible, known to the buyer, and beneficial to the property.
Thus, the sidewalk easement does not impair the marketability of the seller’s title. Therefore, the
buyer’s excuse for her nonperformance is not valid, and because land is involved, the seller can
get specific performance of the contract for purchase of the property.

147
Q

An owner of a 240-acre tract of land entered
into an agreement to sell 20 acres of the land to
his neighbor. The land the neighbor wanted was
located in the northwest quarter of the land. The
owner wrote out the following statement:
The owner agrees to sell to the neighbor, for $50,000, 20 acres in the northwest quarter of the owner’s land, the owner’s land beginning at a
point in the northeast quadrant of the county on the 16th county survey line, six minutes west of the 98th Meridian, and then proceeding due east
1,320 yards and from that point due south to the creek, and then westward along the creek to the point at which it intersects the public road, and then northward along the eastern edge of the public road until the road ends, and then due northward from that point to the original starting point of the survey.
The owner had copied the language describing
the land from his own deed. The owner then
signed the paper and gave it to the neighbor. On
the date set for closing, the neighbor tendered
$50,000 to the owner. The owner refused to
accept the money and refused to convey 20 acres to the neighbor.
If the neighbor sues the owner for specific
performance, what is the likely result?
(A) The owner will prevail, because the writing
did not adequately describe which 20 acres
were to be sold.
(B) The owner will prevail, because the writing
was not signed by both parties.
(C) The neighbor will prevail, because metes
and bounds are a sufficient way to legally
describe property.

A

(A) The owner will prevail because the writing did not adequately describe the 20 acres to be sold.

For a court to grant specific performance, there must be a valid and enforceable contract. The
Statute of Frauds requires that the writing contain a description of the land to be sold sufficient
to identify it. If the description is too indefinite, the contract is not enforceable. In this case, the
description given is 20 acres from the northwest quarter of the owner’s land. The land consists
of 240 acres; therefore, the northwest quarter of the land contains 60 acres. Because there is no
guidance as to which 20 acres were intended by the parties, the contract is not enforceable and
the owner will prevail.

(B) is wrong because the writing need not be signed by both parties. To be enforceable under the Statute of Frauds, the writing need only be signed by the party to be charged. In this case, the owner is the party to be charged, and the writing was signed by him. Thus, the Statute of Frauds was satisfied in this regard.

_(C) is wrong because the metes and
bounds description here described the whole of the land, rather than the property to be conveyed.
_
It is true that metes and bounds is a sufficient way to describe property. In this instance, however, this method was used to describe the wrong property.

148
Q

A landowner owned a large piece of property
containing an inn and a bakery. She entered into
a contract to sell the property to a purchaser
for $1 million. The contract was recorded.
The purchaser gave the landowner $200,000
as earnest money. The closing date was set for
September 10, two months after the signing of
the contract.
On August 10, an arsonist set fire to the inn,
which burned to the ground. On September
10, the landowner appeared at the closing and
tendered the deed to the property. The buyer
refused to tender the remaining $800,000 of the
purchase price and demanded the return of his
earnest money. The landowner sued the buyer
for specific performance of the contract. The
buyer countersued for the return of his earnest
money. Both parties stipulate that the value of
the property without the inn is $600,000, that
insurance on the property had lapsed, and that
the common law, unmodified by statute, applies.
What is the most likely result at trial?
(A) The landowner will not prevail on the issue
of specific performance, but will be allowed
to keep the earnest money.
(B) The landowner will not prevail on the issue
of specific performance and will be ordered
to return the earnest money.
(C) The landowner will prevail on the issue of
specific performance, but the price will be
abated to $600,000.
(D) The landowner will prevail on the issue of
specific performance for the full contract
price.

A

(D) The landowner will succeed in her suit for specific performance at the full contract price. Where property subject to a contract for sale is destroyed without the fault of either party before the date set for closing, the rule in the absence of a statute is that the risk of loss is on the buyer. Thus, the buyer must pay the contract price despite a loss due to fire, unless the contract provides otherwise.
Here, the inn was destroyed by fire after the landowner and the buyer entered into their
contract for the sale of the property, but before the closing date. The contract apparently was silent regarding the risk of loss and there is no applicable statute. Thus, under the common law rule, the risk of loss is on the buyer. As a result, the landowner is entitled to receive specific performance of the contract, meaning that the buyer must pay the full contract price.

(A) and (B) are incorrect because they conclude that the landowner is not entitled to specific performance. As explained above, the landowner is entitled to specific performance because the risk of loss is on the buyer.
(B) is also incorrect because it states that the landowner must refund the earnest money. The
landowner is entitled to the full contract price; thus, there is no reason for her to return the earnest money.

(C) is incorrect because it allows the buyer to tender less than the full contract price. With
the buyer bearing the risk of loss, he must pay the $1 million contract price despite the decrease in the property’s value due to the fire.

149
Q

A man decided to give a cabin he owned to his daughter at his death. To accomplish this goal, he delivered to his attorney a deed that fully complied with the applicable statute of frauds and told his attorney to record the deed when he died unless he later gave the attorney instructions to the contrary.

Three weeks after dropping off the deed, the man properly drafted and executed his own will, which left all of his real property to his son.

One year later, the man died, and the attorney immediately recorded the deed. At the time of the man’s death, the cabin was titled in his name and he owned no other real property. The daughter and the son now disagree as to who is entitled to ownership of the cabin.

Other than the jurisdiction’s statute of frauds and statute of wills, there are no applicable statutes.

Who is entitled to ownership of the cabin?

A. The daughter, because the attorney was, for gift-law purposes, a trustee for the daughter.

B. The daughter, because the deed fully complied with the statute of frauds.

C. The son, because the deed was not delivered to the daughter during the man’s lifetime.

D. The son, because the proper execution of the will revoked the earlier gift to the daughter.

A

(C) is correct.

A deed is not effective to transfer an interest in realty unless it has been delivered. Delivery refers to the grantor’s intent. There must be words or conduct showing the grantor intended that the deed have some present operative effect- that title pass immediately and irrevocably, even though the right of possession may be postponed. If the right of possession is to be postponed until the grantor’s death, the deed may be held “testamentary” and therefore void (unless executed with testamentary formalities). However, most courts hold that if the grantor executes a deed and gives it to another with instructions to give it to the grantee upon the grantor’s death, the grantor’s intent was to presently convey a future interest to the grantee (either a remainder, with a life estate reserved in the grantor, or an executory interest), and so the gift is inter vivos, not testamentary. To make an effective delivery, the grantor must relinquish absolute and unconditional control. That was not the case here. The man gave the deed to the attorney to record when the man died “unless he later gave the attorney instructions to the contrary.” This language shows that the man did not relinquish control. Thus, there was no delivery, and the property passed by the will to the son.

(A) is incorrect. A trustee holds legal title to property and has an equitable duty to deal with it for the benefit of the beneficiary.

Here, the attorney was given a deed to hold and record at the appropriate time. The attorney never acquired title to the property, and the deed was in the daughter’s name. Also, the attorney’s duties were owed to the man; he had to hold the title and record it at the man’s death unless instructed otherwise.

(B) is incorrect. The statute of frauds is only one component of a valid transfer by deed. To transfer title, a deed must not only be in writing, but also be delivered. There must be words or conduct showing the grantor intended that the deed have some present operative effect-that title pass immediately and irrevocably, even though the right of possession may be postponed.

Here, the man gave the deed to the attorney to record at his death “unless he gave the attorney instructions to the contrary.” This language shows that the man did not relinquish control and thus there was no delivery. Without delivery, the daughter acquired no interest in the property.

(D) is incorrect. The execution of a will has no effect on the testator’s property because the will could be changed or revoked at any time until the testator dies. The will does not become effective until the testator dies. Thus, the execution of the man’s will did not revoke the gift of the property to the daughter. Furthermore even an operative will cannot revoke a validly executed and delivered deed. A will has no effect on it because the testator no longer owns the property. Had the man delivered the deed without conditions, the property would belong to the daughter and would not pass by will to the son.

150
Q

A credit card company obtained and properly filed a judgment against a man after he failed to pay a $10,000 debt. A statute in the jurisdiction provides as follows: “Any judgment properly filed shall, for 10 years from filing, be a lien on the real property then owned or subsequently acquired by any person against whom the judgment is rendered.”

Two years later, the man purchased land for $200,000. He made a down payment of $20,000 and borrowed the remaining $180,000 from a bank. The bank loan was secured by a mortgage on the land. Immediately after the closing, the deed to the man was recorded first, and the bank’s mortgage was recorded second.

Five months later, the man defaulted on the mortgage loan and the bank initiated judicial foreclosure proceedings. After receiving notice of the proceedings, the credit card company filed a motion to have its judgment lien declared to be the first lien on the land.

Is the credit card company’s motion likely to be granted?

A. No, because the bank’s mortgage secured a loan used to purchase the land.

B. No, because the man’s down payment exceeded the amount of his debt to the credit card company.

C. Yes, because the bank had constructive notice of the judgment lien.

D. Yes, because the bank is a third-party lender and not the seller of the land.

A

(A) is correct. The bank’s mortgage is a purchase-money mortgage, meaning that the funds the bank advanced were used to purchase the land.

A purchase-money mortgage executed at the same time as the purchase of the real property encumbered takes precedence over any other claim or lien, including a previously filed judgment lien. Therefore, the bank’s purchase-money mortgage takes precedence over the credit card company’s judgment lien.

(B) is incorrect. The relative amounts of the down payment and the credit card debt are irrelevant. Regardless of the amounts, a purchase-money mortgage executed at the same time as the purchase of the real property encumbered takes precedence over any other claim or lien, including a previously filed judgment lien.

(C) is incorrect. It is true that the judgment lien was properly filed and thus provided the bank with constructive notice of the lien. However, as discussed above, a purchase-money mortgage executed at the same time as the purchase of the real property encumbered takes precedence over any other claim, including a previously filed judgment lien.

(D) is incorrect. The bank’s mortgage is a purchase-money mortgage, which may be granted by a seller, by a third party, or both. As discussed above, the bank’s purchase-money mortgage takes precedence over the credit card company’s judgment lien.

151
Q

An easement that reduces the value of the property, such as an easement of way for the benefit of a neighbor, ….

A

… renders title unmarketable.

152
Q

A man and his neighbor owned homes on adjacent lots in a subdivision. The subdivision’s recorded restrictions did not prohibit detached storage sheds, and several homeowners in the subdivision had placed such sheds in their backyards. Because the man and the neighbor thought the sheds were unsightly, they both agreed in writing not to place detached storage sheds in their respective yards. Their agreement was drafted in recordable form and stated that it was enforceable by and against all assignees, heirs, and successors. The agreement was promptly recorded.

Three years later, the neighbor gave his home to his daughter. Shortly after moving into the home, the daughter learned of the restriction. She informed the man that she planned to put a detached storage shed in her backyard, claiming that the restriction was not enforceable against her.

Does the man have the right to enjoin the neighbor’s daughter from placing a detached storage shed in her yard?

A. No, because several homeowners in the subdivision have storage sheds in their yards.

B. No, because there was no horizontal privity between the man and the neighbor.

C. Yes, because the neighbor conveyed the home to the daughter by gift rather than by sale.

D. Yes, because the restriction is binding on the daughter as a successor.

A

(D) is correct.

A restrictive covenant can be enforced at law for damages (as a real covenant running with the land) or in equity for an injunction (equitable servitude). The requirements are different for each.

Here the facts state that the man is seeking an injunction to prevent the neighbor’s daughter from placing a shed in her yard. Thus, the man is trying to enforce the restrictive covenant as an equitable servitude. An enforceable equitable servitude requires that the covenanting parties intended the servitude to be enforceable against successors in interest, the successor in interest must have notice of the covenant, and the covenant must touch and concern the land.

Here, the original parties clearly intended their successors to be bound because they stated in the agreement that it was enforceable by and against all assignees, heirs, and successors. The agreement was recorded. Thus, all successors would have constructive (record) notice of the covenant. For the benefit of a covenant to touch and concern the land, the promised performance must benefit the covenantee and their successors in their use and enjoyment of the benefited land. Here, the performance (not building a shed) benefited the man and his successors in their use and enjoyment of the land because they could enjoy their property without viewing sheds they consider unsightly. All requirements for an equitable servitude are met, and the main has a right to enjoin the neighbor’s daughter.

(A) is incorrect because the question concerns enforcing an agreement between two parties; it does not involve a common development scheme. Thus, what the other neighbors do is irrelevant.

(B) is incorrect. Horizontal privity is required for a burden of a real covenant to run at law to successors in interest. It would have been required had the man been seeking damages rather than an injunction. Horizontal privity requires that the original covenanting parties share some interest in the land independent of the covenant (e.g., grantor-grantee). Here, the parties to the agreement were merely neighbors and had no legal relationship; thus there was no horizontal privity. As noted above, the man may still enforce the agreement as an equitable servitude. (C) is incorrect. The fact that the daughter took the property as a gift prevents her from being a bona fide purchaser for value (“BFP”). A BFP takes free of a restrictive covenant if they had no notice of it. Here, the daughter is not a purchaser and she had record notice of the covenant. Thus, she would take subject to the restriction if it were enforceable against her. So the question is whether the restriction is enforceable against the neighbor’s successor in interest. The answer is yes.

153
Q

A woman owned land in fee simple absolute. The woman conveyed the land to a friend “for life,” and when the friend died the land was to go to the woman’s neighbor “and her heirs.”

The neighbor died and in her duly probated will devised her entire estate to a local charity. If she had died intestate, her daughter would have been her only heir.

One year after the neighbor died, her daughter executed a quitclaim deed conveying any interest she might have in the land to the woman’s friend.

The common law Rule Against Perpetuities is unmodified in the jurisdiction. There are no other applicable statutes.

Who has what interest in the land?

A. The friend has a fee simple absolute, because his life estate merged with the remainder conveyed to him by the daughter.

B. The friend has a life estate and the charity has a vested remainder, because the neighbor’s interest was devisable.

C. The friend has a life estate and the daughter has a vested remainder, because the deed from the woman created an interest in the neighbor’s heirs.

D. The friend has a life estate and the woman has a reversion, because the neighbor’s remainder was void under the Rule Against Perpetuities.

A

(B) is correct.

The grant here gave a life estate to the friend and a remainder in fee simple to the neighbor. It is a remainder because on the friend’s death (the natural termination of the preceding estate), the neighbor has the right to possession. The remainder is vested because it was granted to an ascertainable person in being (the neighbor) and there were no conditions to prevent it from becoming possessory.

A vested remainder is devisable by will; so when the neighbor devised her estate to the local charity, the charity took her vested remainder interest.

(A) is incorrect. Whenever the same person acquires all of the existing interests in land, present and future, a merger occurs, and that person then holds a fee simple absolute.

Here, however, the daughter had nothing to convey to the friend. The woman conveyed a vested remainder in fee simple to the neighbor. That vested remainder is devisable, and the neighbor properly devised it by will to the local charity. The daughter, as an intestate heir, would have taken no interest in the property. A quitclaim deed conveys only the interest the grantor has, and in this case, that was nothing. Therefore, there was no merger. The daughter’s deed conveyed no interest to the friend, and the friend has only his life estate.

(C) is incorrect. The phrase in the grant “and her heirs” indicates that a fee simple interest was being granted; it does not give the heirs any rights in the property. Thus, as discussed, the daughter acquired no interest in the remainder.

(D) is incorrect. The Rule Against Perpetuities states that no interest in property is valid unless it must vest, if at all, within 21 years after a life in being at the creation of the interest.

Here, the neighbor’s interest is already vested on creation; she has a vested remainder. Her estate is a vested remainder because it followed the natural termination of the preceding estate, and it was granted to an ascertainable person in being without any conditions to prevent it from becoming possessory. Because it is a vested remainder, it is valid under the Rule.

154
Q

Ten years ago, a couple bought a building and moved into its second-floor apartment with their teenage daughter. The couple operated a shoe store on the first floor of the building for many years. When the couple purchased the building, the area was predominantly rural and was zoned for nonresidential use. The municipality’s zoning is cumulative.

Five years ago, the municipality rezoned the area to single-family residential use. The daughter was not aware of this change, since she was away at college.

Recently, the daughter inherited the building from her parents. The daughter immediately moved into the apartment and took over the operation of the shoe store on the first floor. The daughter has learned that a developer is planning to build a large residential community in the area surrounding her building.

The daughter has asked her lawyer for advice regarding her ability to continue operating the shoe store.

Should the lawyer advise the daughter that she can continue to operate her shoe store?

A. No, because the nonconforming use of the building terminated when the daughter’s parents died.

B. No, but the municipality must pay her reasonable compensation for her loss resulting from the change in zoning.

C. Yes, because the shoe store is a nonconforming use.

D. Yes, because the zoning is cumulative and the building is also used for single-family residential purposes.

A

(C) is correct.

A cumulative zoning ordinance creates a hierarchy of uses of land, and land that is zoned for a particular use may be used for the stated purpose or for any higher use. A residential use is higher than a nonresidential use.

Here, the building was in an area originally zoned for nonresidential use. The daughter and her parents used the property for a business and their residence. This was appropriate under the cumulative zoning ordinance as the family’s uses met or exceeded the zoned use. Later, the area was rezoned for single-family residential use, which is a higher use than the shoe store. However, a use that exists at the time of passage of a zoning ordinance and that does not conform cannot be eliminated at once. Generally, the nonconforming use may continue indefinitely, but any change in the use must comply with the zoning ordinance. Because the shoe store existed at the time of the rezoning, the daughter may continue to operate the shoe store as a nonconforming use.

(A) is incorrect. Generally, the nonconforming use may continue indefinitely. The key is the use of the property and not the ownership. The parents’ deaths do not affect the zoning status of the building. Thus, the daughter may live in the building and operate the shoe store.

(B) is incorrect. The zoning power is limited by the “no taking without just compensation” clause of the Fifth Amendment. Nevertheless, zoning is generally not a “taking” unless it amounts to a physical appropriation of the property or denies the owner of all economic use, which would not be the case here (the daughter could use the property as a single-family residence). Because the property is a nonconforming use, she will be allowed to continue that use.

(D) is incorrect. A cumulative zoning ordinance creates a hierarchy of uses of land, and land that is zoned for a particular use may be used for the stated purpose or for any higher use. A single-family home is a higher use than an apartment building or a commercial use such as a shoe store. Because the shoe store is a lower use than a single-family residence, it would not be allowed in this area. Nor would the daughter’s apartment above the store. The store and apartment are allowed only because they are a nonconforming use.

155
Q

Cumulative Zoning

A

A cumulative zoning ordinance creates a hierarchy of uses of land (e.g., a single-family
home is a higher use than an apartment building, which is a higher use than a strip mall,
which is a higher use than a factory).

Under a cumulative zoning ordinance, land that is zoned for a particular use may be used for the stated purpose or for any higher use.

Under a noncumulative zoning ordinance, land may be used only for the purpose for which it is
zoned.

156
Q

Nonconforming Use

A

A use that exists at the time of passage of a zoning ordinance and that does not conform
cannot be eliminated at once. Generally, the nonconforming use may continue indefinitely,
but any change in the use must comply with the zoning ordinance.

Amortization is the gradual elimination of such uses.

157
Q

Special Use Permit

A

A special use permit is one that must be obtained even though the zoning is proper for the
intended use. It is often required for hospitals, funeral homes, drive-in businesses, etc.

158
Q

Variance

A

A variance is a departure from the literal restrictions of a zoning ordinance granted by
administrative action.

159
Q

A city filed eminent domain proceedings in order to obtain 40 beach houses fronting a particularly attractive stretch of shoreline. As part of an elaborate plan to increase the city’s tourist trade and revive the local economy, the city planned to sell the beach houses to a company that would demolish the houses and build a luxury hotel in their place.

The owners of the beach houses have challenged the city’s exercise of eminent domain, contending only that the city’s plan is unconstitutional.

Will the owners of the beach houses be likely to prevail?

A. No, because a property owner can challenge an exercise of eminent domain only on the ground of the sufficiency of the compensation.

B. No, because the planned sale to the private developer to increase the tourist trade qualifies as a public use.

C. Yes, because a public entity cannot seize the property of one person in order to transfer that property intact to other private parties.

D. Yes, because the city’s action would deprive the owners of all economic use of their property.

A

(B) is correct.

The Fifth Amendment prohibits taking of private property for public use without just compensation.

The Supreme Court has held that taking property to turn it over to a private developer can still qualify as a public use.

(A) is incorrect because it is not true. The Fifth Amendment requires both just compensation and a public use as prerequisites to exercise of the power of eminent domain.

(C) is incorrect because, as indicated above, it is not true. The Supreme Court has approved such transfers. So long as it appears it was reasonable for the government to believe the transfer to the private entity will benefit the public, such as by promoting economic development, the public use requirement is met.

(D) is incorrect for the reasons stated above.

160
Q

Fifteen years ago, two men who were fishing buddies moved onto vacant rural land owned by a woman they didn’t know and built a small fishing shack on it. Twelve years ago, the men replaced the shack with a fish processing plant and a commercial fishing boat dock. The men maintained their commercial fishery operation on the land until one of them died intestate last year, leaving a sole heir.

The period of time to acquire title by adverse possession in the jurisdiction is 10 years.

The woman has now become aware of the changes that have occurred on the land.

In an action to determine title, for whom should the court decide?

A. For the man who is still alive and the deceased man’s heir, because the men acquired title as tenants in common.

B. For the man who is still alive, because he is the surviving adverse possessor.

C. For the woman, because the use was changed by the men while they were in possession.

D. For the woman, because title cannot be claimed by two adverse possessors simultaneously.

D.

A

(A) is correct. Title to the property was acquired by adverse possession.

For adverse possession, the possession must be:

(i) actual (it must give the true owner notice of the trespass) and exclusive (not shared with the true owner or the public);
(ii) open and notorious possession (use of the property as an owner would be sufficient to put the true owner on notice of the trespass);
(iii) adverse (sometimes called hostile, which means without the owner’s permission); and
(iv) continuous for the statutory period.

Two or more people can work together to take title by adverse possession, and if they meet the requirements, they take as tenants in common.

Here, the men took actual possession of the land by building a fish processing plant and dock. They did not share possession with the woman or the public. Their use of the property was as an owner might use the land, and the improvements and activities on the land were sufficient to put the woman on notice. The men did not have the woman’s permission, and possession was continuous for more than is required by the statute. Therefore, they gained title by adverse possession and took the property as tenants in common. A tenancy in common is a concurrent estate with no right of survivorship. Each tenant has a distinct, undivided interest in the property. This interest is freely alienable and is inheritable. When one of the men died intestate, leaving a sole heir, the heir inherited the man’s interest in the property. Note that building the fishing shack may not have been enough for adverse possession. It was small and possibly not enough to give notice to the woman of the trespass. It was probably used only occasionally for fishing, and that may be a use that an owner of the property would make, but that is not certain. In any case, the plant and dock were open and notorious and for the statutory period. (B) is incorrect. When two people work together to take title by adverse possession, they take as tenants in common, an estate that has no right of survivorship. Thus, the man who is still alive is a tenant in common with the deceased man’s heir. (C) is incorrect. Change of use is not relevant as long as each use meets the requirements for adverse possession. In any case, the use of the processing plant and dock was for more than the statutory period, so the men would take by adverse possession based on that one use. (D) is incorrect. Two or more people may work together to take by adverse possession.

161
Q

A landowner leased 150 acres of farmland to a produce company for 15 years. The produce company used the land for crops along with several other contiguous acres that it owned or leased. About four years into the lease, the state condemned a portion of the leased property because it intended to build a highway. As a result, too little property remained for the produce company to profitably farm, although there still existed the farmhouse on the property, which was being used by one of its foremen. The produce company gave the landowner 30 days’ written notice that it considered the lease to have been terminated because of the condemnation.

In a suit for breach of contract, is the landowner likely to win?

A

The landowner probably will win in a breach of contract suit. In partial condemnation cases, the landlord-tenant relationship continues, as does the tenant’s obligation to pay the entire rent for the remaining period of the lease. The tenant is, however, entitled to share in the condemnation award to the extent that the condemnation affected the tenant’s rights under the lease. Therefore, (B) and (C) are incorrect. (A) is not correct because the law of landlord and tenant traditionally refuses to recognize frustration of purpose as grounds for termination of a lease.

162
Q

A father executed a deed to his art gallery “to my daughter for her life, and on my daughter’s death to her children; provided, however, that if my daughter stops painting, to my brother.” The daughter has two children and is still painting.

At the time of the grant, what is the best description of the interest of the daughter’s two children?

A A contingent remainder.

B A vested remainder subject to open and to total divestment.

C A vested remainder subject to open.

D An executory interest.

A

The daughter’s two children have a vested remainder subject to open and subject to complete divestment.

A remainder is a future interest created in a transferee that is capable of taking in possession on the natural termination of the preceding estate.

A remainder is vested if the beneficiaries are ascertainable and their taking in possession is not subject to a condition precedent.

A vested remainder created in a class of persons that is certain to take but is subject to diminution by reason of others becoming entitled to take is a vested remainder subject to open.

Vested remainders may be subject to total divestment if possession is subject to being defeated by the happening of a condition subsequent.

Here, the daughter’s two children have a remainder because, on the expiration of the daughter’s life estate, they will be entitled to possession of the property. The remainder is not subject to a condition precedent and the beneficiaries are in existence and ascertained, so the remainder is vested, not contingent. The remainder is subject to open because the daughter may have more children. Finally, the remainder is subject to total divestment because the daughter’s children’s right to possession is subject to being defeated by the daughter’s ceasing to paint.

(A) is wrong because the remainder is vested, not contingent; i.e., it is not subject to a condition precedent, and the beneficiaries are ascertainable. (C) is not the best answer because it is incomplete. The vested remainder here is also subject to total divestment. (D) is wrong because the children’s interest does not divest the daughter’s estate, which would indicate an executory interest. Rather, their interest is capable of taking in possession on the natural termination of the daughter’s estate, and thus is a remainder.

163
Q

A vintner divided his vineyard into two parcels, drawing the boundaries so that the single well that had irrigated the entire vineyard fell on the border of the two properties. The vintner then conveyed the eastern parcel to his friend by a deed that contained the following covenant:

“If the well located on the boundary of the eastern and western parcels continues to be used for irrigation purposes and becomes in need of repair or replacement, the grantee, his heirs, and assigns and the grantor, his heirs, and assigns each promise to pay one-half of the cost of such repair or replacement. This covenant shall run with the land.”

The deed from the vintner to the friend was not recorded, and the vintner did not record a copy of the deed with the records for the western parcel.

The friend later sold the eastern parcel to a farmer. The farmer’s deed did not contain the covenant about the well. After 15 years of use by the owners of both the eastern and western parcels, the well began to fail. The farmer took it upon himself to have the well repaired at a cost of $30,000. About two weeks later, the farmer discovered the deed from the vintner to the friend in some old files. By this time, the western parcel had passed to the vintner’s son by inheritance and again to the son’s daughter by inheritance from the now-deceased son. The daughter knew nothing of the covenant concerning the well. The farmer presented the daughter with the bill for the well repair with a copy of the vintner/friend deed and a note that said he expected to be reimbursed for $15,000. The daughter refuses to pay, and the farmer sues.

The jurisdiction has a 10-year statute of limitations for acquiring property by adverse possession, and the following recording statute: “Any conveyance of an interest in land shall not be valid against any subsequent purchaser for value, without notice thereof, unless the conveyance is recorded.”

For whom is the court most likely to rule?

A The daughter, because the deed from the vintner to the friend was never recorded.

B The daughter, because the farmer has acquired the well by adverse possession.

C The farmer, because the covenant runs with the land.

D The farmer, because he is a bona fide purchaser.

A

The farmer will most likely prevail in his suit for one-half the cost of the well repairs because the covenant runs with the land.

When a covenant runs with the land, subsequent owners of the land may enforce or be burdened by the covenant. If all of the requirements for the burden to run are met, the successor in interest to the burdened estate will be bound by the arrangement as effectively as if he had himself expressly agreed to be bound. To be bound:

(i) the parties must have intended that the covenant run with the land;
(ii) the original parties must have been in horizontal privity;
(iii) the succeeding party must be in vertical privity with the original promisor;
(iv) the covenant must touch and concern the land; and
(v) generally, the burdened party must have actual or constructive notice of the covenant.

Here, the intent is shown by the express language of the covenant, which says that it is intended to run with the land. Even without that language, the use of the words “heirs” and “assigns” would show the intent for the covenant to run.

The original parties were in horizontal privity because at the time the vintner entered into the covenant, he and the friend shared an interest in the land independent of the covenant—as grantor and grantee.

The daughter is in vertical privity with the vintner because she holds the entire interest in the western parcel held by the vintner.

The covenant touches and concerns the land because promises to pay money to be used in a way connected with the land are held to touch and concern the property.

Because the daughter was unaware of the covenant, the required notice seems to be missing. While it is generally true that the owner of the burdened land must have notice, it should be remembered that the requirement is a function of the recording statute. (At common law, the covenant was enforceable in an action for damages regardless of notice; this was changed by the recording statutes.) However, because the daughter is a donee (an heir) and not a bona fide purchaser, she is not protected by the recording statute and thus is subject to the covenant even without notice.

164
Q

A landowner included in his will a provision giving “all of my property, both real and personal, wherever situated, to my widow for life, and after her death to any of our children who may survive her.”

What is the gift to the children?

A A contingent remainder.

B A vested reminder.

C A shifting executory interest.

D Void, as violating the Rule Against Perpetuities.

A

The children have a contingent remainder.

A remainder is a future interest created in a transferee that is capable of taking in present possession on the natural termination of the preceding estate created in the same disposition. Note that, as a rule of thumb, remainders always follow life estates. A remainder will be classified as contingent if its taking is subject to a condition precedent, or it is created in favor of unborn or unascertained persons.

Here, the interest in the children follows a life estate and is a remainder because it is capable of taking in possession on the natural termination of the preceding estate. It is subject to the condition precedent of surviving the landowner’s widow and, additionally, is in favor of unascertained persons (the children who survive the landowner’s widow will not be ascertained until her death). Thus, the interest is a contingent remainder.

(B) is incorrect because a vested remainder can be created in and held only by ascertained persons in being, and cannot be subject to a condition precedent. As discussed above, the will provision clearly does not satisfy these requirements because the takers are not ascertained and their interest is subject to a condition of survival. (C) is incorrect because a shifting executory interest is one that divests the interest of another transferee; i.e., it cuts short a prior estate created by the same conveyance. The gift to the children does not divest the interest of the widow; she retains a life estate in the property. The children’s interest takes in possession only on the natural termination of the widow’s estate (i.e., at her death). (D) is incorrect because the interest does not violate the Rule Against Perpetuities. The children’s interest will vest, if at all, not later than 21 years after the lives in being. The landowner’s widow and the children themselves are lives in being. There is no unborn widow problem because the instrument takes effect on the landowner’s death and the gift is to his own widow. She must be in being at his death. Likewise, his children would be in being at his death. Thus, the vesting will be within the period of the Rule.

165
Q

A landowner validly conveyed a small office building to the Green Party “as long as they use it for operating quarters until the next presidential election.” After the next presidential election, which was in three years, the building would go to a private organization that monitors and prepares comprehensive listings of gas prices throughout the country. A year after the conveyance, the landowner died, validly devising all of her property to her son. Although this jurisdiction is a common law jurisdiction with respect to all real property considerations, the state’s probate laws provide that future interests or estates in real property may be passed by will or descent in the same manner as present or possessory interests. Last week, the Green Party and the gas monitoring organization joined together to sell the office building in fee simple absolute to a developer. The son filed suit to prevent the sale of the property to the developer.

In this action, who should prevail?

A The Green Party and the gas monitoring organization, because together they own a fee simple absolute in the building.

B The Green Party and the gas monitoring organization, because the attempted restrictions on the use of the property violate the Rule Against Perpetuities.

C The Green Party and the gas monitoring organization, because the deed restriction was an unlawful restraint on alienation.

D The son, because he did not sign the contract of sale

A

The son may enjoin the sale because he has an interest in the property.

A fee simple determinable is an estate that automatically terminates on the happening of a stated event.

The Green Party’s interest in the office building is a fee simple determinable because it lasts as long as the Party is using the building for operating quarters. However, the grant does not provide for the contingency of the Green Party ceasing to use the building as operating quarters before the next presidential election. This gap would be filled by a possibility of reverter retained by the landowner. Because the landowner passed that interest to her son in her will, there can be no contract to sell the property without his signature.

Note: Although the gas monitoring organization appears to have an indefeasibly vested remainder (i.e., it is created in an ascertained company, is certain to become possessory, and is not subject to being defeated, divested, or diminished in size), its interest is not capable of taking on the natural termination of the preceding estate and so is characterized as a springing executory interest.

(A) is wrong because the son also has an interest in the land. (B) is wrong because the interest in the office building will pass to the gas monitoring organization, if at all, within 21 years. (C) is wrong because the Green Party is not prohibited from transferring any interest; it could pass a defeasible fee.

166
Q

Does adverse possesor need to believe that she has a right to possession?

A

The possessor need not believe that she has a right to possession.

i.e.

Twenty-five years ago, the landowner dug a water well on a section of the property that she thought was hers, but in fact was the neighbor’s.

167
Q

Where an installment land contract is used, the seller’s obligation is to furnish marketable title when ….

A

…. delivery is to occur, e.g., when the buyer has made his final payment

168
Q

A life tenant is entitled to all ordinary uses and profits of the land, but he cannot lawfully do any act that would injure the interests of the remainderman. A grantor intends that the life tenant have the general use of the land in a reasonable manner, but that the land pass to the owner of the remainder, as nearly as practicable, unimpaired in its nature, character, and improvements.

Is ameliorative waste actionable?

A life tenant can substantially alter or even demolish existing buildings if ….

A

Even ameliorative waste, which actually increases the value of the land, is actionable if there is no reasonable justification for the change.

A life tenant can substantially alter or even demolish existing buildings if

(i) the market value of the future interests is not diminished and either (ii) the remainderman does not object, or (iii) a substantial and permanent change in the neighborhood conditions has deprived the property in its current form of reasonable productivity or usefulness.

169
Q

A seller conveyed her residential city property to a buyer by a general warranty deed. On taking possession of the property, the buyer discovered that the garage of his neighbor encroached six inches onto his property.

If the buyer wishes to compel the seller to assist him in a suit against the neighbor, which of the following covenants may he rely on to do so?

A

The buyer would rely on the covenants of warranty and further assurances to compel the seller to assist him in a suit against his encroaching neighbor.

Under the covenant of warranty, the grantor agrees to defend, on behalf of the grantee, any lawful or reasonable claims of title by a third party, and to compensate the grantee for any loss sustained by the claim of superior title.

The covenant for further assurances is a covenant to perform whatever acts are reasonably necessary to perfect the title conveyed if it turns out to be imperfect.

These covenants are “continuous” (run with the land) and require the grantor to assist the grantee in establishing title.

The covenants of seisin and encumbrances do not require such assistance.

A covenant of seisin is a covenant that the grantor has the estate or interest that she purports to convey. Both title and possession at the time of the grant are necessary to satisfy this covenant.

The covenant against encumbrances is a covenant assuring that there are neither visible encumbrances (easements, profits, etc.) nor invisible encumbrances (mortgages, etc.) against the title or interest conveyed.

While the seller may have violated these two covenants because of the garage encroachment, they do not provide the basis to compel her to assist the buyer in a title suit. Instead, the buyer merely has a cause of action against the seller for their breach. Therefore, (A), (C), and (D) are wrong.

170
Q

Covenants in General Warranty Deed

A

a. Usual Covenants
The following are the usual covenants for title contained in a general warranty deed.

1) Covenant of Seisin
The grantor covenants that she has the estate she purports to convey. She must have both title and possession at the time of the grant.

2) Covenant of Right to Convey
The grantor covenants that she has the authority to make the grant. Title alone will satisfy this covenant.

3) Covenant Against Encumbrances
The grantor covenants against the existence of physical (e.g., encroachments) or title (e.g., mortgages) encumbrances.

4) Covenant for Quiet Enjoyment
The grantor covenants that the grantee will not be disturbed in possession by a third party’s lawful claim of title.

5) Covenant of Warranty
The grantor agrees to defend against reasonable claims of title by a third party and to compensate the grantee for any loss sustained by the claim of superior title.

6) Covenant for Further Assurances
The grantor promises to perform acts reasonably necessary to perfect title conveyed.

b. Breach of Covenants

Three of the covenants (seisin, right to convey, against encumbrances) are breached, if at all, at the time of conveyance.

Quiet enjoyment, warranty, and further assurances are future covenants and are breached only upon disturbance of the grantee’s possession.

Present covenants cannot be enforced by remote grantees, but future covenants run with the grantee’s estate.

c. Damages and Remote Grantees
If there are successive conveyances by general warranty deed and the last grantee is evicted by lawful claim of title, he may sue anyone up the line. Some states allow him to recover to the extent of consideration received by a defendant-covenantor. Other states limit recovery to the lesser of what he paid or what the defendant-covenantor received.

171
Q

A seller entered into a written contract with a vintner on April 4, whereby the seller agreed to convey a vineyard to the vintner for $2 million. The terms of the contract set the closing date as June 1. At the time the seller entered into the agreement with the vintner, the seller had no interest in the vineyard. On April 15, the seller entered into a written agreement with a landowner, whom the seller believed to be the owner of the vineyard. According to the terms of the agreement, the landowner was to convey the vineyard to the seller on or before May 25. Another term of the agreement stated “time is of the essence.”

On May 24, the landowner conveyed his interest in the vineyard to the seller. When the seller went to record the deed, she discovered from records in the recorder’s office that the landowner held clear title to only seven-eighths of the vineyard. It took some time for the seller to remove the cloud from the title and procure ownership in full of the vineyard. She finally did so on August 1, and on that day she tendered a warranty deed to the vineyard to the vintner. The vintner refused to tender $2 million or any other sum to the seller, asserting that the seller had broken her agreement by failing to close on June 1. The seller then sued the vintner for specific performance.

If the vintner prevails, what is the likely reason?

B Title was unmarketable, because the seller only owned seven-eighths of the vineyard on the closing date.

D A two-month delay in closing is determined to be unreasonable.

A

If the vintner prevails, it will be because the court determines that the seller’s two-month delay in closing was unreasonable. Generally, courts assume that time is not “of the essence” in real estate contracts. This means that the closing date stated in the contract is not absolutely binding in equity, and that a party, even though late in tendering her own performance, can still enforce the contract if she tenders within a reasonable time after the date.

(B) is incorrect because the seller has a reasonable time after the date set for closing to tender performance unless the contract or circumstances indicate that time is of the essence.

172
Q

A landowner owned a beachfront lot and home in a subdivision occupying several hundred acres near a lake. The recorded subdivision plan grants to each owner in the subdivision an easement to use the private roads therein for personal ingress and egress.

Following seismic activity in the area, the level of the lake dropped substantially, exposing a considerable amount of land between the new shoreline and the old beachfront. It was judicially determined that this “new” land belonged to the county, which put portions of it up for sale. The landowner purchased the land extending from her old property line to the new shoreline, and constructed a boat launching ramp on the new property. She then permitted persons who did not own land in the subdivision to drive through her old property to reach the boat launching ramp on her new property, and thus to utilize the lake, for a small fee. The homeowners’ association brought suit against the landowner, seeking to enjoin her from using or permitting nonresidents of the subdivision from traveling its streets to reach the boat launching ramp.

How should the court rule?

A For the homeowners, because the scope of the easement granted to the landowner as an owner in the subdivision does not extend to the use that she is making of the new property.

B For the landowner, because she has an express easement over the streets of the subdivision.

A

The express easement for the landowner’s old property benefits that property only and cannot be used for the landowner’s expanded access to the new property.

An easement is a liberty, privilege, or advantage that one may hold in the lands of another. The holder of an easement has the right to use a tract of land (called the servient tenement) for a special purpose; e.g., laying utility lines, or for ingress and egress.

An easement can be created, as in this question, by express grant.

If the parties to the original creation of the use specifically state the location of the easement, its dimensions, and the special use or limits to such use, the courts will honor this expression of specific intent.

Absent specific limitations, it will be assumed that the parties intend that the easement meet both present and future reasonable needs of the dominant tenement. However, a basic change in the nature of the use is not allowed. The landowner’s easement by express grant merely allows her to use the private roads in the subdivision for her personal ingress and egress to and from her beachfront property. The use of the easement for access to a new boat launching ramp for which a fee is charged goes beyond the specific language of the grant (and arguably beyond the reasonable needs of the dominant tenement). Therefore, the homeowners will be able to prevent use of the subdivision streets to reach the boat launching ramp.

173
Q

A corporation was in the business of purchasing real property at below-market prices and reselling the properties to investors. The bylaws of the corporation authorized the chief executive officer (“CEO”) and the director of the marketing division to enter into contracts on behalf of the corporation for the purchase or sale of properties. The corporation had recently purchased a large parcel of beachfront property for resale. The CEO secretly opened negotiations with an amusement park to sell the property. However, unknown to the CEO or anyone else in the corporation, the marketing director had already reached an agreement with a hotel for the sale of the property.

On April 23, the marketing director and the hotel signed a written contract providing for sale of the property by the corporation to the hotel for $35 million. On April 25, the board of directors amended its bylaws, effectively depriving the marketing director of authorization to bind the corporation in purchase or sale transactions. This action was immediately publicized and became known to both the marketing director and the hotel. On April 26, the hotel duly recorded its contract. On May 1, the CEO, still unaware of the marketing director-hotel agreement, approved sale of the property to the amusement park for $39 million. The necessary documents of title were prepared and properly recorded by the amusement park on May 5. Two days later, the amusement park learned of the marketing director-hotel agreement. On May 10, the date scheduled for closing of the hotel’s sale agreement, the CEO refused to accept the hotel’s tender of $35 million and refused its demand for a deed to the property.

The hotel subsequently brings action against the corporation and the amusement park for specific performance and to quiet title to the property. For whom will the court likely rule?

A The defendants, because the board of directors had deprived the marketing director of authority to bind the corporation in the sale of real property.

B The defendants, because the amusement park is the only purchaser who properly recorded a deed to the property.

C The hotel, because the amusement park had constructive notice of the hotel’s interests in the property when the agreement with the CEO was made.

D The hotel, because the attempt to divest the marketing director of authority to approve sales of the corporation’s property was invalid.

A

Judgment should be for the hotel regardless of whether the jurisdiction has a notice statute or a race-notice statute. Under either type of recording statute, the only persons protected by the statute are bona fide purchasers. To attain this status, the person must take without notice—either actual, constructive, or inquiry—of the prior instrument. Because the marketing director-hotel contract was properly recorded, the amusement park had constructive notice of the hotel’s interest in the property. Thus, the park could not become a bona fide purchaser when it entered into its contract. (A) is wrong because the marketing director had not been deprived of authority to bind the corporation at the time she signed the agreement with the hotel, and any subsequent change in her powers did not affect the validity of that agreement, nor the hotel’s power to subsequently record the agreement. (B) is wrong because the hotel recorded its contract of sale. Any instrument creating or affecting an interest in land (e.g., deed, mortgage, contract to convey) can be recorded, providing constructive notice to subsequent purchasers. Thus, the hotel’s failure to record a deed does not deprive it of protection of the recording statute. (D) is wrong because regardless of the validity of the board’s attempt to divest the marketing director of authority, it is immaterial to the hotel’s rights; it came after the valid marketing director-hotel contract was properly signed by the marketing director.

174
Q

A developer owned a 240-acre parcel of land zoned for commercial and residential use. He prepared and recorded, after obtaining approval from all appropriate agencies, a subdivision plan that included a commercial center and a number of lots for single- and multi-family residences. The list of covenants, conditions, and restrictions recorded with the plan included provisions that required every building constructed in the subdivision to be of “simulated adobe style” architecture approved in advance by an association. A year later, the developer sold many of the lots in the commercial center, including several to a real estate firm. Each deed prepared by the developer contained a reference to the design restriction in the recorded plan. The developer also sold almost all of the residential lots, the deeds of which contained the same reference to the restriction. The following year, the real estate firm sold one of its lots to a burger franchise. The deed contained no reference to the design restriction. The franchise’s prefabricated restaurant, complete with a giant burger logo mounted on the roof, was constructed over the weekend.

A merchant, an original purchaser of one of the commercial lots, owned the lot next to the burger franchise. She did not learn of construction of the restaurant until she came in to work on Monday, and saw the giant burger logo. The merchant brings an action seeking a mandatory injunction compelling the burger franchise to demolish the restaurant. At trial, the merchant proves that the burger franchise did not seek or obtain approval of the association for its building.

Should the court issue the injunction?

A No, because destruction of the restaurant would be a tremendous waste of resources.

B No, because the burger franchise’s deed contained no restriction on the type of building that could be constructed on the lot.

C Yes, because the restrictive covenant runs with the land.

D Yes, unless the burger franchise can establish to the court’s satisfaction that its restaurant design has at least as much aesthetic merit as any “simulated adobe style” design.

A

The court should issue the injunction because the covenant runs with the land.

A covenant will be enforceable as an equitable servitude—allowing a covenantee, covenantor, or successor to enforce the covenant in equity by way of injunction—when there is

(i) a covenant in a writing satisfying the Statute of Frauds, that
(ii) touches and concerns the land (i.e., the effect of the covenant makes the land more useful or valuable to the benefited party) and that
(iii) indicates an intention that the servitude exists, and
(iv) notice is given to future owners of the burdened land.

Here, the covenant was in writing in the subdivision plan and presumably it satisfied the Statute of Frauds.

It touches and concerns the land—benefiting all of the lots and burdening all of the lots.

The intention to create the servitude is established by the writing and can also be implied from the common scheme for development.

There was sufficient record notice of the covenant because the plan was recorded and was noted in all of the original deeds prepared by the developer, including the one in the burger franchise’s chain of title. Thus, the covenant is enforceable and (C) is the best answer.

(D) is incorrect because a court will not modify the covenant—it will enforce it or not enforce it, but will not substitute its judgment of what is aesthetically pleasing for the requirements of the covenant.

175
Q

The statute of limitations that determines the time period for adverse possession does not run against …

A

… the holder of a future interest (e.g., a remainder) until that interest becomes possessory, because the holder of the future interest has no right to possession (and thus no cause of action against a wrongful possessor) until the prior present estate terminates.

i.e.

Here, the society has possessed the home for eight years; however, as against the granddaughter’s daughter, the holder of the remainder, the statute did not begin to run until the death of the grandson. Prior to the termination of the grandson’s life estate, the granddaughter’s daughter had no cause of action against the society because she had no right to possession. Upon the grandson’s death, when the granddaughter’s daughter’s interest became possessory, the statute began to run against her. Thus, as against her, the society has not been in adverse possession for the requisite period.

176
Q

A mother died, bequeathing all of her property to a trustee “to pay the income to my husband for life, and to distribute the principal to my son and daughter if they graduate from college. If they do not graduate from college, then the principal shall be distributed to charity.” Subsequently, the son and the daughter graduated from college.

Upon their graduation from college, how would the interests of the son and the daughter in the trust principal be classified?

A Tenants in common to a vested remainder.

B Joint tenants to a vested remainder.

C Tenants in common to a fee simple absolute.

D Tenants by the entirety to a fee simple absolute.

A

Upon their graduation, the interests of the son and the daughter in the trust principal would be classified as tenants in common to a vested remainder.

A remainder is classified as contingent if its taking in possession is subject to a condition precedent. Here, because the condition precedent—that the son and the daughter graduate from college—has been satisfied, the son and the daughter’s contingent remainder has “vested.” Also, at common law, it was held that any conveyance to two or more persons was presumed to create a joint tenancy unless a contrary intention was clearly expressed. But today all courts hold that such a conveyance creates a tenancy in common. To create a joint tenancy, words such as “as joint tenants with right of survivorship” must normally be used to show the necessary intent. Thus, the son and the daughter are tenants in common to a vested remainder. (B) is therefore wrong. (C) is wrong because the son and the daughter’s estate is in a remainder interest, not a fee simple absolute, because the facts do not indicate that the life tenant (the husband) has died. (D) is wrong for the same reason and, additionally, because a tenancy by the entirety can be held only by a husband and wife, which is not the case here.

177
Q

As a result of a personal injury lawsuit, a victim obtained a judgment against a tortfeasor for $100,000. The tortfeasor, who had few assets, did not pay the judgment. On April 1 of the following year, the tortfeasor inherited a parcel of land from her uncle. On May 1, the tortfeasor entered into a contract with a buyer to sell the land for $120,000. The contract was not recorded. The buyer immediately applied to a bank for a loan of $100,000. The bank approved the buyer’s loan, and on May 15, a closing was held. The tortfeasor deeded the land to the buyer, and the buyer executed a mortgage for $100,000 to the bank. Due to an error by the title company, the deed from the tortfeasor to the buyer was not recorded, although the mortgage to the bank was recorded. Neither the buyer nor the bank had any knowledge of the victim’s judgment. On May 20, the victim recorded his judgment in the county recorder’s office where the land was located. At that time, he had no knowledge of the buyer’s or the bank’s rights. When he learned about them, he immediately brought a proceeding to foreclose his judgment lien, naming the tortfeasor, the buyer, and the bank as parties.

The jurisdiction has a typical grantor/grantee recording index, and has enacted the following statute:

“Any judgment properly filed in the county recorder’s office shall, for 10 years from filing, be a lien on the real property then owned or subsequently acquired by any person against whom the judgment is rendered. No conveyance or mortgage of real property shall be good against subsequent bona fide purchasers for value and without notice unless the same be recorded according to law.”

As between the victim and the bank, which party’s interest in the land will be given priority?

A The bank, because the bank recorded its mortgage before the victim recorded his judgment lien.

B The bank, because the victim is not protected by the recording statute.

C The victim, because the victim’s judgment was filed in the recorder’s office before the buyer’s deed was recorded.

D The victim, because the judgment lien extends to after-acquired property.

A

The victim will not likely prevail against the bank because a majority of courts hold that the judgment lienor is not protected by the recording statute.

If the statute here, which is a notice statute, were applicable to protect the victim, he would have priority over the bank because his judgment lien was recorded before the buyer’s deed was recorded.

Under this view, the bank’s mortgage would have been considered “wild” and would be deemed unrecorded because the preceding conveyance, the buyer’s deed, was actually unrecorded. A searcher in the public records would therefore have been unable to find the mortgage. Hence, if the statute were applicable to protect the victim, he would have priority over the bank.

However, most courts reason that either

(i) a judgment creditor is not a bona fide purchaser because he did not pay contemporaneous value for the judgment, or

(ii) the judgment attaches only to property “owned” by the debtor, and not to property previously conveyed away, even if that conveyance was not recorded.

Under the statute in the present question, a judgment does not attach until it is recorded.

Here, the victim’s judgment did not attach to the land until after the bank obtained a mortgage on it, and the recording statute does not change that result. The failure of the buyer to record, and the resultant treatment of the bank as unrecorded, is irrelevant. Thus, the bank’s mortgage is superior to the victim’s lien. (A) is wrong because it does not matter whether the bank’s mortgage was recorded, as against a subsequent judgment lien creditor. The judgment lien creditor is not protected by the recording statute, so the bank prevails even though its mortgage would be deemed unrecorded, as discussed above. (C) is wrong because, as discussed above, a majority of courts hold that the judgment lienor is not protected by the recording statute. (D) is wrong because the land was not after-acquired property, because the judgment lien was not filed until the tortfeasor had obtained—and conveyed away—an interest in the property. However, if the victim had in fact recorded his lien before the tortfeasor inherited the land, the after-acquired property provision of the statute would have applied, the victim would have had a recorded lien on the land as soon as the tortfeasor acquired it, and the victim would have gained priority over the bank.

178
Q

An investor owned two adjacent lots in a downtown area, one fronting directly on a public street and the other behind the first. The investor ran a small dry cleaning business on the lot next to the street, and had built a café on the rear lot. Because the rear lot had no access to any public street, the investor used the parking lot of the dry cleaning business, which extended from the street all the way back to the rear lot, for access to the café. The café was only open during the tourist season, from May through September. After several years, the investor sold the rear lot to a chef by a deed that granted an easement over the dry cleaning business’s parking lot, to be used as an accessway to the café for the chef and her customers. The chef promptly recorded the deed.

Two years later, in February, the investor sold the dry cleaning lot to a sub sandwich franchise. The deed did not mention the easement previously granted to the chef. The franchise immediately demolished the small dry cleaning building and constructed its own restaurant. An outdoor patio area completely blocked access to the chef’s rear lot. Because the chef’s café was closed for the winter, nothing came of the franchise’s construction until April, when the chef returned to open her café for the summer season. The shortest alternate route over other parcels from the chef’s lot to a public street would have to pass through several buildings and lots.

If the chef brings an action to compel the sub sandwich franchise to demolish the outdoor patio, how should the court rule?

A For the franchise, because its deed contained no mention of the access easement.

B For the franchise, because construction of the outdoor patio extinguished the chef’s rights to the access easement.

C For the chef, because ownership of the easement gives her the right to use it for access to her lot.

D For the chef, because she has no other access to her lot.

A

The court should rule for the chef because her easement gives her the right to use the sub sandwich franchise’s property for access to her lot. An easement is an interest in land that gives the holder a right to use the land for certain purposes. Here, the investor granted the chef an easement to use a portion of his property for access to her lot. The presumption when an easement is granted is that it is perpetual unless otherwise stated. Thus, the chef’s easement was perpetual and was not destroyed by the transfer of the servient tenement (the investor’s property) to the sub sandwich franchise. Moreover, the easement is valid against the franchise because it was recorded. Easements, like other interests in land, are good against subsequent holders of the burdened (servient) tenement as long as the interest is recorded. The facts here state that the chef properly recorded her easement, so it is good against the franchise; thus, (C) is correct.

179
Q

A landowner and his friend owned a tract of land as joint tenants with right of survivorship. The landowner executed a deed conveying his interest in the land to his grandson. The landowner gave the deed to his attorney with instructions to deliver it to the grandson upon the landowner’s death. The grandson first learned of the deed at the landowner’s funeral the following year. The next day, the grandson recorded the deed.

Who owns the land?

A The friend and the grandson, as joint tenants.

B The friend and the grandson, as tenants in common.

C The friend.

D The grandson.

A

The friend owns the land. A joint tenancy is a concurrent estate in land in which each co-tenant has an undivided right in the property and a right of survivorship—when one joint tenant dies, the property is freed of his concurrent interest and the survivor retains an undivided right in the property that is no longer subject to the interest of the deceased co-tenant.

An inter vivos conveyance by one joint tenant, even a “secret” deed that is to take effect only upon the grantor’s death, severs a joint tenancy. *However, although acceptance (presumed or otherwise) usually “relates back” to the date of delivery of the deed in escrow, many courts refuse to relate back an acceptance where it would defeat the rights of intervening third parties, such as surviving joint tenants.* Thus, the grantee’s acceptance of the deed after the grantor’s death does not relate back to defeat the right of survivorship. Here, the grandson did not accept the deed until after the landowner’s death. In the meantime and because of that death, the friend’s right to the whole property had accrued to her as the surviving joint tenant. Thus, the grandson has no interest in the land, and (A), (B), and (D) are incorrect. Note that if the landowner’s conveyance had been effective, (B) rather than (A) would have been the correct answer. Upon severance of a joint tenancy by inter vivos conveyance, the new tenant holds as a tenant in common with the remaining joint tenant, and not as a joint tenant.

180
Q

A landowner owned two adjoining parcels of land. The landowner sold the western parcel to a buyer by a deed that contained the following clause: “Grantee promises for himself, his heirs, successors, and assigns to not erect a structure over two stories on the land.” The buyer recorded the deed and built a two-story house on the property and lived there for 30 years, after which he sold the land to a movie star by a deed that did not contain the structure height restriction. The movie star decided to tear down the existing house on the land and erect a three-story house. Her neighbor, who purchased the eastern parcel from the landowner 10 years earlier, discovers that the movie star’s house will be larger than his and files suit to enforce the covenant.

Who will prevail?

A The neighbor, because the restrictive covenant runs with the land.

B The neighbor, because privity is not required to enforce an equitable servitude.

C The movie star, because her deed did not contain the restrictive covenant.

D The movie star, because she had no notice of the structure height restriction.

A

The neighbor will prevail because the restrictive covenant runs with the neighbor’s land, which is benefited, and with the movie star’s land, which is burdened. A covenant at law will run with the land and be enforceable against subsequent grantees if: (i) the contracting parties intended it to run; (ii) there is privity of estate between the original promisor and promisee (horizontal privity), as well as between the promisor and his successor (vertical privity); (iii) the covenant touches and concerns the property; and (iv) the burdened party has notice of the covenant. Here, the use of the words “heirs, successors, and assigns” in the covenant shows the intent for the covenant to run. The original parties were in horizontal privity because at the time the buyer entered into the covenant, he and the landowner shared an interest in the land independent of the covenant—as grantor and grantee. Because the neighbor and the movie star hold the entire interest in the parcels held by the landowner and the buyer, respectively, there is vertical privity. The covenant touches and concerns the land because it diminishes the movie star’s rights in connection with her enjoyment of the western parcel. Finally, the movie star has notice of the covenant because it was recorded in her chain of title (in the landowner-buyer deed). Thus, the covenant runs with the land.

(B) is incorrect because although privity is not required to enforce an equitable servitude, the covenant here runs with the land at law. Moreover, privity does exist under these facts, as explained above. (C) is incorrect. The fact that the structure height restriction was not contained in the movie star’s deed does not shield her from enforcement of the covenant because it was contained in a previous deed relating to the property (record notice). (D) is incorrect because the movie star had at least record notice of the restriction.

181
Q

A landlord entered into a written lease of a bakery for a term of 25 years with a baker. The parties agreed to a right of first refusal if the bakery was offered for sale during the term of the lease. The lease also permitted assignments and subleases on notice to the landlord. Three years later, the baker retired and, after notifying the landlord, transferred the lease to a chocolatier. Twenty-one years later, the landlord entered into a contract with a buyer for the sale of the bakery for $100,000. The landlord had informed the buyer of the lease but had forgotten about the right of first refusal. When the chocolatier learned of the sale to the buyer, she informed both the landlord and the buyer that she wanted to exercise her option and was prepared to purchase the bakery for the contract price. The jurisdiction’s Rule Against Perpetuities is unmodified by statute.

Can the chocolatier enforce the option?

A Yes, because an option held by a tenant on leased property cannot be separated from the leasehold interest.

B Yes, because the option touches and concerns the leasehold estate.

C No, because the transfer to the chocolatier made the option void under the Rule Against Perpetuities.

D No, because the option was not specifically included when the lease was transferred to the chocolatier.

A

The chocolatier can enforce the option to purchase because it is a covenant that runs with the land. When a tenant makes a complete transfer of the entire remaining term of his leasehold interest, it constitutes an assignment. The assignee and the landlord are then in privity of estate, and each is liable to the other on all covenants in the lease that run with the land. The covenant runs with the land if the original parties so intend and the covenant “touches and concerns” the leased land, i.e., burdens the landlord and benefits the tenant with respect to their interests in the property. Here, the transfer of the lease to the chocolatier was an assignment, making all covenants in the lease that run with the land enforceable by the assignee. The right of first refusal burdens the landlord’s power of alienation over the bakery, and there is nothing to indicate that the parties intended the option to be personal to the baker. Hence, the chocolatier can enforce the option and purchase the property. (A) is incorrect because most courts do not bar an option from being separated from the leasehold interest if that is the parties’ intent. The tenant may transfer the leasehold interest while retaining the option to purchase, or vice versa. Whether the option in this case stayed with the leasehold interest depends on whether it was a covenant that runs with the land. (C) is incorrect because options and rights of first refusal are not subject to the Rule Against Perpetuities when connected to leaseholds. If a tenant assigns the leasehold, the option generally is considered a running covenant, exercisable by the assignee absent a contrary intent. If the option had been separated from the leasehold estate, so that it was no longer exercisable by the tenant, the Rule would have become applicable to the option (and it would have invalidated the option here because it could have been exercised more than 21 years after a life in being). Here, however, the option was not severed from the leasehold; the entire interest was transferred to the chocolatier as the new tenant. Hence, the Rule Against Perpetuities is not applicable. (D) is incorrect because, as discussed above, the option is a covenant that runs with the land regardless of whether it was specified in the assignment to the chocolatier. The chocolatier, as the assignee of the leasehold, can enforce the option on privity of estate grounds

182
Q

A landowner’s will left his ranch to a rancher, his heirs, and assigns, so long as the property was used exclusively for ranch purposes, then to the landowner’s grandson. The remainder of the landowner’s property passed through the residuary clause of his will to the grandson. Seven years after the landowner’s death, the rancher began strip mining operations on the ranch. The grandson brought an action to quiet title to the ranch against the rancher, and the rancher counterclaimed on the same theory.

Who should prevail?

A The rancher, because the condition imposed on his interest under the will is void as violating the Rule Against Perpetuities.

B The rancher, because the condition imposed is a restraint against alienation.

C The grandson, pursuant to the residuary clause.

D The grandson, because the condition imposed is valid and he takes according to the subsequent provision.

A

Executory interest following defeasable fee

The grandson prevails because the ranch passed through the residuary clause. Under the Rule Against Perpetuities, the attempt to give the grandson an executory interest is void, so (D) is incorrect. However, the courts would simply read the conveyance without the language of the executory gift, leaving a possibility of reverter in the grantor, the landowner. Thus, (A) is incorrect. Because the grandson succeeded to the landowner’s interest as grantor via the residuary clause of the will, he will prevail. (B) is incorrect because there is no restraint on alienation contained in the will.

183
Q

A landowner owns 15 acres of undeveloped property. He plans to build a stadium complex on the property to house a football team two years from now, but would like to open the 15 acres to public use for picnicking and similar activities until then.

Which of the following would best accomplish the landowner’s goal?

A Dedicate the 15 acres for use as a public park.

B Lease the 15 acres to the city for two years.

C Grant the city an easement for public recreational uses for two years.

D Covenant that the city may use the 15 acres for recreation for two years.

A

The best way for the landowner to accomplish his goals is to grant the city an easement for recreational use for two years. An easement would allow the city to use the land only for the purposes provided for in the easement, and the landowner could limit the purposes to recreational uses. Thus, (C) is the best answer.

(A) would not be a good choice because if the landowner dedicated the land to public use, he would be giving title to the land to the government, so he would not be able to reclaim the land and build his stadium in the future.

(B) would not be a good choice because a lease would give the city more control over the land than an easement, and would be more complicated to create. A lease grants the lessee the exclusive right to possess the premises, and broad rights to use them in any manner, unless specifically restricted. Thus, if the landowner leases the land to the city, he would not have access to the land, and if he wanted it used only for recreational purposes, he would have to specifically restrict any undesired uses. Any restriction not included in the lease will be unenforceable. An easement, on the other hand, grants only a limited interest in the land—to use it for only those purposes stated in the easement; thus, it would be better than a lease. (D) is not a good choice because covenants usually are made in conjunction with a lease, deed, or other instrument; they promise some act or forbearance with respect to property and are generally not used to grant rights for access to property.

4 An owner of land gave his friend a deed for a specified parcel of property. After the owner’s death, the friend discovered that the owner had sold part of the property. The purchaser had been given an easement to cross over the owner’s property to get to the property she had purchased, although there is no evidence that the purchaser had ever used the “right of way.”

184
Q

The covenant of quiet enjoyment is breached when …

The covenant of right to convey is breached if …

A

the grantee is evicted by a third party with paramount title, i.e., title or ownership of the estate conveyed that is superior to the grantor’s title.

the grantor lacks title, i.e., ownership of the estate conveyed, at the time of the grant.

185
Q

Abatement

A

When a buyer has a remedy of specific performance in a land sale contract, a court of equity will order a seller to convey the title if the buyer tenders the purchase price. If the seller cannot provide marketable title under the terms of the contract, but the buyer wishes to proceed with the transaction, the buyer can usually get specific performance with an abatement of the purchase price in an amount reflecting the title defect. A defect as to the quantity of land conveyed is usually corrected by a pro rata abatement of the price.

186
Q

A property owner conveyed commercial property in joint tenancy to his two daughters as a birthday present. The deed from the property owner to his daughters was never recorded. After a few years, the property owner no longer wished the daughters to control valuable commercial property, and so he demanded that they return the deed with which he conveyed the property to them. The daughters returned the deed, and the property owner destroyed it. The property owner then sold and conveyed the property to a third party. The jurisdiction’s recording act states the following: “No conveyance or mortgage of an interest in land is valid against any subsequent purchaser for value without notice thereof whose conveyance is first recorded.”

If the third party brings a quiet title action and is successful, which of the following best explains this result?

A As owner of the property, the property owner was entitled to convey it to the third party.

B The daughters failed to record the deed they took from their father.

C The daughters failed to record their deed, and the third party was unaware of their interest when she paid the property owner market value for the property.

D The daughters failed to record their deed, the third party was unaware of their interest when she purchased the property, and the third party recorded her deed.

A

The third party’s success will be because the daughters failed to record their deed, the third party was unaware of their interest when she purchased the property, and the third party recorded her deed. When a deed is delivered, title passes to the grantee upon effective delivery. Therefore, returning the deed to the grantor has no effect; it constitutes neither a cancellation nor a reconveyance. A reconveyance requires the execution of a new deed. Thus, the property owner had no interest in the property that he purportedly conveyed to the third party, so (A) is wrong. However, if a grantee does not record her instrument, she may lose out against a subsequent bona fide purchaser. By recording, the grantee gives constructive (or “record”) notice to everyone. Hence, proper recording prevents anyone from becoming a subsequent BFP. There are three major types of recording acts, classified as “notice,” “race-notice,” and “race” statutes. Here, the statute is a race-notice statute because a subsequent BFP is protected only if she records before the prior grantee. The third party only wins if the daughters failed to record their deed, the third party was unaware of their interest when she bought the property, and the third party recorded her deed. (B) and (C) are incorrect because they do not contain every element necessary for the third party to prevail.

187
Q

A landowner conveyed his land “to my son for life, then to my son’s widow for her life, then to my son’s children.” At the time of the conveyance, the son was 20 years old and unmarried. The son eventually married and had two children, the landowner’s grandson and granddaughter.

Many years later, the landowner and the grandson were involved in a train accident. The landowner was killed instantly. The grandson died a short time later of his injuries. The landowner left his entire estate by will to his friend. The grandson’s will devised his entire estate to the city zoo. The son’s wife was so grief-stricken that she became ill and died the next year, leaving her entire estate to her husband.

Eventually the son met and married a 21-year-old. Ten years later, the son died, leaving everything to his second wife. When the second wife moved onto the land, the granddaughter filed suit to quiet title to the land, joining all of the appropriate parties.

If the jurisdiction recognizes the common law Rule Against Perpetuities, unmodified by statute, in whom will the court most likely find that title to the land is held?

A One-half in the granddaughter and one-half in the city zoo, subject to the second wife’s life estate.

B One-half in the granddaughter and one-half in the second wife, because the second wife took the son’s interest.

C Entirely in the friend, subject to the second wife’s life estate, because the gift to the son’s children violates the Rule Against Perpetuities.

D Entirely in the granddaughter, subject to the second wife’s life estate, because the grandson did not survive the son.

A

The granddaughter and the city zoo each own one-half of the land, subject to the second wife’s life estate.

At the time of the conveyance by the landowner, the son had a life estate, the son’s widow had a contingent interest (because the son’s “widow” cannot be ascertained until the son’s death), and the son’s children had a contingent remainder (because they have not yet been born). When the grandson and the granddaughter were born, however, their interests became vested subject to open (i.e., if the son had more children). Thus, when the grandson died, he had a vested remainder subject to open that he was free to devise by will; the city zoo took his vested remainder subject to open. At the son’s death, the class of his “children” closed (because the son could not have any more children), and the granddaughter’s and the zoo’s vested remainders subject to open became indefeasibly vested. Also at the son’s death, his widow was ascertained and her interest vested in possession. Because the second wife was the son’s widow, she is entitled to the valid life estate. Thus, the granddaughter and the city zoo hold one-half interests, subject to the second wife’s life estate.

(B) is wrong because the son had no interest in the land when he died. He merely had a life estate, which ended at his death. He did not inherit any interest in the property from anyone else. The only person he inherited from in these facts was his first wife, and she had no interest in the land. Furthermore, this choice overlooks the city zoo’s interest, which was inherited from the grandson. (C) is wrong because the son’s children’s interest does not violate the Rule Against Perpetuities. To be valid under the Rule, an interest must vest if at all within a life in being at its creation plus 21 years. The son is a life in being. At the son’s death, his children’s interest is certain to vest or fail: If the son had any children, at his death, the children’s interest would become indefeasibly vested (i.e., the class would close and the children’s interest would no longer be subject to open). Note that the children need not come into possession within the perpetuities period; the only requirement is that their interests vest within the period. Likewise, if the son had no children, the gift to them was certain to fail at his death. Thus, the children’s interest does not violate the Rule. Because the son had children and their interest was valid, there was no interest to revert to the landowner and to be devised to the friend. Note that the unborn widow aspect of this question is a red herring. The fact would be relevant only if the children’s gift were conditioned on their surviving the widow, in which case the takers would remain unascertained and their interest would remain contingent until that time. But because the children’s interest vested at the son’s death, it is irrelevant that the son’s “widow” was not a life in being at the creation of the interest. (D) is wrong because the gift to the son’s children was not conditioned on their survival of the son. The law does not imply such a condition. The grandson’s interest was vested subject to open and could be disposed of by his will.

188
Q

The insanity disability prevents the running of the statute of limitations for adverse possession if

A

the disability existed on the day the adverse possession began.

Because the landowner was insane when the trespasser began the period of adverse possession, the clock did not begin to run until the landowner was free of the disability 12 years later

189
Q

The uncle wants to evict the nephew where the nephew continues making payments and the uncle and the nephew entered into an oral agreement to deliver the deed to the nephew until the end of the five-year term.

A

Although this is the sale of land, which normally needs to be in writing to satisfy the Statute of Frauds, there is an exception for part performance.

An oral contract for the sale of land is valid with a showing of a combination of any, or all three, of the following: (1) payment of all or part of the purchase price; (2) taking of possession; and (3) making substantial improvements to the property.

Here, all three have happened. Therefore, the uncle cannot evict the nephew or treat him as though he were a tenant.

190
Q

If a senior mortgage is modified, a junior mortgage prevails over the modification but only if

A

the modification materially prejudices the holder of the junior mortgage, such as by:

(1) increasing the amount of principal; or
(2) increasing the interest rate (if the rate under the original mortgage was fixed).

Modifications that normally do not materially prejudice the holder of the junior mortgage include:

(1) extension of the mortgage maturity date; and
(2) rescheduling installment payments.

191
Q

The Statute of Frauds requires that contracts for the sale of land be in writing and signed by the party against whom enforcement is sought. There is an exception to this rule for part performance, which requires two or more of the following three things: (1) payment of part or all of the purchase price; (2) taking possession; or (3) making substantial improvements.

A

However, where the buyer made improvements and tendered payment at closing BUT the seller didn’t accept the check, contract for the sale of land won’t be enforced.

192
Q

The owner of the adjacent tract gained title to the land by adverse possession and sold the land to the developer. He did not provide marketable title to the developer. The owner cannot force the developer to purchase the property absent an action to quiet title, where a judge would determine title to the property. However, the developer can claim title via the purchase from the owner of the adjacent tract and then seek a judicial declaration as to the title to the property.

The developer will get the land even though the original owner, in his will, devised the land to his daughter, and even though the adverse possessor renaunciated his interest in the land.

A

The owner of the adjacent property acquired title through adverse possession, which cannot be turned over orally through a renunciation. He then sold that interest to the developer.

Because a will speaks at the death of the testator, the daughter has no interest.

193
Q

The landowner’s infancy, legal incompetency, or imprisonment can toll the statute of limitations for adverse possession. However, ….

A

the disability must be in existence at the time that the adverse possessor enters the property.

Awareness, with nothing else, is not sufficient to negate any of the elements for adverse possession. Something affirmative needs to be performed, such as a confrontation with the trespassers by the landowner.

There is no requirement that the trespassers know who the true owner is, just that they are aware that they are not the owner and otherwise satisfy the elements showing the world their intent to possess the property for the statutory period of time.

194
Q

Taking:

temporary restriction

regulation doesn’t advance legitimate state interest i.e. for mayor’s personal purposes

A

A temporary restriction causing a diminution in value (such as a 32-month restriction on development in an area) is not a taking of the parcel as a whole [Tahoe-Sierra Preservation Council, Inc. v. Tahoe Regional Planning Agency, 505 U.S. 1003 (2002)]. A court will also look at the dimensions of a property interest to determine whether a temporary restriction constitutes a taking.

The Supreme Court has held that whether a regulation advances a legitimate state interest is not a relevant factor in determining if a taking has occurred [Lingle v. Chevron U.S., Inc., 544 U.S. 528 (2005)].

195
Q

A friend taking property under estoppel by deed - BFP?

Son sold property to friend even though his father had the title and was still alive. After father’s death, the son sold the land to creditor.

A

The friend initially took by deed from the son, at a time when the son did not own the property.

Although the father’s subsequent death and the son’s acquisition of title via the will gave the friend title via estoppel by deed, the friend still cannot be a bona fide purchaser, because he would have had notice at the initial time of the conveyance that the father was still alive.

Additionally, the creditor cannot be a bona fide purchaser, either. Although the friend’s filing at the time of his initial deed from the son did not impose notice on the creditor (because it was ineffective given that the father was alive and held title at the time), when the creditor took his deed from the son, he would have had actual or inquiry notice by virtue of the fact that the friend was then physically on the property, having built a house on it and living there. Therefore, because neither the friend nor the creditor are bona fide purchasers, the rule reverts to the common law “first in time, first in right” rule, meaning friend has superior title.

196
Q

What reders title unmarketable - easement vs. mortgage?

A

All contracts for the sale of real property include an implied promise to convey marketable title. Marketable title is title that is reasonably free from doubt in both fact and law. An easement that reduces the value of the property renders title unmarketable.

A seller may satisfy a mortgage or lien at closing with the proceeds of the sale. If the purchase price is sufficient and satisfaction occurs simultaneously with the transfer of the land, the buyer may not complain because closing will result in marketable title.

197
Q

the Uniform Vendor and Purchaser Risk Act

A

The majority of jurisdictions have not adopted the Uniform Vendor and Purchaser Risk Act. In these majority jurisdictions, the risk of loss passes to the buyer when he has entered into a binding land sale contract. It is called equitable conversion, and it reflects the idea that the buyer has become the equitable owner of title upon execution of the valid contract. After the contract becomes binding, any loss on the property prior to the closing falls on the buyer and the seller will be granted specific performance, should he seek that equitable remedy.

198
Q

A judgment creditor is not considered a “subsequent purchaser for value” under a notice recording statute.

True or False?

A

A majority of jurisdictions interpreting notice statutes treat judgment creditors as protected in full (i.e., against all unrecorded interests subject to the recording act) only if they purchase the debtor’s property at a judicial sale. Mere entry of the judgment, creating a lien against the judgment debtor’s property, is not considered a “purchase for value” under the ordinary notice-type recording statute.

mere entry of the judgment, creating a lien against the judgment debtor’s property, is not considered a “purchase for value” under the ordinary notice-type recording statute

Yes, because the bank did not record the mortgage. This answer choice is wrong because the woman did not purchase the property at a judicial sale, so she is not considered to have “given value” for the property. As such, she would lose out in the priority to the bank’s mortgage, even though it was never recorded.

199
Q

A landowner owned a large tract of land in an area zoned for medium residential use. Permitted uses in this zone are single-family dwellings, condominium and townhouse developments, and moderate density apartment complexes. The landowner subdivided her land into 10 lots and conveyed each lot by a deed restricting the land to single-family use. All deeds were duly recorded and all lots were developed as single-family homes.

The owner of lot 1 died and his property passed by will to his niece. Some time later, the owner of lot 3 sold his property to a buyer by a deed that did not contain the covenant limiting use to single-family dwellings. The buyer subsequently sold lot 3 to her friend, and did not include the covenant in the deed. Both deeds were duly recorded. A storm destroyed the friend’s home, and in its place he began to build a three-unit townhouse.

May the niece sue to enforce the covenant against the friend to prevent him from building the townhouse on lot 3?

A No, because there is no privity with the friend.

B No, because the zoning laws have not been violated.

C No, because there was no restriction in the friend’s deed.

D Yes, because the friend’s townhouse would alter the landowner’s common scheme.

A

The niece may enforce the covenant as an equitable servitude against the friend because the friend’s townhouse would alter the common scheme the landowner created by using restrictive covenants in all of her deeds.

For successors of the original promisee and promisor to enforce an equitable servitude, both the benefit and burden of the servitude must run with the land. For the burden to run and thus bind the successor of the promisor (the friend):

(i) the covenanting parties must have intended that the servitude be enforceable by and against assignees;
(ii) the covenant must touch and concern the land; and
(iii) the party to be bound must have had actual, constructive (record), or inquiry notice.

The common scheme (all 10 lots were developed as single-family homes) is evidence that the original parties intended that the restriction be enforceable by assignees.

The covenant touches and concerns the friend’s property because it restricts him in his use of the property.

The friend had constructive and inquiry notice because the restriction is in his chain of title.

Therefore, the burden of the servitude runs with the land. The next issue is whether the benefit of the servitude runs with the niece’s land. For a benefit to run, it must be so intended by the original parties and the covenant must touch and concern the land. As noted above, intent may be inferred from the common scheme. The benefit touches and concerns the niece’s land because it benefits her in her use and enjoyment of the lot. Thus, the niece may enforce the covenant.

200
Q

Shortly before their wedding, a man and a woman bought a tract of land, taking title in both names. They had intended to build a summer cottage there, but many years after their marriage the land was still a vacant lot. The man decided that their introverted son would have more confidence if he were a landowner; thus, the man drew up a deed conveying a one-quarter interest in the land to him. Not wanting to show favoritism, two weeks later the man drew up a deed conveying a one-quarter interest in the same land to their daughter.

Who owns the land?

A The man and woman share ownership of the land with rights of survivorship, and the son and daughter have no interests.

B The son has a one-quarter interest, the daughter has a one-quarter interest, and the woman has a one-half interest.

C The son has a one-quarter interest, the daughter has a one-quarter interest, and the woman has a one-half interest, with rights of survivorship.

D The woman owns the land.

A

The son has a one-quarter interest, the daughter has a one-quarter interest, and the woman has a one-half interest. There is a presumption that the man and woman are tenants in common. For a joint tenancy to exist, there must be an express creation of such tenancy; thus, there is a presumption of tenancy in common unless the conveyance is to a husband and wife in a state that recognizes tenancy by the entirety. Here, the man and woman were not married when they took title to the land. Each tenant in common has an undivided interest, which may be conveyed by inter vivos transfer. The man started with an undivided one-half interest, one-half of which he conveyed to his son and the other half of which he conveyed to his daughter. The man has thus conveyed all of his interest in the land, and so (A) is incorrect. There is no right of survivorship in a tenancy in common; therefore, (C) is incorrect. (D) is not supported by the facts.

201
Q

An owner of a parcel of land instructed his lawyer to draw up an instrument deeding the land to his friend’s “nieces.” The owner acknowledged the deed before a notary and signed it. As directed by the owner, the lawyer recorded the deed and then returned it to the owner. The owner put the deed in the drawer of his desk, intending to present it to the friend’s nieces when they came to visit him next month.

The following week, however, the owner died, leaving his daughter as his sole heir at law. The daughter discovered the deed to the land in the owner’s desk. She filed an appropriate action to quiet title in the land, naming the friend’s only two nieces as defendants. The only evidence presented at the trial was the deed itself, the evidence of recordation, and the lawyer’s testimony regarding the owner’s intent.

Who should the court rule owns the land?

A The nieces, because recordation of a notarized deed is prima facie evidence of delivery.

B The nieces, because a deed is prima facie valid absent evidence to the contrary.

C The daughter, because the evidence is insufficient to support a valid delivery.

D The daughter, because the grantees in the deed are too indefinite.

A

The nieces own the land because recordation is prima facie evidence of delivery.

To be valid, a deed must be “delivered,” which means that the grantor must have taken some action (not necessarily a manual handing over of the deed) with the intent that it operate to pass title immediately. Recording a deed that has been acknowledged before a notary is such an action and is presumed to carry with it the requisite intent. Even without the knowledge of the grantee, delivery to the recorder’s office will satisfy the delivery requirement. If the grantor intends the recording of the document to be the final act in vesting title in the grantee, then such recording constitutes delivery. (B) is wrong because a deed alone is not prima facie valid absent delivery. There must be evidence of delivery. (C) is wrong because recordation can constitute valid delivery, and there is sufficient evidence that the deed was recorded. Note that a rebuttable presumption of no delivery may arise from the grantor’s retention of the deed. However, this presumption is rebutted by the recording of the deed. (D) is wrong because a description of the grantees in a deed is sufficient if it describes the grantees with sufficient particularity that it can be determined who is to take the property. The grantee need not actually be named. Because the friend has a finite number of nieces and they are easy to locate and identify, the deed from the owner satisfies this requirement.

202
Q

An entrepreneur opened a specialized business on her land. After using up most of her capital to purchase inventory, however, the entrepreneur needed more funds and asked her friend for a $30,000 loan, to be secured by the business’s inventory. The friend declined the loan. A desperate entrepreneur then told the friend she would convey the land, which had a fair market value of $100,000, to him if he would give her the loan at the current market rate of interest. The friend agreed, and the entrepreneur conveyed the land to the friend the next day. At that time, the friend gave the entrepreneur $30,000 in cash, and the parties orally agreed that the entrepreneur would pay the friend back at the rate of $1,000 per month, and that after the loan was paid in full, the friend would reconvey the land to the entrepreneur. The friend immediately recorded his deed to the land.

The entrepreneur made three $1,000 payments to the friend and then paid no more. She continued to live on the land but, being very much in debt, could not repay the loan. The friend, meanwhile, had received an offer to buy the land for $100,000.

Which of the following most accurately states the friend’s right to sell the property?

A The friend may sell the land and keep the entire proceeds.

B The friend may sell the land, but he must give $73,000 of the proceeds to the entrepreneur.

C The friend may sell the land only after formally foreclosing on the property.

D The friend may not sell the land.

A

The friend may sell the land, but only after formally foreclosing on the property. If a deed is given for security purposes rather than as an outright transfer of the property, it will be treated as an “equitable” mortgage and the creditor will be required to foreclose it by judicial action like any other mortgage.

In determining whether an absolute deed is really a mortgage, the court considers the following factors:

(i) the existence of a debt or promise of payment by the deed’s grantor;
(ii) the grantee’s promise to return the land if the debt is paid;
(iii) the fact that the amount advanced to the grantor/debtor was much lower than the value of the property;
(iv) the degree of the grantor’s financial distress; and
(v) the parties’ prior negotiations.

Here, the entrepreneur owed the friend a debt; the friend promised to return the property if the debt was paid; the amount advanced ($30,000) was much lower than the value of the property ($100,000); the entrepreneur was in great financial distress; and the parties’ negotiations reveal that this transaction was intended as security for the loan. Thus, the friend must bring a judicial foreclosure proceeding before he can sell the land.

203
Q

A property owner owned a tract of commercial property that he conveyed in joint tenancy to his twin sons as a birthday present. Unfortunately, a few years after the conveyance, the property owner and his sons had a serious falling out over how to run the family business. The property owner no longer wished the sons to control valuable commercial property, and so he demanded that they return the deed with which he conveyed the property to them. The sons returned the deed, and the property owner destroyed it. A few months later, one of the twins learned that he was seriously ill and not likely to live much longer. He executed a quitclaim deed conveying “any interest I have in the commercial property conveyed to me and my brother from my father” to his daughter. The twin who conveyed the property subsequently died.

Who owns the property?

A The living twin.

B The property owner.

C The living twin and the deceased twin’s daughter as tenants in common.

D The living twin and the deceased twin’s daughter as joint tenants.

A

The living twin and the deceased twin’s daughter own the property as tenants in common. A conveyance of a co-tenant’s interest in joint tenancy property severs the joint tenancy, and that interest is subsequently held as a tenancy in common with the other co-tenants. Thus, (D) is incorrect. Because the joint tenancy with right of survivorship was severed before the deceased twin’s death, (A) is incorrect. Returning the deed to the property owner did not return ownership of the property to him; that would require a reconveyance. Thus, (B) is incorrect.

204
Q

An investor owned a 100-acre parcel that contained several natural asphalt lakes. A construction company was erecting highways for the state in the vicinity of the investor’s land and needed a supply of asphalt. The investor executed a document that, in return for a payment of $1 per barrel, gave the company the right to enter on the land and take asphalt in whatever quantities the company desired. The investor reserved the right to remove asphalt herself and to grant this right to others. Last year, the state commenced an action in eminent domain to take the investor’s land for a public park.

Is the construction company entitled to compensation?

A No, because the nonexclusive nature of the company’s right makes it a license, which is not an interest in property.

B No, because a nonexclusive profit, although an interest in property, has no value separate and apart from the land itself.

C Yes, because the company has a nonexclusive profit, which is a property right for which it is entitled to compensation.

D Yes, because the company has a license coupled with an interest, which is a property right for which it is entitled to compensation.

A

The construction company is entitled to compensation because it has a property right to enter and remove minerals.

Like an easement, a profit is a nonpossessory interest in land.

The holder of the profit is entitled to enter on the servient tenement and take the soil or the substance of the soil (e.g., minerals, timber, oil, or game).

When an owner grants the sole right to take a resource from her land, the grantee takes an exclusive profit and is solely entitled to the resources, even to the exclusion of the owner of the servient estate.

By contrast, when a profit is nonexclusive, the owner of the servient estate may grant similar rights to others or take the resources herself.

Although here the profit is nonexclusive, it is nevertheless an interest in property for which the company is entitled to compensation in any condemnation proceeding.

(A) is incorrect because a license is merely revocable permission to enter on another’s land. Unlike a profit, a license is not an interest in land; it is merely a privilege, ordinarily terminable at the will of the licensor. (B) is incorrect because a profit is the right to take something from another person’s land; it has a value apart from the land itself and is alienable. (D) is incorrect because a license coupled with an interest has the effect of making the license irrevocable, but it does not convert the license into an interest in land for which compensation is required.

205
Q

A landlord who owned a strip mall entered into a written five-year lease of one of the units with a discount retail perfumery. The lease provided for a monthly rent of $1,000, payable on or before the first day of each month. The perfumery dutifully paid its rent on time for two years and three months. At that time, with the oral permission of the landlord, the perfumery transferred its interest in the remainder of the lease to a dry cleaner in writing, and added a clause requiring the dry cleaner to get permission from the perfumery for any subsequent assignments. The dry cleaner promptly paid rent to the landlord for 14 months, and then asked the landlord to approve a transfer of its interest in the lease to a video rental store. The landlord gave her oral assent. To obtain the perfumery’s approval of the transfer to the video store, the dry cleaner wrote a letter to the perfumery, promising that if any problems arose and anyone tried to go after the perfumery for money, the dry cleaner would “make it good.”

After the perfumery sent a letter back to the dry cleaner agreeing to the transfer, the dry cleaner executed a written transfer of its interest to the video store. The video store promptly paid rent for three months. Having failed to make any profits, the video store ceased paying any rent to the landlord and cannot be located. The landlord has been unable to find anyone interested in the unit.

Given that any judgment against the video store would be worthless, from whom can the landlord collect the unpaid rent owed on the lease?

A Either the perfumery or the dry cleaner.

B The perfumery only, but the perfumery may recover in turn from the dry cleaner.

C The perfumery only, and the perfumery has no recourse against the dry cleaner.

D Neither the perfumery nor the dry cleaner.

A

The landlord may collect the unpaid rent from either the perfumery or the dry cleaner.!!!!!!!

A complete transfer of the tenant’s entire remaining term is an assignment of the lease. However, the original tenant can still be held liable on his original contractual obligation in the lease to pay rent; i.e., on privity of contract.

(D) is therefore incorrect because the perfumery is liable for the rent.

(B) and (C) are also incorrect. Because the covenant to pay rent touches and concerns, and hence runs with the tenant’s leasehold estate, an assignee owes the rent directly to the landlord. If the assignee reassigns the leasehold interest, his privity of estate with the landlord ends, and he generally is not liable for the subsequent assignee’s failure to pay rent in the absence of a specific promise to the landlord.

However, even if the assignee made no promise to the landlord but did promise the original tenant that he would pay all future rent, the landlord may sue the assignee as a third-party beneficiary of the promise to the original tenant. !!!!!!!!!

Here, while the dry cleaner made no promise to the landlord, the dry cleaner did make a promise to the perfumery regarding the obligation that the perfumery owed to the landlord. Thus, the landlord can sue either the perfumery or the dry cleaner for the unpaid rent.

206
Q

An uncle validly executed and notarized a deed conveying his beach house to his nephew, and then validly recorded the deed. When the nephew, who was experiencing financial difficulties, learned of the recordation of the deed, he immediately told his uncle that he did not want the beach house and could not accept such an expensive gift anyway. Later, the nephew filed for bankruptcy and the trustee in bankruptcy asserted an ownership interest in the beach house on behalf of the debtor’s estate. The bankruptcy court ruled that the property belonged to the uncle and not to the nephew, and thus was not part of the debtor’s estate subject to distribution.

Which of the following is the strongest reason in support of the bankruptcy court’s ruling?

A There was no presumption of delivery created by recordation of the deed because the nephew did not know of the recordation.

B The nephew’s statements to the uncle were a constructive reconveyance of the property.

C There was never an effective acceptance of delivery of the deed by the nephew.

D The recordation of the deed was invalid because it was done without the nephew’s permission.

A

The nephew’s express rejection of the deed was sufficient to rebut any presumption of acceptance. As a general rule, delivery of the deed is the final operative act to complete a conveyance of title to the grantee, because courts will infer the grantee’s acceptance if the conveyance is beneficial to him. However, all courts will consider evidence that is contrary to the presumption or inference. Hence, the nephew’s express rejection of the gift is sufficient to establish that no conveyance of the property took place.

(A) is an incorrect statement of law. If the grantor intends the recording of the deed to be the final act in vesting title in the grantee, then such recording creates a presumption of delivery even where the grantee did not know of the recordation. (B) is wrong because there is no such thing as a constructive reconveyance. Had the nephew accepted the gift (completing the conveyance) and later changed his mind, the nephew would have had to execute a new deed to convey the property back to the uncle. (D) is wrong because knowledge or permission of the grantee has no effect on the validity of the recordation; rather, it determines whether there has been an effective acceptance.

207
Q

An inability to establish a record chain of title will generally …

A

render the title unmarketable.

If the buyer determines, prior to closing, that the seller’s title is unmarketable, he must notify the seller and allow a reasonable time to cure the defect. If the seller is unable to acquire title before closing, so that title remains unmarketable, the buyer can rescind, sue for damages caused by breach, or obtain specific performance with an abatement of the purchase price.

208
Q

A landowner validly conveyed a parcel of land to a veterinarian “for so long as the property is used as a veterinary practice, but if the property is used for any other purpose, it is to go to the American Cancer Society.” Two years later, the landowner died, validly devising all of his property to his friend. The landowner’s only heir is his daughter. Although this jurisdiction is a common law jurisdiction with respect to all real property considerations, the state’s probate laws provide that future interests or estates in real property may be passed by will or descent in the same manner as present or possessory interests.

Last month, the veterinarian approached the daughter and asked her to join with him to sell the parcel of land, which he had been using as an animal shelter, in fee simple absolute to a developer. The veterinarian and the daughter entered into a contract of sale with the developer. However, after consultation with an attorney, the veterinarian decided against the sale. The developer sued the daughter and the veterinarian for specific performance.

Will the requested relief likely be granted?

A No, because the American Cancer Society did not join in the contract of sale.

B No, because the friend did not join in the contract of sale.

C Yes, because the veterinarian had the power to sell his interest.

D Yes, because together, the daughter’s and the veterinarian’s interests would merge and they would have a fee simple estate.

A

Executory interest following defeasable fee is void under the Rule Against Perpetuities

The requested relief will be denied because the friend did not join in the contract of sale. The conveyance purported to create a fee simple determinable in the veterinarian subject to an executory interest in the American Cancer Society.

A fee simple determinable subject to an executory interest is an estate that, on the happening of a stated event, is automatically divested in favor of a third person, who holds the executory interest. However, the executory interest in the American Cancer Society is void under the Rule Against Perpetuities because it might vest beyond lives in being plus 21 years. *The charity-to-charity exception to the Rule does not apply because the veterinarian is not a charitable organization.* Because any interest that violates the Rule is void and stricken from the instrument, what is left is a fee simple determinable in the veterinarian and a possibility of reverter in the landowner. On the landowner’s death, the possibility of reverter passed to the friend.

(A) is incorrect because the American Cancer Society does not have any interest in the land. (C) is incorrect because, although the veterinarian had the power to sell his interest, he did not own a fee simple absolute. (D) is incorrect because the daughter does not have any interest in the land. Even if she did, the merger doctrine—which provides that whenever the same person acquires all of the existing interests in land, present and future, a merger occurs and that person holds a fee simple absolute—would not apply because the friend also has an interest in the land.

209
Q

Title may be unmarketable where the owners of the present and future interests attempt to convey a fee simple absolute title if the future interests are held by …

A

persons who are unborn or unascertainable.

i.e.

A landowner executed a will, devising a parcel of land “to my sister for life, then to my brother for life, then to my nieces and nephews.” When the landowner died, he was survived by the sister and the brother’s son.

The sister has a life estate, and the brother’s son has a vested remainder subject to open because there may be other nieces and nephews born during the sister’s life who become entitled to share in the remainder.

Life estates and vested remainders are freely transferable, which means that the sister and the brother’s son together can transfer the land to a purchaser, but the title is not marketable. It may turn out that the sister has a child, who is entitled to share in the remainder, but who did not join in the conveyance to the buyer. Because the child would not be bound by the conveyance, he would own an interest in the land. provide marketable title.

210
Q

A developer owned a large urban property, which she subdivided into 10 lots. The developer conveyed Lot 1 to an architect by a deed that contained a restriction banning commercial use of the property. The developer subsequently conveyed Lots 2 through 7 to six separate purchasers. Each of the deeds to these purchasers also contained the restriction on commercial use. The architect left Lot 1 undeveloped, but the purchasers of Lots 2 through 7 all used their lots for commercial purposes. The developer subsequently conveyed Lot 8 to a florist. The florist’s deed contained the restriction banning commercial use of the lot, but he decided that he wished to use Lot 8 commercially. The developer retains ownership of Lots 9 and 10. The florist wants to bring suit to establish his rights to use Lot 8 for commercial purposes.

Which of the following best describes the parties the florist should join in his lawsuit?

A The developer only.

B The developer and the architect only.

C The other commercial users only.

D All landowners in the subdivision.

A

The florist should join all of the landowners in the subdivision in a suit to terminate the servitude on the grounds of abandonment.

If a covenant in a subdivision deed is silent as to who holds its benefit, any neighbor in the subdivision will be entitled to enforce the covenant if a general scheme or plan is found to have existed at the time she purchased her lot.

In addition, a prior purchaser can enforce a restriction in a subsequent deed from a common grantor under either a third-party beneficiary theory or an implied reciprocal servitude theory.

Under the implied reciprocal servitude theory, an implied reciprocal servitude attaches to the grantor’s retained land at the moment she deeds a lot with the restriction. Thus, all of the other landowners in the subdivision could potentially enforce the covenant as an equitable servitude against the florist.

All parties would probably fail in an attempt to enforce the servitude, but the florist should join them now to avoid multiple litigation.

(Note that had the other landowners tried to enforce the equitable servitude against the florist, they would all have been subject to the equitable defense of acquiescence, which provides that if a benefited party acquiesces in a violation of the servitude by one burdened party, he may be deemed to have abandoned the servitude as to other burdened parties. In addition, the other commercial users are subject to the defense of unclean hands. It is important to remember that these are defenses and do not terminate the servitude; therefore, it would be best for the florist to join all possible complainants in a suit to have the servitude declared extinguished.)

(A) is wrong because, as explained, the other landowners also could try to enforce the covenant. (B) is wrong for the same reason. Although the architect has not violated the covenant and thus is not subject to the defenses possible against the other landowners, he and the developer are not the only possible plaintiffs (remember the question in effect asks who can bring suit, not who can win it). (C) is wrong because the developer and the architect can also bring suit. (Note that the above discussion applies only to sparing the florist from the enforcement of the restriction as an equitable servitude. The developer may try to enforce the restriction as a real covenant. She will, however, be limited to recovering damages, which might be very difficult to prove under the circumstances.)

211
Q

Where an installment land contract is used, the seller’s obligation is to furnish marketable title when …

A

… delivery is to occur, e.g., when the buyer has made his final payment.

Thus, a buyer cannot withhold payments or seek other remedies on grounds that the seller’s title is unmarketable prior to the date of promised delivery.

212
Q

To create an easement by express grant, there must be a writing signed by the grantor. If validly created, an easement is presumed to be of perpetual duration unless …

A

END CRAMP

213
Q

A grantor executed a valid deed conveying a tract of land to a city “for the purpose of constructing a planetarium thereon.” The city held the property for a number of years, but decided on another site for the planetarium. When presented an offer to purchase the property by a privately owned garbage collection company, the city accepted and conveyed the land to the company.

Which of the following statements about the title of the tract of land is true?

A The grantor’s conveyance to the city created a fee simple determinable in the city and a possibility of reverter in the grantor.

B Upon conveyance of the land to the company, the property reverted back to the grantor.

C The company owns the land in fee simple absolute.

D The company owns the land, but it will revert to the grantor or his successors in interest if the property is used for anything other than a planetarium.

A

The garbage collection company owns the tract of land in fee simple absolute because the city had a fee simple absolute, which it conveyed to the company.

The language in the deed “for the purpose of constructing a planetarium” merely expresses the grantor’s motive for conveying the property; the city received the estate that the grantor had, a fee simple absolute. Because the city held a fee simple absolute, that is what it conveyed to the company. (A) is wrong because the grant does not create a fee simple determinable. A fee simple determinable is an estate that automatically terminates on the happening of a stated event. To create a fee simple determinable, durational language (e.g., “for so long as,” “until”) must be used. Here, the grant does not contain the durational language necessary to create a fee simple determinable. Because the interest is not a fee simple determinable, the grantor cannot have a possibility of reverter. (B) is wrong for two reasons: (i) as explained above, a fee simple determinable was not created by the grant; and (ii) even if a fee simple determinable had been created, the transfer of the property would not by itself cause it to revert back to the grantor. Determinable estates are alienable; the successor merely takes subject to the condition. The conveyance of a fee simple determinable would not automatically result in the property reverting to the grantor (i.e., the company could build a planetarium on the property and avoid the property reverting back to the grantor). (D) is wrong because the grant did not create a fee simple determinable. (Had the grant contained the proper durational language, rather than “for the purpose of,” (D) would have been correct. In that case, the company would own the land subject to the estate being terminated if the land is not used for a planetarium.)

214
Q

The owner of a hotel in a resort town was approached by a seminar speaker who wanted to lease space in which to conduct a two-week seminar. The owner leased to the speaker the hotel’s grand ballroom, the period of the lease being August 1 through August 14. To provide the proper atmosphere for the seminars, the speaker attached curtain rods to the walls of the ballroom, using lightweight screws to attach the rods. The speaker then strung light blue ring curtains through the rods. After the seminar, on August 16, the speaker arrived to remove the curtains and rods. The owner brought an action to enjoin the speaker from removing the curtains and the rods from the grand ballroom.

How should the court rule?

A In favor of the speaker, because he had a short-term lease and the curtains and rods were easily removable.

B In favor of the speaker, because curtains and rods are trade fixtures.

C In favor of the owner, because the curtain rods were attached by screws, and as such were fixtures, which became part of the realty.

D In favor of the owner, because the speaker did not remove the curtains and rods before the lease expired.

A

The court should rule in the owner’s favor.

A tenant must remove annexed chattels before the termination of the tenancy or they become the property of the landlord. Although the seminar speaker was probably entitled to remove the curtains and rods at the end of the lease, he forfeited them by waiting for two days after the lease expired to remove them.

(A) is wrong because it goes to whether the curtains and rods were intended to be fixtures. Because of the delay in their removal, whether the curtains or rods were fixtures is irrelevant. This choice would be correct, however, had the speaker attempted to remove the curtains on August 14. T_he short-term lease and the fact that the rods are easily removable constitute evidence that the speaker lacked the requisite intent to permanently improve the property, and thus he could have removed them if he had acted promptly._

(B) is wrong for the same reason. The delay in the removal of the items results in their becoming the property of the landlord regardless of whether they are trade fixtures.

Trade fixtures (i.e., fixtures installed for the purpose of carrying on a trade or business) are removable prior to the end of the lease term.

Thus, because the speaker installed the curtains to carry on his business, this would have been a correct choice had the speaker attempted to remove the curtains prior to the end of his lease term.

(C) is wrong because the mere fact that the curtain rods were attached by screws does not make them fixtures that must remain with the realty.

“Fixtures” are chattels affixed to the land that become part of the land. The intent of the person affixing the chattel is relevant. The curtains and rods would probably not be considered fixtures because the speaker did not have the requisite intent to permanently improve the property, as evidenced by the short-term lease and the easily removable nature of the attached chattels. In the absence of an express agreement to the contrary, if removal of the chattel does not leave unrepaired damage to the premises or cause destruction of the chattel, the tenant has not manifested an intention to permanently improve the property.

Here, removing the screws, rods, and curtains would not result in substantial damage to the premises or destruction of the chattels. Also, even if the curtains and rods were found to be fixtures, they would be trade fixtures, which are removable by the tenant.a

215
Q

A cyclist was injured when a driver ran a red light. The cyclist subsequently sued the driver to recover for her injuries, and obtained a money judgment of $50,000. The state where the cyclist and the driver reside has the following statute: “Any judgment properly filed shall, for 10 years from filing, be a lien on the real property then owned or subsequently acquired by any person against whom the judgment is rendered.”

The cyclist filed the judgment in the county where the driver owned a valuable ranch. Sometime later, the driver, who was also injured in the accident, undertook to remodel all the buildings on the ranch to make them wheelchair-accessible. The driver borrowed $30,000 from a bank for the improvements, securing the loan with a mortgage on the ranch. The bank properly recorded its mortgage. Before he paid any principal on the bank’s loan, the driver decided to build a new barn. He borrowed $20,000 from a financing company for this purpose, also secured by a mortgage on the ranch. The financing company properly recorded its mortgage.

The driver subsequently defaulted on the bank’s mortgage, and the bank brought a foreclosure action, joining the financing company in the proceeding. The foreclosure sale resulted in $90,000 in proceeds after all expenses and fees were paid. The driver still owes the cyclist $50,000, the bank $30,000, and the financing company $20,000.

How should the foreclosure proceeds be distributed?

A The cyclist is entitled to $50,000, the bank is entitled to $30,000, and the financing company is entitled to the remaining $10,000.

B The cyclist is entitled to $50,000, the bank is entitled to $30,000, and the driver is entitled to the remaining $10,000.

C The bank is entitled to $30,000, the financing company is entitled to $20,000, and the driver is entitled to the remaining $40,000.

D The bank is entitled to $30,000, and the driver is entitled to the remaining $60,000.

A

The bank is entitled to $30,000 of the foreclosure proceeds, the financing company is entitled to $20,000 of the proceeds, and the driver is entitled to the $40,000 balance.

When an interest is foreclosed, after the expenses and fees are paid, the proceeds of the sale are first used to pay the principal and accrued interest on the loan that was foreclosed, next to pay off any junior liens, and finally any remaining proceeds are distributed to the mortgagor.

Here, there are enough proceeds to satisfy the bank’s (the foreclosing party’s) $30,000 mortgage and the financing company’s (the junior lienor’s) $20,000 mortgage. The remaining balance ($40,000) is distributed to the driver (the mortgagor).

(A) and (B) are wrong because the cyclist’s interest, an interest senior to the bank’s, is not affected by the foreclosure. Although foreclosure destroys all interests junior to the mortgage being foreclosed, it does not affect any senior interests. The buyer at the foreclosure sale takes subject to such interests. Without the cyclist foreclosing her lien, she is not entitled to a share of the proceeds, and her lien continues on the property in the buyer’s hands. (B) and (D) are wrong because the financing company is entitled to have its mortgage fully discharged.

216
Q

Two partners bought a commercial building from an owner. They paid cash for the building and took title as joint tenants with right of survivorship. Several years later, the first partner executed a mortgage on the building to secure a personal loan to a bank. The second partner had no knowledge of the mortgage to the bank. The state in which the commercial building is located recognizes the lien theory of mortgages. The first partner died before paying off his loan. He left all of his property by will to his daughter, his only heir.

Who has title to the commercial building?

A

The second partner has title free and clear of the mortgage. When the partners bought the property, they took title as joint tenants with right of survivorship. If the joint tenancy continued until the first partner’s death, then the property would pass immediately on death to the second partner. Because the second partner did not sign the mortgage, she would not be subject to it, regardless of whether she knew about it. The key to answering this question is to know whether execution of the mortgage by the first partner caused a severance of the joint tenancy. If it did cause a severance, then the first partner’s one-half would not pass to the second under right of survivorship but instead would pass to the first’s estate, and thus would go to the daughter by will. Whether a mortgage creates a severance or not depends on whether the state follows the lien theory or the title theory of mortgages. Lien theory means no severance; title theory means severance. Because this is a lien theory state (majority rule on the MBE), there was no severance; thus, the joint tenancy remained intact. On the first partner’s death, the joint tenancy ended and the first partner’s interest instantly passed to the second partner. The first partner’s estate got nothing; hence, the daughter could get nothing.

217
Q

A landowner conveyed her parcel of land to “my brother and my sister jointly, with right of survivorship.” Shortly thereafter, the brother was in an automobile accident. The driver of the other vehicle sued the brother on a theory of negligence, and obtained a judgment in the amount of $250,000. Because the brother did not have insurance or enough cash to satisfy the judgment, the driver levied on the brother’s interest in the land.

What interest will the driver most likely take?

A

The driver will get an undivided one-half interest in the land regardless of the status of the brother and the sister’s title.

A joint tenancy is a concurrent estate with a right of survivorship, while a tenancy in common does not have a right of survivorship. At common law, the conveyance here would qualify as a joint tenancy because the unities of time, title, interest, and possession are present in the conveyance. Although under modern law a joint tenancy must be created with specific language or else it will be presumed to be a tenancy in common, the conveyance here still would probably qualify as a joint tenancy, even though it did not use the words “joint tenancy,” because it contained the “right of survivorship” language. However, regardless of whether the estate is characterized as a joint tenancy or a tenancy in common, one tenant’s interest may be transferred without the consent of the other tenant, and a creditor may levy on the interest. In most jurisdictions, a lien against one joint tenant’s interest does not sever the joint tenancy until the lien holder proceeds to enforce it by foreclosure. At that point, the purchaser at the foreclosure sale will hold the property as a tenant in common with the other tenant, but will still have an undivided one-half interest in the property unless and until he brings an action to partition the estate.

218
Q

In which of the following situations must the tenant continue to pay a portion of the rent?

  1. If a paramount title holder takes possession of an unused barn on the leased premises and stores farm equipment in it,
  2. If the landlord or a paramount title holder obtains a judgment in an ejectment action against the tenant,
A
  1. If a paramount title holder takes possession of an unused barn on the leased premises and stores farm equipment in it,

the tenant must continue to pay a portion of the rent. Every lease contains an implied covenant that neither the landlord nor someone with paramount title will interfere with the tenant’s quiet enjoyment and possession of the premises. This covenant is breached by the tenant’s total or partial actual eviction from the leased premises. Total actual eviction occurs when the landlord or a paramount title holder excludes the tenant from the entire leased premises. This terminates the tenant’s obligation to pay rent. Partial actual eviction occurs when the tenant is excluded from only part of the leased premises. Partial eviction by the landlord relieves the tenant of the obligation to pay rent for the entire premises, even though the tenant continues in possession of the remainder of the premises. Partial eviction by a paramount title holder results in an apportionment of rent; i.e., the tenant is liable for the reasonable rental value of the portion that he continues to possess. A paramount title holder’s taking possession of an unused barn constitutes partial actual eviction. Thus, rent will be apportioned.

  1. If the landlord or a paramount title holder obtains a judgment in an ejectment action against the tenant, the total actual eviction terminates the tenant’s obligation to pay rent.
219
Q

A landlord rented an art studio to an artist. Under the terms of the signed, written, two-year lease, the artist agreed to pay the landlord $1,000 per month and to assume responsibility for all necessary repairs. After the first year of the lease, the artist assigned the balance of his lease to a sculptor. The landlord approved the sculptor as a tenant and accepted two rent payments from her, and then the landlord sold the building to an investor. The sculptor had made two payments to the investor when an electrical fire broke out in the studio, injuring the sculptor. The fire was caused by faulty wiring. The landlord was aware that there was a dangerous wiring problem when he leased the property to the artist. But when the landlord discovered how costly repairs would be, he decided it would be more profitable to sell the property than to repair it. The problem was not easily discoverable by anyone other than an expert electrician, and the landlord did not tell the artist, the sculptor, or the investor about the problem. The sculptor sues to recover damages for her injuries.

From whom can the sculptor recover?

A

The landlord is liable for the sculptor’s injuries because he failed to disclose a latent defect. If, at the time the lease is entered into, the landlord knows of a dangerous condition that the tenant could not discover upon reasonable inspection, the landlord has a duty to disclose the dangerous condition. Failure to disclose the information about the condition results in liability for any injury resulting from the condition. Because the landlord knew of the dangerous electrical problem at the time he leased the premises to the artist and did not disclose it to either the artist or the sculptor, he is liable for any injuries resulting from that condition. (A) is wrong for two reasons: (i) the implied warranty of habitability does not apply to commercial leases; and (ii) even if this were a residential lease, it is doubtful that the investor would be liable for a condition of which she had no knowledge or notice. (A) is wrong for two reasons: (i) the implied warranty of habitability does not apply to commercial leases; and (ii) even if this were a residential lease, it is doubtful that the investor would be liable for a condition of which she had no knowledge or notice.

220
Q

A landlord leased an apartment to a tenant for five years. The lease provided that the landlord will: (i) keep the apartment building at a comfortable temperature 24 hours per day, and (ii) have the carpets cleaned once a year. Two years later, the landlord began turning off the air conditioning at 10 p.m. The tenant’s apartment became hot and stuffy, and she demanded that the landlord honor the covenant. The landlord refused. The following month, the pipes burst in the tenant’s only bathroom, rendering it unusable. The resultant flooding soiled some of the carpeting, which had not been cleaned in the past 12 months. The tenant reported the problems to the landlord, who did not return the tenant’s phone calls.

Which of the following are valid reasons for the tenant to terminate the lease?

A

The tenant will be successful in terminating the lease because the landlord breached the implied warranty of habitability by failing to fix the bathroom pipes.

The general rule at common law was that the landlord was not liable to the tenant for damages caused by the landlord’s failure to maintain the premises during the period of the leasehold.

Today, however, a majority of jurisdictions, usually by statute, provide for an implied warranty of habitability for residential tenancies. In the absence of a local housing code, the standard applied is whether the conditions are reasonably suitable for human residence. If the landlord breaches the implied warranty, the tenant may: (i) terminate the lease, (ii) make repairs and offset their cost against future rent, (iii) abate rent, or (iv) seek damages.

Here, a court is likely to consider the lack of a functioning bathroom as making the premises unsuitable for human residence, allowing the tenant to terminate the lease.

(C - That the landlord did not keep the apartment building at a comfortable temperature 24 hours per day and did not fix the bathroom pipes) would be a stronger answer if the tenant had vacated the premises within a reasonable time. The doctrine of constructive eviction provides that where a landlord does an act or fails to perform some service that he has a legal duty to provide, and thereby makes the property uninhabitable, the tenant may terminate the lease and seek damages.

However, a tenant cannot claim a constructive eviction unless: (i) the injurious acts were caused by the landlord, (ii) the premises are uninhabitable, and (iii) the tenant vacates the premises within a reasonable time.

Here, the landlord’s failing to keep the apartment building at a comfortable temperature 24 hours per day meets conditions (i) and perhaps (ii), but the tenant remains in possession. Therefore, the tenant cannot claim constructive eviction.

The fact that the damage was caused by a third party would be relevant only if the tenant were relying on the judicially developed remedy of constructive eviction, which requires that the damage making the premises uninhabitable have been caused by the landlord. The warranty of habitability is not limited in this way.

221
Q

Which of the following transfers creates an assignment of the lease from T to T2?

If one year into a five-year tenancy for years, T transfers his interest “to T2 for four years; however, if T2 breaches the original lease terms, T may reenter and retake the premises,”

*

If six months into a tenancy at will, T transfers “my entire interest to T2,”

**

If two years into a four-year tenancy for years, T “assigns my entire interest to T2 for one year,” the effect of the transfer is to create

***

If four years into a six-year tenancy for years, T orally transfers his entire interest to T2 for two years,

***

A

the effect of the transfer is an assignment of the lease from T to T2. A complete transfer of the entire remaining lease term constitutes an assignment of the lease. If the tenant retains any part of the remaining lease term, other than a right of reentry for breach of the original lease terms, the transfer is a sublease.

*

the attempted assignment is void and terminates the tenancy at will by operation of law. A tenancy at will is a leasehold estate that is terminable at the will of either the landlord or the tenant. Such a tenancy terminates by operation of law if:

1. Either party dies;

2. The tenant commits waste;

3. The tenant attempts to assign his tenancy;

4. The landlord transfers her interest in the property; or

5. The landlord executes a term lease to a third person.

**

a sublease between T and T2. The label given to the transfer by the parties does not determine whether the transfer is an assignment or a sublease. The nature of the transaction is determined by what interest, if any, the tenant retains. Here, although T “assigned” his interest to T2, he transferred only one of the remaining two years of the lease. Thus, the transfer is a sublease rather than an assignment.

***

the attempted assignment is ineffective under the Statute of Frauds. Most states require that a lease creating a tenancy for more than one year, including an assignment of an interest in a lease for more than one year, be in writing to satisfy the Statute of Frauds.

222
Q

If L leases property to T, and L subsequently assigns L’s interest to L2, whom may T hold liable when X, a paramount title holder, ejects T?

A

If L leases property to T, and L subsequently assigns L’s interest to L2, T may hold L or L2 liable when X, a paramount title holder, ejects T.

A landlord may assign the rents and reversion interest that he owns.

The assignee is liable to the tenants for performance of all covenants made by the original landlord in the lease, provided that those covenants run with the land.

The original landlord also remains liable on all of the covenants he made in the lease.

X’s evicting T from the entire leased premises breaches the covenant of quiet enjoyment, which runs with the land. Thus, L and L2 are personally liable to T.

223
Q

Rule in Dumpor’s Case

A

If a landlord consents to one transfer that violates a covenant against assignment or sublease, he waives his right to avoid future transfers.

The landlord may reserve the right to avoid future transfers, but such reservation must take place at the time of granting consent.

*

A covenant against assignment does NOT prevent the tenant from subleasing her interest. Covenants against assignment or sublease are strictly construed against the landlord. Thus, a covenant prohibiting assignment does not prohibit subleasing and vice versa.

*

If a tenant transfers her interest in violation of a covenant against assignment or sublease, the transfer is NOT void. However, the landlord usually may terminate the lease under the lease terms or a statute or sue for damages.

224
Q

Does a landlord’s promise in a lease to maintain the property terminate because the property is sold?

A

No.

Although no longer in privity of estate, the original landlord and tenant remain in privity of contract, and the original landlord remains liable on the covenant unless there is a novation. A novation substitutes a new party for an original party to the contract. It requires the assent of all parties and completely releases the original party.

When leased property is sold, the purchaser may be liable for his predecessor’s promises if the promise runs with the land. A covenant in a lease runs with the land if the parties to the lease so intend and the covenant touches and concerns the land. Generally, promises to do a physical act, such as maintain or repair the property, are considered to run with the land. Thus, the buyer is liable because he is in privity of estate with the auctioneer and the covenant to repair runs with the land. Consequently, both the landlord and the buyer are potentially liable to the auctioneer for the repairs.

225
Q

Restraints on alienation are traditionally strictly construed.

A

Thus, a covenant prohibiting assignment does not prohibit subleasing and vice versa.

226
Q

Easement implied by operation of law (“quasi-easement”).

A

An easement may be implied if,

prior to the time the tract is divided,

a use exists on the “servient part” that is reasonably necessary for the enjoyment of the “dominant part,”

and a court determines that the parties intended the use to continue after division of the property.

To give rise to an easement, a use must be apparent and continuous at the time the tract is divided.

In this case, the landowner used the servient part of his property (the southern parcel) to run an overhead power line to the dominant part of his property (the northern parcel). Overhead wires are clearly visible and would be readily discoverable on reasonable inspection. The lines are, therefore, apparent. The use must also be reasonably necessary. Whether a use is reasonably necessary depends on many factors, including the cost and difficulty of the alternatives. This use was reasonably necessary to the enjoyment of the dominant parcel because electricity is important to the enjoyment of the property, and the cost (100 times as much) and difficulty of the alternatives are excessive. Thus, the fact that the use of the southern parcel is reasonably necessary would bolster the brother’s case.

227
Q

A landowner and her neighbor owned adjoining parcels of land. The landowner’s property was situated to the west of the neighbor’s property. A highway ran along the east of the neighbor’s property. Twelve years ago, the landowner asked the neighbor if it would be all right for the landowner to use an eight-foot strip along the northern part of the neighbor’s land to access the highway. The only other way for the landowner to get to the highway was to use a one-lane unpaved road that meandered through the woods for two miles. The neighbor agreed, and the landowner used the strip of land regularly to access the highway. The statutory period for adverse possession in this jurisdiction is 10 years.

What is the landowner’s interest in the neighbor’s eight-foot strip of land?

A

The landowner’s interest in the neighbor’s eight-foot strip of land is not an easement. In effect, the landowner only has a “license” (i.e., a revocable privilege) to use the land.

Because an easement is an interest in land, the Statute of Frauds applies. Here, the agreement between the landowner and the neighbor was not in writing; thus, the Statute of Frauds requirements for the creation of an express easement were not met.

An easement by necessity is created when the owner of land sells a part of it and deprives the part sold of access to the public road.

Here, the facts do not indicate that the landowner’s and the neighbor’s parcels were once part of a common tract, and the landowner has an alternate, albeit inconvenient, way to access the highway—the one-lane road.

Landowner’s use of the land was permissive. To acquire a prescriptive easement, the use must be open and notorious, adverse, and continuous and uninterrupted for the statutory period. Although the landowner used the strip for the requisite 10-year period, she does not meet the adverse requirement necessary to obtain a prescriptive easement.

228
Q

Who has the right to choose the location of an easement by necessity?

A

the holder of the servient estate

An easement by necessity arises when the owner of a tract of land sells a part of the tract and by this division deprives one lot of access to a public road or utility line. The owner of the servient parcel has the right to locate the easement, provided the location is reasonably convenient.

229
Q

When the owner of an easement uses it in a way that exceeds its legal scope (i.e., the easement is surcharged),

A

the servient landowner may enjoin the excess use and possibly collect damages.

230
Q

An easement is deemed appurtenant when

A

the right of special use benefits the easement holder in her physical use or enjoyment of another tract of land. The land subject to the easement is the servient tenement, while the land having the benefit of the easement is the dominant tenement. The benefit of an easement appurtenant passes with transfers of the benefited land, regardless of whether the easement is mentioned in the conveyance. All who possess or subsequently succeed to title to the dominant tenement are entitled to the benefit of the easement.

231
Q

A covenant touches and concerns the land when it

A

makes the land itself more useful or valuable to the benefited party.

Here, an agreement to purchase electrical power only from a specified source probably does not touch and concern the land.

232
Q

Is common development scheme necessary for an equitable servitude?

A

No. Generally, equitable servitudes are created by covenants contained in a writing that satisfies the Statute of Frauds. Negative equitable servitudes that may be implied from a common scheme for development are one exception to the writing requirement.

233
Q

When a subdivision is created with similar covenants in all deeds, there is a mutual right of endorsement (each lot owner can enforce against every other lot owner) if two things are satisfied:

A

(i) a common scheme for development existed at the time that sales of parcels in the subdivision began; and (ii) there was notice of the existence of the covenant to the party sued.

Here, there was a common scheme evidenced by the recorded plan, and the fact that the covenant was in the architect’s chain of title gave her constructive notice of the restriction. Therefore, not only does the covenant apply to the architect’s land, but the lawyer (or any other lot owner) can enforce it as a reciprocal negative servitude.

234
Q

When a developer subdivides land into several parcels and some of the deeds contain negative covenants but some do not, negative covenants or equitable servitudes binding all the parcels in the subdivision may be implied under the doctrine of “reciprocal negative servitudes.” Two requirements must be met before reciprocal negative servitudes will be implied:

A

(i) a common scheme for development, and (ii) notice of the covenants. The second requirement may be satisfied by actual notice, record notice, or inquiry notice.

Here, the buyer has not been given actual notice, and the antenna restriction is not so obvious that the appearance of the neighborhood would provide the buyer with inquiry notice. Finally, the buyer has no record of the restriction in his chain of title to establish record notice. If the buyer had been the first purchaser of the lot, some courts might require him to read all deeds given by a common grantor, but the better view does not require such a search. In any case, the buyer’s grantor here is the homeowner, and the restriction was not contained in her deed; the buyer thus does not have record notice of it and is not bound.

235
Q

A developer created an exclusive residential subdivision. In his deed to each lot, the following language appeared:

“Grantee agrees for himself and assigns to use this property solely as a single-family residence, to pay monthly fees as levied by the homeowners’ association for upkeep and security guard services, and that the backyard of this property shall remain unfenced so that bicycle paths and walkways may run through each backyard, as per the subdivision master plan [adequately described], for use by all residents of the subdivision.”

The developer sold lots to an actuary, a baker, and a coroner. All deeds were recorded. The subdivision was developed without backyard fences, with bicycle paths and walkways in place in accordance with the general plan. The actuary in turn sold to an accountant by a deed that omitted any mention of the covenants above, and the accountant had no actual knowledge thereof. Shortly thereafter, the accountant started operating a tax preparation business out of his home. The baker in turn sold to a barber, who knew of, but refused to pay, the monthly fees levied by the homeowners’ association. The coroner leased her property for 10 years to a chiropractor, who erected a fence around the backyard, unaware of the covenant against such fencing.

According to common law principles, which of the following statements is correct?

A

If the barber sues the chiropractor to remove her backyard fence, the barber would win because the covenant regarding fencing is enforceable against the chiropractor as an equitable servitude.

An equitable servitude is a covenant that, regardless of whether it runs with the land at law, equity will enforce against the assignees of the burdened land who have notice of the covenant.

The benefit of an equitable servitude runs to successors if: (i) the original parties so intended, and (ii) the servitude touches and concerns the land. The burden runs if (i) and (ii) are met and (iii) the subsequent purchaser has actual or constructive notice of the covenant. Privity of estate is not needed to enforce an equitable servitude because it is enforced not as an in personam right against the owner of the servient tenement, but as an equitable property interest in the land itself.

Here, the original parties intended for the fencing covenant to be enforceable by and against assignees, as shown by the specific language of the covenant (“Grantee agrees for himself and assigns”) and its purpose to provide bicycle paths and walkways running through each backyard for the use of all subdivision residents. The benefit of the covenant touches and concerns the barber’s property because it increases his enjoyment thereof by providing him with such paths and walkways. Therefore, the barber is entitled to enforce the covenant. The burden of the covenant touches and concerns the land occupied by the chiropractor because it restricts the landholder in her use of the parcel (i.e., her rights in connection with the enjoyment of the land are diminished by being unable to fence in the backyard). The chiropractor will be deemed to have inquiry notice of the restriction because the subdivision is sufficiently developed in accordance with a general plan for the subdivision. Moreover, any neighbor in a subdivision can enforce a covenant contained in a subdivision deed if a general plan existed at the time he purchased his lot. As has been noted, the maintenance of access to all backyards for use as bike paths and walkways was part of such a general plan. Finally, the fact that the chiropractor did not succeed to the coroner’s entire estate, but rather a leasehold interest, is irrelevant because privity is not required to enforce an equitable servitude. Therefore, all of the requirements are in place for the existence of an equitable servitude, which can be enforced by the barber against the chiropractor.

(A) is incorrect because there is privity between the developer and the accountant. There was horizontal privity between the original covenanting parties because, at the time the actuary entered into the covenant with the developer, they shared an interest in the land independent of the covenant (i.e., they were in a grantor-grantee relationship). The accountant holds the entire interest held by the actuary at the time the actuary made the covenant; thus, there is vertical privity.

(B) is incorrect because the remedy sought is the payment of money. Breach of a real covenant, which runs with the land at law, is remedied by an award of money damages, whereas breach of an equitable servitude is remedied by equitable relief, such as an injunction or specific performance. Because the homeowners’ association seeks to obtain from the barber the payment of money, it is inaccurate to refer to this as a situation involving an equitable servitude.

(D) is incorrect because, as explained above, if equitable relief is sought, the covenant must be enforced as an equitable servitude rather than a real covenant.

236
Q

Regardless of whether an adjacent landowner is encroaching on the seller’s land or vice versa, an encroachment will not render title unmarketable if:

A
  1. It is very slight (only a few inches) and does not inconvenience the owner on whose land it encroaches;
  2. The owner encroached upon has indicated that he will not sue on it; or
  3. It has existed for so long (many decades) that it has become legal by adverse possession (if the state recognizes adverse possession title as marketable).

In contrast, a significant encroachment constitutes a title defect that renders title unmarketable.

237
Q

If the buyer determines that the seller’s title is unmarketable, he must notify the seller and give her a reasonable time to cure, even if this requires extending the closing date, and even if

A

time is of the essence.

238
Q

When a seller of land breaches the implied covenant of marketable title and fails to cure, the buyer may

A
  • rescind the contract,
  • sue for damages for breach,
  • get specific performance with abatement of the purchase price, or
  • (in some jurisdictions) require the seller to quiet title.

A court also may order rescission before the delivery date of an installment land contract if the buyer shows that the seller cannot possibly cure the defects in time. In contrast with the buyer’s remedies, the seller CANNOT obtain specific performance or damages (unless the seller cures the title defect within a reasonable time).

When a seller of land breaches the implied covenant of marketable title, the buyer may NOT sue for breach after closing. The implied covenant of marketable title applies at the contract stage of a land sale transaction, before the closing (i.e., exchange of purchase price and deed). The closing extinguishes the contract, which is said to merge with the deed. Then, absent fraud, the seller is no longer liable on this implied covenant; the buyer must rely on any assurances made in the deed.

239
Q

the state of mind of the adverse possessor is

A

irrelevant

240
Q

A majority of courts have held that a beneficial easement that was

A

visible or known to the buyer does not constitute an encumbrance.

241
Q

O conveys life estate to A with a remainder to B. If during A’s lifetime, X enters into actual, exclusive possession that is open and notorious and hostile for the statutory period, will X obtain title to the land?

A
Yes, if X had color of title

B
No, because land subject to a future interest cannot be acquired by adverse possession

C
No, but X will acquire A’s life estate

D
Yes, because X satisfied the elements of adverse possession

A

No, X will not obtain title to the land. If during A’s lifetime, X enters into actual, exclusive possession that is open and notorious and hostile for the statutory period, X will not obtain title to the land, but X will acquire A’s life estate.

If a landowner does not commence an action to eject a would-be adverse possessor before the statute of limitations expires, she is barred from suing for ejectment, and title vests in the possessor.

However, the statute of limitations does not run against the holder of a future interest (e.g., remainder, reversion) until her interest becomes possessory. The future interest holder has no right to possession until the prior present estate terminates, and thus no cause of action for ejectment that accrues until that time.

Here, X will acquire A’s life estate by adverse possession (i.e., a life estate pur autre vie, measured by A’s life), but not B’s remainder, which remains nonpossessory while A is living. Thus, upon A’s death, X’s interest will terminate.

242
Q

If an adverse possessor uses land in violation of a recorded real covenant for the limitations period,

A

takes title free of the real covenant. The nature of the title obtained through adverse possession depends on the occupier’s activities on the land. If an adverse possessor uses the land in violation of a real covenant (i.e., a written promise to do or refrain from doing something on the land), she takes title free of the covenant EVEN IF she had knowledge of it. However, if she complies with the covenant for the statutory period, she takes title subject to the real covenant.

243
Q

Color of title merely means

A

possession of a document purporting to convey title. (So one may obtain color of title through a forged deed.) Color of title is not usually necessary to gain title by adverse possession. In most jurisdictions, the possessor need not believe she has a right to possession; she can be a trespasser.

244
Q

Must a junior mortgagee be named as a party to a senior mortgagee’s foreclosure action?

A

Yes, a junior mortgagee must be named as a party to a senior mortgagee’s foreclosure action because it has the right to pay off the senior mortgage to avoid being wiped out by foreclosure. Foreclosure destroys interests ( e.g., liens, mortgages, leases, easements) junior to the mortgage being foreclosed. Thus, if a senior mortgage is in default, a junior mortgagee has the right to pay it off (i.e., redeem it) to avoid being wiped out by its foreclosure. Failure to join the junior mortgagee results in the preservation of its interest despite foreclosure and sale. In contrast, those with interests senior to that of the foreclosing party are not necessary parties because their interests are not affected by foreclosure. The buyer at the sale takes subject to senior interests, which remain on the land.

245
Q

The order of priority for allocating mortgage foreclosure sale proceeds is as follows, from first to last:

A

1. Expenses of the sale, including attorneys’ fees, and court costs;

2. The principal and accrued interest on the foreclosing party’s loan;

3. Any junior lienors in the order of their priority; and then

4. The mortgagor.

In many cases, no surplus remains after the principal debt is paid off. Senior lienors receive none of the proceeds. Because a senior lien remains on the property (i.e., may itself be foreclosed in the future), a senior lienor is not entitled to any of the money from the sale, even if there is a surplus.

246
Q

A landowner in fee simple signed a promissory note for $10,000 to a bank, and secured the note by a mortgage of her land to the bank. The mortgage was duly recorded. The landowner then sold the property to an attorney, who assumed and agreed to pay the mortgage to the bank on the land. The attorney did not make payments on the mortgage note to the bank. The bank, following appropriate statutory procedures, foreclosed the mortgage and gave notice to both the landowner and the attorney that it intended to sue for any deficiency. At the foreclosure sale, the property sold for $6,000. The bank now sues both the landowner and the attorney for $5,000, which is the remaining amount of the unpaid principal and interest on the note plus costs of foreclosure.

Against which party will the bank be successful in obtaining a judgment?

A

The bank will be successful in obtaining a judgment against both the landowner and the attorney, although it may only collect once. When a grantee assumes the mortgage, the grantee expressly promises the grantor-mortgagor that he will pay the mortgage obligation as it becomes due. The mortgagee then becomes a third-party beneficiary of the grantee’s promise to pay and can sue the grantee directly if the grantee fails to pay. After the assumption, the grantor-mortgagor becomes a surety who is secondarily liable to the mortgagee on the note if the grantee fails to pay. The landowner and the attorney are jointly liable, even though the attorney is primarily liable and the landowner is secondarily liable as a surety.

247
Q

An owner obtained a loan of $60,000 from a bank in exchange for a promissory note secured by a mortgage on his land, which the bank promptly and properly recorded. A few months later, the owner obtained another loan of $60,000 from a lender, in exchange for a promissory note secured by a mortgage on the land, which the lender promptly and properly recorded. Subsequently, the owner sold the land to a buyer for $150,000 and conveyed a warranty deed. The buyer expressly agreed with the owner to assume both mortgages, with the consent of the bank and the lender. A few years later, the bank loaned the buyer an additional $50,000 in exchange for an increase in the interest rate and principal amount of its mortgage on the land. At that time, the balance on the original loan from the bank was $50,000. Shortly thereafter, the buyer stopped making payments on both mortgages and disappeared. After proper notice to all appropriate parties, the bank instituted a foreclosure action on its mortgage, and purchased the property at the foreclosure sale. At that time the principal balance on the lender’s mortgage loan was $50,000. After fees and expenses, the proceeds from the foreclosure sale totaled $80,000.

Assuming that the jurisdiction permits deficiency judgments, which of the following statements is most accurate?

A

The bank’s original mortgage has priority in the proceeds, followed by the lender’s mortgage, and only the lender can proceed against the owner because the bank modified its mortgage after the owner had transferred to the buyer. Generally, the priority of a mortgage is determined by the time it was placed on the property, and the proceeds of a foreclosure sale will be used to pay off the mortgages in the order of their priority. However, if the landowner enters into a modification agreement with the senior mortgagee, raising its interest rate or otherwise making the agreement more burdensome, the junior mortgage will be given priority over the modification. Thus, if the first mortgage debt is larger because of the modification, the second mortgage gains priority over the increase in the debt. Here, the bank and the buyer modified the original mortgage by increasing the principal amount and the interest rate. This modification is not given priority over the lender’s mortgage, and foreclosure proceeds will not be applied against it because the senior lender’s mortgage was not fully satisfied from the proceeds. With regard to the deficiency, the owner is liable to the lender because when a grantee signs an assumption agreement, becoming primarily liable to the lender, the original mortgagor remains secondarily liable on the promissory note as a surety. Here, the buyer assumed the lender’s mortgage and became primarily liable; however, the owner remained secondarily liable as surety and can be required to pay off the rest of the lender’s mortgage loan_. On the other hand, the owner will not be liable to pay off the balance of the bank’s loan, because when a mortgagee and an assuming grantee subsequently modify the original obligation, the original mortgagor is completely_ discharged of liability. The owner had nothing to do with the modification agreed to by the bank and the buyer that increased the amount of the mortgage debt, and will not be even secondarily liable for that amount.

248
Q

Donees, heirs, and devisees are not BFPs because

A

they do not give value for their interests; i.e., they are not purchasers.

249
Q

notice statute

race-notice statute

pure race statute

A

notice statute: “Any conveyance of an interest in land, other than a lease for less than one year, shall not be valid against any subsequent purchaser for value, without notice thereof, unless the conveyance is recorded.” Under a notice statute, a later purchaser of land will prevail over an earlier grantee if she takes without actual or constructive (e.g., record) notice of the earlier grant.

race-notice statute: “Any conveyance of an interest in land, other than a lease for less than one year, shall not be valid against any subsequent purchaser for value, without notice thereof, whose conveyance is first recorded.” Under a race-notice statute, a later purchaser will prevail over an earlier grantee only if she takes without actual or constructive (e.g., record) notice of the earlier grant and records before he does.

pure race statute: “Any conveyance of an interest in land, other than a lease for less than one year, shall not be valid against any subsequent purchaser whose conveyance is first recorded.” Under a race statute, notice is irrelevant. The first party to record, regardless of the date of her conveyance, wins.

250
Q

A landowner gratuitously conveyed his interest in land to a friend by quitclaim deed. The friend promptly and properly recorded her deed. Six months later, the landowner conveyed his interest in the same land to an investor for $50,000 by warranty deed, which was promptly and properly recorded.

As between the friend and the investor, who has the superior right to title to the land?

A

Because the friend recorded prior to the subsequent conveyance, she has the superior right to title regardless of the type of recording statute. A conveyance that is recorded can never be divested by a subsequent conveyance through operation of the recording statutes. By recording, the grantee gives constructive (or “record”) notice to everyone. Hence, proper recording prevents anyone from becoming a subsequent bona fide purchaser (“BFP”). Because the landowner’s conveyance to the friend was recorded at the time of the landowner’s conveyance to the investor, the investor cannot prevail. The investor will clearly lose under a pure race statute because the friend recorded first. The investor will also lose under notice and race-notice statutes because the conveyance to the friend was recorded at the time of the conveyance to the investor. The investor, therefore, had record notice and cannot claim the protection that these types of statutes provide for subsequent purchasers for value who take without notice. Thus, (A) is correct and (C) is incorrect. _The fact that the friend is merely a donee rather than a BFP does not mean that her recording has no effec_t. It is only the subsequent taker who has to be a BFP rather than a donee to utilize the recording statute. The prior grantee, regardless of her status, protects her interest by recording because it prevents anyone from becoming a subsequent BFP.

The quitclaim/warranty deed distinction does not affect who has title to the land; that status merely affects the parties’ respective causes of action and ability to recover against the landowner.

251
Q

A homeowner leased his home to a tenant for three years. The following year, the homeowner conveyed the house to a buyer, who never recorded her deed nor did anything with regard to the house. The tenant continued paying rent to the homeowner. Three months after the conveyance to the buyer, the homeowner conveyed the property to his cardiologist, who knew nothing of the prior conveyance to the buyer. The homeowner took the cardiologist’s money and skipped town. The cardiologist told the tenant that he now owned the house and that all rents should be paid to him. The tenant complied.

Six months later, the cardiologist went to his local bank for a loan. He offered to put up the property as security. The bank discovered that the cardiologist had never recorded his deed and that, just two weeks prior to his loan application, the buyer had recorded a deed to the house that bore an earlier date than the deed the cardiologist had shown the bank. Because of this cloud on the title, the bank refused the loan request. When the tenant discovered this, she quit paying rent to the cardiologist. The state has a recording statute that provides, “a conveyance of an interest in land, other than a lease for less than one year, shall not be valid against any subsequent purchaser for value, without notice thereof, unless the conveyance is recorded.”

If the cardiologist sues the tenant to compel the payment of rent, is the cardiologist likely to win?

A

The cardiologist will likely win. When the cardiologist bought the property from the homeowner, he was a bona fide purchaser who gave value and who had no notice of the earlier sale to the buyer. Not only did the cardiologist not have actual notice of the earlier sale, he did not have constructive notice either because the buyer did not record before the cardiologist bought. The recording statute in the jurisdiction is a notice statute. In a jurisdiction with a notice recording statute, a subsequent purchaser who gives value and takes without notice wins over the earlier grantee. If the facts had shown a jurisdiction with a race-notice recording act, the cardiologist would have been in trouble. With race-notice, the cardiologist would not only have to take without notice, he would have to be the first to record. Because the facts do not show that the cardiologist recorded at all, he would lose. But because this is a notice act jurisdiction, the fact that the buyer finally recorded before the cardiologist is irrelevant.

252
Q

To satisfy a debt owed to a creditor, a son executed and delivered to the creditor a warranty deed to a large tract of undeveloped land. The creditor promptly recorded the deed. Shortly thereafter, she built a house on the property and has lived there ever since. The son never actually owned the land. It belonged to his father, but the father had promised to leave the property to the son.

Later, the father died and his will devised the property to the son. Pressed for money, the son then sold the land to an investor by warranty deed, which the investor promptly recorded. Although the investor paid full value for the property, he purchased it strictly for investment and never visited the site. He therefore did not realize that the creditor was living there, and knew nothing of the son’s earlier deed to the creditor.

The jurisdiction in which the land is located has the following statute: “A conveyance of an estate in land (other than a lease for less than one year) shall not be valid against any subsequent purchaser for value without notice thereof unless the conveyance is recorded.”

Which of the following is the most likely outcome of a quiet title action brought by the creditor against the investor?

A

The creditor will prevail in a suit to quiet title because the investor had notice of the creditor’s interest in the property and, thus, is not a bona fide purchaser for value. When a grantor purports to convey property that he does not own, his subsequent acquisition of title to that property vests in the grantee under the doctrine of estoppel by deed.

Most courts, however, hold that this is personal estoppel, which means that title inures to the grantee’s benefit only as against the grantor, not a subsequent bona fide purchaser.

If the grantor transfers his after-acquired title to an innocent purchaser for value, the bona fide purchaser gets good title.

There is a split of authority as to whether the original grantee’s recordation of the deed imparts sufficient notice to prevent a subsequent purchaser from being a bona fide purchaser, but the majority view is that it does not because it is not in his chain of title. Thus, it is not the fact that the creditor recorded that prevents the investor from being a bona fide purchaser. The fact that the creditor built a home and was living on the property gave the investor constructive notice of her interest. A title search is not complete without an examination of possession. If the possession is unexplained by the record, the subsequent purchaser is charged with knowledge of whatever an inspection of the property would have disclosed and anything that would have been disclosed by inquiring of the possessor. Therefore, the investor is charged with knowledge of the creditor’s possession and with what the creditor would have told him about her possession; i.e., that the property was conveyed to her by the son prior to his conveyance to the investor. Consequently, the investor does not qualify as a bona fide purchaser, and (C) is an incorrect choice.

253
Q

On February 10, an owner took out a $10,000 mortgage on her land with a bank. On February 15, the owner conveyed the land for $50,000 to a buyer who was not aware of the mortgage. On February 17, the bank recorded its mortgage interest in the land. On February 21, the buyer recorded his deed to the land.

Does the buyer hold the land subject to the bank’s mortgage?

A

The buyer takes subject to the bank’s mortgage in a race-notice jurisdiction because it was recorded first.

All recording acts apply to mortgages as well as deeds. Thus, a subsequent purchaser of the property will take subject to a prior mortgage unless the recording act changes the result.

A race-notice recording act would change this result only where a subsequent purchaser did not have notice of the mortgage at the time of purchase and recorded his deed before the mortgage was recorded. Here, the buyer did not have notice of the mortgage but he recorded after the bank; thus, he takes subject to the bank’s interest.

254
Q

To secure a loan of $100,000 from a bank, the owner in fee simple of a parcel of land conveyed a deed of trust for the land to the bank. The deed of trust contained a “power of sale” clause, permitted by the jurisdiction, which allowed the bank to sell the property in the event of default without the necessity of a judicial foreclosure action. After several years, the owner defaulted on his loan payments to the bank. The bank informed the owner that it was exercising its power of sale. After appropriate notices, the bank conducted a public sale of the land. The bank was the sole bidder and obtained the property for $80,000, which was $10,000 less than the outstanding balance on the loan plus the expenses of the sale. One month later, the owner notified the bank that he wanted to pay off the loan and extinguish the deed of trust, and was prepared to tender $80,000 to do so. The bank insisted that the owner must tender $90,000 to pay off the loan.

If a court in the jurisdiction will require the bank to accept only $80,000 under the circumstances above, what is the likely reason?

A

If the owner can compel the bank to accept his offer, it will be because he has a statutory power to redeem the property after the foreclosure sale has occurred.

In all states, the equity of redemption provides the borrower with an equitable right, at any time prior to the foreclosure sale, to redeem the land or free it of the mortgage or lien by paying off the amount due or, if an acceleration clause applies, the full balance due. Only about half the states, however, give the borrower a statutory right to redeem for some fixed period after the foreclosure sale has occurred; the amount to be paid is generally the foreclosure sale price, rather than the amount of the original debt. Thus, if the owner can redeem the land for $80,000, it will be based on the jurisdiction’s statutory power of redemption.

255
Q

A developer subdivided a 25-acre tract of land into 100 quarter-acre lots. On each lot she built a two-unit townhouse. The deeds to each of the purchasers contained a covenant that “the grantee, his heirs and assigns” would use the property only for single-family use. All deeds were promptly and properly recorded. Subsequently, the zoning laws were amended to allow multifamily use within the subdivision. Six months later, a social worker offered to purchase an original owner’s unit that was for sale. The social worker informed the owner that she planned to operate a halfway house out of the unit, an activity in conformity with the applicable zoning regulations. Therefore, the owner did not include the single-family restriction in the deed to the social worker.

If a neighbor, who purchased his lot from the developer, seeks to enjoin the operation of the halfway house, will he succeed?

A

The neighbor will succeed in enjoining the operation of the halfway house because the social worker had notice of the restrictive covenant.

A covenant runs with the land to a subsequent purchaser with notice of the covenant if it touches and concerns the land and is intended to run. Notice may be actual or constructive.

Here, the social worker was on record notice of the covenant because the original owner’s deed was recorded. Restricting land to single-family use touches and concerns the land, and it is evident that the developer and the original owners, including the neighbor, intended it to run with the land by use of the language “grantee, his heirs and assigns.” The social worker thus will be bound even though her deed did not refer to the covenant.

(B) is incorrect because compliance with zoning regulations does not excuse noncompliance with an enforceable covenant; both must be complied with.

(D) is incorrect because the neighbor can prevail without needing to show a servitude implied from a common scheme, which comes into play when a developer subdivides land into several parcels and some of the deeds contain negative covenants and some do not. Here, the covenant relating to single-family use was in all of the original deeds and, as discussed above, it runs with the land. A covenant that runs with the land may be enforced as an equitable servitude if the assignees of the burdened land have notice of the covenant; the usual remedy is an injunction. Here, the social worker had record notice of the covenant and it runs with the land, so the neighbor can enforce the covenant as an equitable servitude without resort to implying a reciprocal negative servitude.

256
Q

A landlord leased office space to a business owner for five years, ending on November 1, reserving a yearly rent of $24,000, payable monthly. On October 1 of the fifth year, the business owner notified the landlord that he was preparing to move, but would greatly appreciate if the landlord could extend the lease for a month or two. On October 10, the landlord wrote to the business owner that she thought they could reach a satisfactory arrangement, but did not hear back from the business owner. The business owner did not vacate the office until November 20. On November 30, the landlord received a check from the business owner in the amount of $1,333 for “November’s rent” and a note that he had vacated the premises.

If the landlord brings an action against the business owner for additional rent, how will the court rule?

A

The court will rule that the business owner is bound to a year-to-year tenancy because he is a hold-over tenant. When a tenant fails to vacate the premises after the termination of his right to possession, the landlord may: (i) treat the hold-over tenant as a trespasser and evict him; or (ii) bind the tenant to a new periodic tenancy. The terms and conditions of the expired tenancy apply to the new tenancy. At least in commercial leases, the new tenancy will be year-to-year if the original lease term was for one year or more. Here, the businessman was a tenant for years because his lease was for a five-year fixed period of time. A tenancy for years ends automatically on its termination date. Therefore, as of November 1, the business owner became a hold-over tenant and the landlord had a right to bind him to a new periodic tenancy. Because the original lease was for more than one year, the business owner may be held to a year-to-year tenancy, at the stipulated rent of $24,000 per year.

257
Q

A landlord owned a prestigious downtown office building. A law firm leased the entire building from the landlord for a term of 20 years. The lease included a provision that taxes on the building would be paid by “the lessee, his successors, and assigns.” The law firm occupied the building and paid the rent and taxes for eight years. At the end of the eight-year period, the law firm assigned the balance of the lease to an accounting firm and vacated the premises. The assignment was written, but there was no provision concerning the accounting firm’s assumption of the duties under the lease.

The accounting firm occupied the building and paid the rent and taxes for five years. At the end of the five-year period, the accounting firm subleased the building for five years to an investment company and vacated the premises. The sublease was written, but there was no provision concerning the investment company’s assumption of the duties under the lease. The investment company now occupies the building and has paid the rent but not the taxes. The landlord has sued all three (i.e., the law firm, the accounting firm, and the investment company) for failure to pay the taxes.

The landlord should prevail against whom?

A

The law firm and the accounting firm are liable. After an assignment, the original tenant is no longer in privity of estate with the landlord. However, a tenant may still be held liable on its original contractual obligations to the landlord on privity of contract grounds. Here, the law firm is liable because it made the original deal with the landlord, which included the obligation to pay taxes on the building. The law firm remains in privity of contract with the landlord throughout the term of the lease unless it is otherwise discharged. In an assignment, the assignee stands in the shoes of the original tenant in a direct relationship with the landlord. Each is liable to the other on all covenants in the lease that run with the land, which would include the obligation of the lessee to pay taxes on the property. Here, the accounting firm is liable because as an assignee it is in privity of estate with the landlord. The accounting firm remains in privity of estate until it assigns to someone else. The sublease to the investment company is not an assignment. A sublessee is not personally liable to the landlord for rent or for the performance of any other covenants made by the original lessee in the main lease (unless the covenants are expressly assumed) because the sublessee does not hold the tenant’s full estate in the land (so no privity of estate). Here, the investment company is not liable because, as a nonassuming sublessee, it is not in privity of contract or estate with the landlord.

258
Q

An environmentalist divided her 25-acre property into 100 quarter-acre residential lots. At the time the environmentalist sold her lots, there was a recycling center about one mile from the western boundary of the development. She included in the deed of all 100 grantees the following provision:

“Grantee covenants for herself and her heirs and assigns that all aluminum cans, glass bottles, and grass clippings of Grantee and her heirs and assigns shall be recycled. This covenant runs with the land and shall remain in effect as long as there is a recycling center within five statute miles of the development.”

A buyer purchased a lot in the development. Her deed, which contained the recycling clause, was duly recorded. Two years later, the buyer decided to give the property to her niece as a gift. The niece’s deed to the property contained the recycling covenant, and she too recorded her deed. Shortly after the niece took possession of the house, the recycling center moved its location to a new site about four and a half miles from the development. When the niece put the house up for sale, she said nothing to prospective buyers about recycling.

The house was purchased by a veteran who had lost the use of his legs. The veteran’s deed did not contain the recycling clause, and he hired a local disposal service to carry away his garbage and a landscaper to maintain the yard. The landscaper bagged the grass clippings and they were removed by the disposal service, which put all the trash and clippings in a landfill. When the veteran’s neighbors informed him of his duty to recycle, he told them that he knew nothing of the covenant and that it would be difficult for a person in his physical condition to haul cans, bottles, and clippings to the recycling center. Unfazed, the neighbors filed suit to require the veteran to comply with the covenant or pay damages.

The veteran’s best defense is which of the following?

A

The veteran’s best defense is that the covenant does not clearly “touch and concern” the land. While recycling may benefit the community at large, “touch and concern” involves the relationship between landowners at law. Recycling by the veteran does not directly benefit the other landowners in the use and enjoyment of their land. Thus, (B) is correct. (A) is wrong because even though the veteran’s deed does not contain the covenant, he has record notice because the restriction is in his chain of title.

259
Q

A vendor entered into a written contract with a purchaser for the sale of a large tract of land. The contract set forth an accurate metes and bounds description of the land based on a professional survey. At closing, the purchaser discovered that the deed was incorrectly transcribed and did not agree with the description of the land in the contract. The deed described the property to be conveyed as follows:

“(i) from the southwest corner of [a specified starting point], proceed South 45 degrees East 200 feet to [a specified point]; (ii) from that point, proceed South 45 degrees West 100 feet to [a specified point]; (iii) from that point, proceed North 45 degrees West 200 feet to [a specified point]; and (iv) from that point, proceed South 45 degrees East 100 feet to the starting point.”

The purchaser refused to proceed with the closing and brought an action to reform the deed to make it conform to the intention of the parties.

Which of the following corrections should be made for the deed to properly describe the land?

A

Direction (iv) needs to be corrected in its course but not its distance. In land contracts and deeds, property may be described in various ways as long as the description is unambiguous. From a designated starting point that can be identified by reference to a government survey or a natural or artificial monument, the boundaries of the property can be described by successive calls of courses (e.g., angles) and distances until returning to the starting point. A course is a statement of direction generally stated as some number of degrees east or west of due north or south. In each call a distance must be stated together with the course. Thus, the boundary in direction (iv) runs at an angle 45 degrees east of due south (i.e., southeast) for a distance of 100 feet. However, because direction (i) went southeast, direction (ii) went southwest, and direction (iii) went northwest, the fourth direction has to be northeast for a distance of 100 feet to bring the final boundary back to the starting point. (In this type of question, diagram the boundaries as shown below to help you visualize the property.) Therefore, the correction in choice (D) is correct. (A), (B), and (C) are incorrect because none of those proposed changes in distance or direction would be sufficient to bring the final call back to the starting point.

260
Q

A woman purchased a tract of land from a man by warranty deed. Unbeknownst to the woman, the man was not the actual owner of the tract. The woman built a home on the tract and moved into it. Two years later, the actual owner learned of the man’s transaction with the woman and prevented the woman from entering the tract from that point forward. This led to a costly court battle. When the woman notified the man and told him that she thought it was his duty to straighten this out, he ignored her.

The woman would succeed in a suit for damages against the man for breach of which of the following covenants of title?

A

The woman would succeed in a suit for damages against the man for breach of the covenants of seisin, right to convey, quiet enjoyment, warranty, and further assurances, but not on the covenant against encumbrances.

A general warranty deed gives the grantee six covenants of title:

the right to seisin,

the right to convey,

a covenant against encumbrances,

the covenant of quiet enjoyment,

a general warranty

the covenant of further assurances.

Under the covenants of quiet enjoyment, warranty, and further assurances, the man promised that

(i) the woman would not be disturbed in her possession of the tract;
(ii) he would defend the woman’s title against lawful claims; and
(iii) he would perform whatever acts are necessary to perfect the woman’s title.

Because the man neither owned the tract of land nor was acting as the actual owner’s agent, he breached the covenants of seisin and right to convey at the time of the conveyance to the woman.

When the actual owner prevented the woman from re-entering the property, this interfered with the woman’s quiet enjoyment, and the man’s refusal to “straighten this out” was a breach of the covenant of further assurances.

There is nothing in the facts to suggest the property is encumbered; thus, the man did not breach the covenant against encumbrances.

261
Q
A